*NURSING > EXAM > N675 Final Exam :Acute Care Exam 1 Questions and Answers 100% CORRECT (All)

N675 Final Exam :Acute Care Exam 1 Questions and Answers 100% CORRECT

Document Content and Description Below

N675 Final Exam :Acute Care Exam1Questions and Answers Week 1 – EENT A primary care provider notes painless, hard lesions on a patient’s external ears that expel a white crystalline substanc... e when pressed. What diagnostic test is indicated? Rheumatoid factor Endocrine studies Biopsy of the lesions Uric acid chemical profile A patient has painful oral lesions and the provider notes several white, verrucous lesions in clusters throughout the mouth. What is the recommended treatment for this patient? Oral hygiene measures Nystatin oral suspension Surgical excision Oral acyclovir A patient has sore throat, a temperature of 38.5° C, tonsillar exudates, and cervical lymphadenopathy. What will the provider do next to manage this patient’s symptoms? Prescribe empiric penicillin Perform a rapid antigen detection test Refer to an otolaryngologist Order an antistreptolysin O titer A 61 year old male presents with a 12 hour history of extremely painful left red eye. The patient complains of blurred vision, haloes around lights, and vomiting. It began yesterday evening. On exam, the eye is red, tender and inflamed. The cornea is hazy and pupil reacts poorly to light. The most likely diagnosis in this patient is: Macular degeneration Acute angle glaucoma Increased intracranial pressure Detached cornea A patient has recurrent epistaxis without localized signs of irritation. Which laboratory tests may be performed to evaluate this condition? (Select all that apply.) CBC with platelets BUN and creatinine PT and PTT Liver function tests PT/INR A patient reports a feeling of fullness and pain in both ears and the practitioner elicits exquisite pain when manipulating the external ear structures. What is the likely diagnosis? Chronic otitis externa Acute otitis externa Otitis media with effusion Acute otitis media Patient has been diagnosed with acute rhinosinusitis. Symptoms began 3 days ago. Based on the most likely etiology, how should this patient be managed? Azithromycin and decongestant Decongestant and analgesic Levofloxacin Amoxicillin with clavulanate The vast majority of rhinosinusitis is of viral etiology, antibiotics would not be helpful and would only lead to continued antibacterial resistance. If symptoms persist for longer than 10 days, reevaluation is necessary with possible antibiotics at that time A 20-year-old male of Hispanic descent who reports a history of a cold that resolved 2 weeks ago except for a dry cough and pain over his right cheek that worsens when he bends down. The patient denies fever. The patient tells you that he is very allergic to Keflex and erythromycin. Vital signs are stable except temperature is 99.2°F. Which showed the following conditions is most likely? Fever secondary to previous viral URI Acute sinusitis Acute bronchitis Hay fever Patient's symptoms match most closely to acute sinusitis which includes cough, facial pain, low- grade fever An adult patient has epiglottitis secondary to a chemical burn. Which medication will be given initially to prevent complications? Chloramphenicol biopsyrecu Dexamethasone Metronidazole Clindamycin What are factors associated with acute suppurative parotitis? (Select all that apply.) Anticholinergic medications Diabetes mellitus Radiotherapy Hypervolemia Allergies A 39 year old has a sudden onset of painful right red eye. He reports sensitivity to light and the sensation of a foreign body, though his history for a foreign body is negative. He does not wear contact lenses. How should the NP manage this? Observe for 24 hours if visual acuity is normal Treat for bacterial conjunctivitis Treat for viral conjunctivitis Refer to ophthalmology No clear diagnosis can be made from signs/symptoms, but there are several red flags. collectively the red flags necessitate a referral. There is no mention of eye discharge necessary for conjunctivitis. Red flags present point more towards active corneal process although glaucoma should also be a differential. A 17-year-old has a complaint of ear pain. If he has otitis externa, which complaint is most likely/most common? Concurrent URI Fever Difficulty hearing TV Tragal pain A patient has an initial episode otitis external associated with swimming. The patient’s ear canal is mildly inflamed and the tympanic membrane is not involved. Which medication will be ordered? Cipro HC Neomycin Fluconazole Vinegar and alcohol A patient has gingival inflammation with several areas of separate ulceration and a small amount of purulent discharge. What is required to diagnose this condition? Culture and sensitivity Tzank smear Physical examination Microscopic exam of oral scrapings A patient reports tooth pain in a lower molar and the provider notes a mobile tooth with erythema and edema of the surrounding tissues without discharge. Which is the initial course of action by the provider? Recommend oral antiseptic rinses and follow up in one week Perform an incision and drainage of the edematous tissue Prescribe amoxicillin and refer to a dentist in 2 to 3 days Refer to an oral surgeon for emergency surgery A patient presents to your clinic with a painless red eye. Her vision is normal, but her sclera has a blood red area. What is this termed? Conjunctivitis Glaucoma Acute iritis Subconjunctival hemorrhage Which physical examination finding suggests viral rather than bacterial parotitis? Unilateral edema of parotid glands Enlargement and pain of affected glands Gradual reduction in saliva production Clear discharge from Stensen’s duct A child is hit with a baseball bat during a game and sustains an injury to the nose, along with a transient loss of consciousness. A health care provider at the game notes bleeding from the child’s nose and displacement of the septum. What is the most important intervention at this time? Immobilize the child’s head and neck and call 911 Turn the child’s head to the side to prevent aspiration of blood Place nasal packing in both nares to stop the bleeding Apply ice to the injured site to prevent airway occlusion A pediatric patient has otalgia, fever of 38.8° C, and a recent history of upper respiratory examination. The examiner is unable to visualize the tympanic membranes in the right ear because of the presence of cerumen in the ear canal. The left tympanic membrane is dull gray with fluid levels present. What is the correct action? Remove the cerumen and visualize the tympanic membrane Perform a tympanogram on the right ear Recommend symptomatic treatment for fever and pain Treat empirically with amoxicillin 80 to 90 mg/kg/day A patient presents with findings of pain, warmth, redness, and swelling below the inner canthus toward nose. Tearing is present and when pressure is applied to the lacrimal sac, purulent discharge from the puncta is noted. This is suggestive of: Belpharitis A chalazion A hordeolum Dacryocystitis Group A strep pharyngitis: Is commonly accompanied by inflamed uvula Can be accompanied by abdominal pain Is characterized by single symptom Usually does not have exudative symptoms Group A strep is usually accompanied by multiple symptoms with abrupt onset. GI symptoms are common such as nausea, vomiting, no abdominal pain. Inflamed uvula is not common Which symptoms may occur with vestibular neuritis? (Select all that apply.) Nausea and vomiting Disequilibrium Fever Tinnitus Hearing loss A patient who has acute suppurative parotitis has been taking amoxicillin-clavulanate for 4 days without improvement in symptoms. The provider will order an antibiotic for Methicillin-resistant S. aureus. Which other measure may be helpful? Discouraging chewing gum Topical corticosteroids Cool compresses Surgical drainage A NP preforms a fundoscopic exam. He identifies small areas of dull, yellowish-white coloration in the retina. What might these be? Cotton wool spots Hemorrhages Exudates Microaneurysm The provider sees a child with a history of high fever and sore throat. When entering the exam room, the provider finds the child sitting in the tripod position and notes stridor, drooling, and anxiety. What is the initial action for this patient? Administer empiric intravenous antibiotics and steroids Obtain an immediate consultation with an otolaryngologist Perform a thorough examination of the oropharynx Have the child lie down and administer high-flow, humidified oxygen A patient reports ear pain after being hit in the head with a baseball. The provider notes a large perforated tympanic membrane. What is the recommended treatment? Prescribe analgesics and follow up in 1 to 2 days Order antibiotic ear drops if signs of infection occur Refer the patient to an otolaryngologist for evaluation Reassure the patient that this will heal without problems Papilledema is noted n a patient with a headache. What is the importance of papilledema in this patient? it is not related to the headache this is a common finding in patients with headache it is an incidental finding in patient with migraines It could be an important finding in this patient A patient has seasonal rhinitis symptoms and allergy testing reveals sensitivity to various trees and grasses. What is the first-line treatment for this patient? Intranasal steroids Antihistamine sprays Intranasal comolyn Oral antihistamine A patient with gingival inflammation with several areas of separated ulceration and a small amount of purulent discharge. What is required to diagnose this condition? Microscopic exam of oral scrapings Physical examination Tzank smear Culture and sensitivity A 70 year old male has a yellowish, triangular nodule near the iris. This is probably: A chalazion A pinguecula A stye Subconjunctival hemorrhage. A 32-year-old patient is a newly diagnosed diabetic. She has developed a sinus infection. Her symptoms have persisted for 10 days. 6 weeks ago, she was treated with amoxicillin for a URI. It cleared without incident. Which be recommended today? Prescribe amoxicillin again Do not prescribe an antibiotic, only a decongestant as indicated Prescribe amoxicillin-clavulanate today Prescribe a decongestant and an antihistamine A patient has recurrent sneezing, alterations in taste and smell, watery, itchy eyes, and thin, clear nasal secretions. The provider notes puffiness around the eyes. The patient’s vital signs are normal. What is the most likely diagnosis for this patient? Allergic rhinitis Acute sinusitis Viral rhinitis Chronic sinusitis The patient presents with complaints of morning eyelash crusting and itchy red eyes. It began on the left and now has become bilateral. Based on the most likely diagnosis, what should the NP tell the caregivers about this condition? Anterior cervical lymphadenopathy is common This usually begins as a viral infections Pain is normal in the affected eye It produces blurred vision in the affected eye An adolescent has fever, chills, and a severe sore throat. On exam, the provider notes foul-smelling breath and a muffled voice with marked edema and erythema of the peritonsillar tissue. What will the primary care provider do? Perform a rapid strep and throat culture Refer the patient to an otolaryngologist Prescribe empiric oral antibiotics Evaluate for possible epiglottitis Which patient may be given symptomatic treatment with 24 hours follow-up assessment without initial antibiotic therapy? A 4 year old, afebrile child with bilateral otorrhea A 6 month old with fever of 39.2° C, poor sleep and appetite and bulging TM A 36 month old with fever of 38.5° C, mild otalgia, and red, non-bulging TM A 5 year old with fever of 38.0° C, severe otalgia, and red, bulging TM A patient has nasal congestion, fever, purulent nasal discharge, headache, and facial pain and begins treatment with amoxicillin-clavulanate. At a follow-up visit 10 days after initiation of treatment, the patient continues to have purulent discharge, congestion, and facial pain without fever. What is the next course of action for this patient? A referral to an otolaryngologist A trial of azithromycin A CT scan of the paranasal sinuses A second course of amoxicillin-clavulanate A patient reports several episodes of acute vertigo, some lasting up to an hour, associated with nausea and vomiting. What is part of the initial diagnostic workup for this patient? Auditory brainstem testing Electrocochleography Vestibular testing Audiogram and MRI A patient has two palpable, tender, left preauricular nodes that are about 0.5cm in diameter. What condition might this be associated with? Ear infection Conjunctivitis Ulceration on the tongue Sore throat A kindergarten teacher was diagnosed with acute streptococcal pharyngitis. On exam, her throat is bright red color with no tonsillar exudate, and clear mucus is seen on the lower nasal turbinates. The urinalysis shows a large amount of white blood cells and is positive for nitrates. The patient has a significant sulfa allergy and thinks she is also allergic to penicillin. Which with the following is the best treatment choice? Levofloxacin Amoxicillin–clavulanic acid Trimethoprim–sulfamethoxazole Clarithromycin During routine physical exam of an elderly woman, a triangular thickening of the bulbar conjunctiva on the temporal side is noted to be encroaching on the cornea. She denies any eye pain or visual changes. Which of the following is most likely? Corneal arcus Pterygium Chalazion Pinguecula Group A strep pharyngiitis: Can be accompanied by abdominal pain Group A strep is usually accompanied by multiple symptoms with abrupt onset. GI symptoms are common such as nausea, vomiting, no abdominal pain. Inflamed uvula is not common A 70 year old male patient complains of a bright red colored spot that has been present in his left eye for 2 days. He denies eye pain, visual changes, or headaches. He ha a new onset cough from a recent URI. The only medicine he is taking is aspirin 1 tablet daily. Which of the following actions is appropriate follow up for this patient? Refer the patient to an opthalmologist Refer the patient to an optometrist Prescribe an ophthalmic antibiotic solution Advise the patient that the condition is benign and will resolve spontaneously A patient reports painful oral lesions 3 days after feeling pain and tingling in the mouth. The provider notes vesicles and ulcerative lesions on the buccal mucosa. What is the most likely cause of these symptoms? 1. Herpes simplex virus 2. Bacterial infection 3. Candida albicans 4. Human papillomavirus A patient diagnosed with strep throat received a prescription for azithromycin. She has not improved in 48 hours. What course of action is acceptable? 1. Different macrolide antibiotic should be prescribed 2. Penicillin or cephalosporin with beta-lactamase coverage should be considered 3. The patient should wait another 24 hours for improvement 4. That antibiotics should be changed to a first generation cephalosporin A patient is concerned about frequent nasal stuffiness and congestion that begins shortly after getting out of bed in the morning. The patient denies itching and sneezing. A physical examination reveals erythematous nasal mucosa with scant watery discharge. What treatment will the provider recommend for this patient? 1. Consultation for immunotherapy 2. Oral antihistamines each morning 3. Oral decongestants as needed 4. Daily intranasal steroids A patient has bilateral bleeding from the nose with bleeding into the pharynx. What is the initial intervention for this patient 1. Assess airway safety and vital signs 2. Clear the blood with suction to identify site of bleeding 3. Apply firm, continuous pressure to the nostrils 4. Have the patient sit up straight and tilt the head forward A patient presents to your clinic with a painless red eye. Her vision is normal, but her sclera has a blood red area. What is this termed? 1. Acute iritis 2. Conjunctivitis 3. Subconjunctival hemorrhage 4. Glaucoma During a routine physical examination, a provider notes a shiny, irregular, painless lesion on the top of one ear auricle and suspects skin cancer. What will the provider tell the patient about this lesion? It is benign and will not need intervention. Immediate surgery is recommended. This is most likely malignant. A biopsy should be performed. A patient with allergic rhinitis develops acute sinusitis and begins treatment with an antibiotic. Which measure may help with symptomatic relief for patients with underlying allergic rhinitis? Oral mucolytics Intranasal steroids Saline solution rinses Topical decongestants A 32-year-old patient is a newly diagnosed diabetic. She has developed a sinus infection. Her symptoms have persisted for 10 days. 6 weeks ago, she was treated with amoxicillin for a URI. It cleared without incident. Which be recommended today? Prescribed amoxicillin–clavulanate today Prescribed amoxicillin again Prescribe a decongestant and an antihistamine Do not prescribe an antibiotic, only a decongestant as indicated [Rationale: Amoxicillin is not indicated when a beta-lactamase producing organism and suspected, this is the case as patient took antibiotics 6 weeks ago. Amoxicillin–clavulanate is a good choice as it covers a beta-lactamase producers. And bacteria etiology is suspected as symptoms have persisted for 10 days. A decongestant can be added, however this does not substitute antibiotics in this case] A patient has been taking amoxicillin for treatment of a dental abscess. In a follow-up visit, the provider notes edema of the eyelids and conjunctivae. What is the next action? Hospitalize the patient for an endodontist consultation Suggest using warm compresses to the eyes for comfort Recommend follow up with a dentist in 2 to 3 days Prescribe amoxicillin clavulanate for 10 to 14 days A 58 year old farmer presents with a wedge shaped, pinkish, clear growth on the nasal side of his eye. He states that it has been present for a while, but only recently began to feel as if a foreign body was in his eye. This is probably a: Pterygium Xanthelasma Stye Pinguecula A 39 year old has a sudden onset of painful right red eye. He reports sensitivity to light and the sensation of a foreign body, though his history for a foreign body is negative. He does not wear contact lenses. How should the NP manage this? Treat for viral conjunctivitis Refer to ophthalmology Treat for bacterial conjunctivitis Observe for 24 hours if visual acuity is normal No clear diagnosis can be made from signs/symptoms, but there are several red flags. collectively the red flags necessitate a referral. There is no mention of eye discharge necessary for conjunctivitis. Red flags present point more towards active corneal process although glaucoma should also be a differential A child has recurrent impaction of cerumen in both ears and the parent asks what can be done to help prevent this. What will the provider recommend? Removing cerumen with a cotton-tipped swab Try thermal-auricular therapy when needed Use an oral irrigation tool to remove cerumen Clean the outer ear and canal with a sof t cloth A screening audiogram on a patient is abnormal. Which test may the primary provider perform next to further evaluate the cause of this finding? Speech reception test Impedance audiometry Pure tone audiogram Tympanogram A 12-year-old complains of itching in his right ear and pain when the pinna is pulled or the tragus is pushed. Examination reveals slight redness in the ear canal with a clear odorless fluid. This is suggestive of: Otitis externa Ear pain when pinna is pulled or tragus is pushed is indicative of otitis externa. Clear fluid is not indicative of pus formation such as with ruptured tympanic membrane. A patient is suspected of having vestibular neuritis. Which finding on physical examination is consistent with this diagnosis? Vertigo with changes in head position Facial palsy and vertigo Fluctuating hearing loss and tinnitus Spontaneous horizontal nystagmus A 20-year-old male of Hispanic descent who reports a history of a cold that resolved 2 weeks ago except for a dry cough and pain over his right cheek that worsens when he bends down. The patient denies fever. The patient tells you that he is very allergic to Keflex and erythromycin. Vital signs are stable except temperature is 99.2°F. Which showed the following conditions is most likely? Fever secondary to previous viral URI Hay fever Acute bronchitis Acute sinusitis Acute otitis media can be best diagnosed by identifying which otic characteristic(s)? Cloudy, bulging TM with impaired mobility Opacity and erythema of the tympanic membrane Decreased mobility of the tympanic membrane Marked redness of the tympanic membrane Cloudy, bulging TM with impaired mobility is the best predictor for AOM. Decreased mobility of TM can be a result of fluid behind the TM, as in middle ear effusion. A patient complains of otalgia and difficulty hearing from one ear. The provider performs an otoscopic exam and notes a dark brown mass in the lower portion of the external canal blocking the patient’s tympanic membrane. What is the initial action? Ask the patient about previous problems with that ear. During an eye exam of a 50 year old hypertensive patient who is complaining of an onset of a severe headache, you find that the borders of the disc margins on both eyes are blurred. What is the name of this clinical finding? Papilledema. A provider is recommending a cerumenolytic for a patient who has chronic cerumen buildup. The provider notes that the patient has dry skin in the ear canal. Which preparation is FDA approved for this use? Carbamide peroxide Mineral oil Hydrogen peroxide Liquid docusate sodium A patient reports a sudden onset of sore throat, fever, malaise, and cough. The provider notes mild erythema of the pharynx and clear rhinorrhea without cervical lymphadenopathy. What is the most likely cause of these symptoms? Viral pharyngitis Infectious mononucleosis Allergic pharyngitis Group A streptococcus A 93 year old demented adult has been recently treated for an upper respiratory infection but drainage from the right nostril persists. What should the NP suspect? Presence of foreign body Allergic rhinitis Unresolved URI Dental caries The NP preforms a fundoscopic exam on a patient who has recently been diagnosed with hypertension. What is the significant of AV nicking? This is indicative of long-standing hypertension This is a normal variant This is indicative of retinal detachment The patient should be screened for diabetes While checking for the red reflex on a 3 year old boy during a well child visit, the NP notes a white reflection on the child's left pupil. Which of the following conditions should be ruled out? Retinoblastoma of the lef t eye Color blindness of the left eye Unilateral strabismus Unilateral cataracts Papilledema is noted in a patient with a headache. What is the importance of papilledema in this patient? It could be an important finding in this patient it is an incidental finding in patient with migraines this is a common finding in patients with headache it is not related to the headache Papilledema could represent swelling of the optic nerve head and disc secondary to increased intracranial pressure (ICP). The cardinal symptom of ICO is a headache; papilledema is a secondary finding. It is not a common finding in patient with general headache A provider performs a nasal speculum examination on a patient who sustained nasal trauma in a motor vehicle accident. The provider notes marked swelling of the nose, instability and crepitus of the nasal septum with no other facial bony abnormalities and observes a rounded bluish mass against the nasal septum. Which action is necessary at this time? Urgent drainage of the mass CT scan of facial structures Ice packs to reduce facial swelling Surgery to reduce the nasal fracture Week 2 – Pulmonary A young adult, previously healthy clinic patient has symptoms of pneumonia including high fever and cough. Auscultation reveals rales in the left lower lobe. A chest radiograph is normal. The patient is unable to expectorate sputum. Which treatment is recommended for this patient? Empiric treatment with a macrolide antibiotic A 22 year old tall, thin and athletic man comes into your primary care clinic complaining of pain with breathing and progressively worsening shortness of breath. In order of sequence, what will be your next steps? -Obtain more history, auscultate the lungs and send pt to the ER for a stat CXR and further evaluation. -Obtain more history, and immediately send him to the ER A patient reports coughing up a small amount of blood after a week of cough and fever. The patient has been previously healthy and does not smoke or work around pollutants or irritants. What will the provider suspect as the most likely cause of this patient’s symptoms? Infection Patients with pneumonia reports that he has rust-colored sputum. With pathogen should the nurse practitioner suspect? Streptococcus pneumoniae Clinical description of mucus does not really help and clinical decision making regarding pneumonia, but certain clinical characteristics are associated with specific types of pneumonia. Scant or watery sputum is associated with atypical pathogens like mycoplasma and clamydophila. Thick, discolored sputum may be associated with bacterial pneumonia. A young adult patient without a previous history of lung disease has an increased respiratory rate and reports a feeling of “not getting enough air.” The provider auscultates clear breath sounds and notes no signs of increased respiratory effort. Which diagnostic test will the provider perform initially? Complete blood count A 65-year-old patient who has not had an influenza vaccine is exposed to influenza and comes to the clinic the following day with fever and watery, red eyes. What will the provider do initially? Observe for improvement or worsening for 24 hours Begin treatment with an antiviral medication Administer LAIV influenza vaccine Perform a nasal swab for RT-PCR assay Which patient might be expected to have the worst FEV1? Patient with bronchiolitis A controlled asthma patient A 65 her old with emphysema A 60-year-old with pneumonia Forced expiratory volume in 1 second (FEV1)is the worst in patients with obstructive disease such as emphysema. An FEV1 should not be performed in patients with pneumonia and bronchiolitis because they would have diminished respiratory capacity related to the infection An older patient with COPD is experiencing dyspnea and has an oxygen saturation of 89% on room air. The patient has no history of pulmonary hypertension or congestive heart failure. What will the provider order to help manage this patient’s dyspnea? Breathing exercises Anxiolytic drugs Supplemental oxygen Opioid medications Which are causes of pleural effusions? (Select all that apply.) Allergies Bronchiectasis Breast cancer Congestive heart failure Dehydration Pleurisy is not a diagnosis but rather a symptom of many localized and systemic disease that needs further evaluation in order to find the cause of the problem. True False A patient with cough and fever is found to have infiltrates on chest x-ray. Would this is likely diagnosis Tuberculosis Pneumonitis Pneumonia Acute bronchitis Infiltrates on x-ray in conjunction with clinical findings of fever and cough should direct the NP to consider pneumonia as diagnosis Dyspnea, tachypnea and pleuritic CP are classic presentation of a pulmonary emboli. If your pt is complaining of calf or thigh leg pain, you should suspect compartment syndrome pulmonary embolism (PE) peripheral neuropathy with fracture deep vein thrombosis (DVT) What does peak flow meter measure? Peak flow capacity Oxygen saturation Exercise capacity Expiratory flow A patient comes to an emergency department with chest pain. The patient describes the pain is sharp and stabbing and reports that it has been present for several weeks. Upon questioning, the examiner determines that the pain is worse after eating. The patient reports getting relief after taking a friend’s nitroglycerin during one episode. What is the most likely cause of this chest pain?orrect! Esophageal pain Pleural pain Cardiac pain Aortic dissection pain A previously healthy patient develops influenza which is confirmed by RT-PCR testing and begins taking an antiviral medication. The next day, the patient reports increased fever and cough without respiratory distress. The patient’s lungs are clear and oxygen saturations are 97% on room air. What will the provider recommend? Empiric antibiotics to treat a possible secondary infection Referral to a specialist for evaluation and treatment Correct! Symptomatic treatment with close follow up in clinic Admission to the hospital for treatment of complications Hemoptysis can be confuse with other bleeding disorder. To narrow down your differential diagnosis, it is important to evaluate and obtain these pertinent data regarding your patient's history of: (Choose all that apply) sequence of birth number of soda intake per day Correct! occupational and environmental exposures travel history A patient is seen in clinic for an asthma exacerbation. The provider administers three nebulizer treatments with little improvement, noting a pulse oximetry reading of 90% with 2 L of oxygen. A peak flow assessment is 70%. What is the next step in treating this patient? Admit to the hospital with specialist consultation Prescribe an oral corticosteroid medication Give epinephrine injections and monitor response Administer three more nebulizer treatments and reassess What is the most common complication of influenza? Bacterial pneumonia Cough Bronchitis You Answered Viral pneumonia A patient develops acute bronchitis and is diagnosed as having influenza. Which medication will help reduce the duration of symptoms in this patient? Oseltamivir Whoever put this answer its incorrect the correct answer is Azithromycin The most common cause of bloody expectorant in primary care are due to the following: GERD and esophagitis Acute bronchitis and Pneumonia URI and Asthma AAA and varices Buttaro, Chapter 108 Hemoptysis, pg 471 An adult develops chronic cough with episodes of wheezing and shortness of breath. The provider performs chest radiography and other tests and rules out infection, upper respiratory, and gastroesophageal causes. Which test will the provider order initially to evaluate the possibility of asthma as the cause of these symptoms? Spirometry Allergy testing Peak expiratory flow rate Methacholine challenge test The major laboratory abnormality noted in patients who have pneumococcal pneumonia is: Gram stain positive Leukopenia Eosinophilia Leukocytosis Increased white count is typical in patients with bacterial pneumonia. Eosinophils can be increased in patients but developed pneumonia secondary to irritating substances such as toxic gas. Leukopenia is an ominous finding, especially in older patients, indicating poor prognosis The parent of a 4-month-old infant who has had an episode of bronchiolitis asks the provider if the infant may have an influenza vaccine. What will the provider tell this parent? The infant should have an influenza vaccine now with a booster in 1 month. The rest of the family and all close contacts should have the influenza vaccine. The infant should be given prophylactic antiviral medications. The infant should have the live attenuated influenza vaccine (LAIV). When initially treating adults with acute bronchitis, which of the following should the nurse practitioner be least likely to order: Antitussives Expectorants Antibiotics Bronchodilators Which of the following infections can cause a "barky" cough? Croup Which method of treatment is used for traumatic pneumothorax? 1. Placement of small-bore catheter 2. Needle aspiration of the pneumothorax 3. Observation for spontaneous resolution 4. Tube thoracostomy A patient who has undergone surgical immobilization for a femur fracture reports dyspnea and chest pain associated with inspiration. The patient has a heart rate of 120 beats per minute. Which diagnostic test will confirm the presence of a pulmonary embolism? 1. CT angiography 2. Electrocardiogram 3. D-dimer 4. Arterial blood gases A patient has a cough and fever and the provider auscultates rales in both lungs that do not clear with cough. The patient reports having a headache and sore throat prior to the onset of coughing. A chest radiograph shows patchy, nonhomogeneous infiltrates. Based on these findings, which organism is the most likely cause of this patient’s pneumonia. 1. Tuberculosis 2. S. pneumoniae 3. Mycoplasma 4. TrueA virus A high school athlete reports recent onset of chest pain that is aggravated by deep breathing and lifting. A 12-lead electrocardiogram in the clinic is normal. The examiner notes localized pain near the sternum that increases with pressure. What will the provider do next? 1. Prescribe an antibiotic 2. Recommend an NSAID 3. Order a chest radiograph 4. Refer to a cardiologist A patient who has asthma calls the provider to report having a peak flow measure of 75%, shortness of breath, wheezing, and cough, and tells the provider that the symptoms have not improved significantly after a dose of albuterol. The patient uses an inhaled corticosteroid medication twice daily. What will the provider recommend? 1. Taking an oral corticosteroid 2. Administering two more doses of albuterol 3. Going to the emergency department 4. Coming to the clinic for evaluation The following requires immediate intervention and hospitalization (choose all that apply) 1. Primary spontaneous pneumothorax (PSP) 2. Secondary spontaneous pneumothorax 3. PSP with less than 2-3 cm between the lung and chest wall 4. Tension pneumothorax An adult patient who had pertussis immunizations as a child is exposed to pertussis and develops a runny nose, low-grade fever, and upper respiratory illness symptoms without a paroxysmal cough. What is recommended for this patient? 1. Isolation if paroxysmal cough develops 2. Symptomatic care only 3. Pertussis vaccine booster 4. Azithromycin daily for 5 days What disease is usually managed with short acting oral long-acting inhaled anticholinergic medications?! COPD Bronchitis Asthma Bronchiectasis Anticholinergics can improve lung function and COPD. Anticholinergics are not to be used as a lone agent to manage symptoms of asthma. Bronchitis is a viral infection and is self-limiting, does not usually require inhalers. The patient has cough, pharyngitis, nasal discharge, and fever. He has been diagnosed with acute bronchitis. Which symptom is least likely in the first 3 days of this illness? Cough A patient complains of shortness of breath when in a recumbent position and reports coughing and pain associated with inspiration. The provider notes distended neck veins during the exam. What is the likely cause of these findings? Hepatic disease Pulmonary infection Pulmonary embolus Congestive heart failure A patient reports shortness of breath with activity and exhibits increased work of breathing with prolonged expirations. Which diagnostic test will the provider order to confirm a diagnosis in this patient? Blood cultures Spirometry Arterial blood gases Ventilation/perfusion scan Mycoplasma pneumoniae is: Only identifiable on chest x-ray A disease with extrapulmonary manifestations Of diagnosis of exclusion And on common respiratory pathogens Mycoplasma is an atypical pathogen that produces atypical pneumonia. It is often difficult to diagnose because symptoms are varied often involving extrapulmonary symptoms such as GI symptoms, myalgia and arthralgia, rash. Chest x-ray may have some unique findings with mycoplasma but it is not the only way to diagnose it What is the most common cause of pneumonia and people of all ages?Correct! S. pneumoniae Group A Strep S. aureus Mycoplasma sp. Which clinical sign is especially worrisome in a patient with a pulmonary embolism? Abnormal lung sounds Hypotension Dyspnea Tachycardia You have an elderly patient with a history of a myocardial infarction with residual paresthesia, and dysphagia. Your patient was brought in by his caretaker in the nursing home. The patient presents with one week of fever, productive cough and malaise. Besides his presentation, you suspect pneumonia due to his risk factors. (Choose all that apply) lack of proper housing Diminished gag reflex that can prevent entry of particles, mucus and food debris into the lungs Inadequate nutrition possible compromise airway filtration and humidification from his h/o CVA You see a college student in college health clinic. She complains of abrupt onset of sore throat, nasal congestion, runny nose, and malaise. Vital signs show temperature of 99.8°F, otherwise normal physical exam reveals an erythematous throat, swollen nasal turbinates, and rhinitis. The NP suspect viral URI. Although the following treatments are appropriate except: Pseudoephedrine Saline nasal spray Oral prednisone Ibuprofen Week 3 – Endocrinology A 20-year-old female patient with tachycardia and weight loss but no optic symptoms has the following laboratory values: decreased TSH, increased T3, and increased T4 and free T4. A pregnancy test is negative. What is the initial treatment for this patient? Beta blocker medications A patient who is obese has recurrent urinary tract infections and reports feeling tired most of the time. What initial diagnostic test will the provider order in the clinic at this visit? C-peptide level Hemoglobin A1C Random serum glucose Thyroid studies In order to determine how much T4 replacement of patient needs to reestablish a euthyroid state, the nurse practitioner considers: Patient's body weight Replacement as based on body weight and is usually calculated in kilograms. Body weight is multiplied by 1.6 to determine replacement needed in one day. This is the amount that should be prescribed in otherwise healthy, less than 50 years old, no evidence of underlying cardiac disease patients Which findings are part of the 2009 diagnostic criteria for metabolic syndrome? (Select all that apply.) Triglycerides ≧150 mg/dL HDL cholesterol </= 40 mg/dL Elevated waist circumference Fasting plasma glucose ≧100 mg/dL Decreased plasminogen activator inhibitor 1 levels A patient recently diagnosed with type 1 diabetes mellitus is in clinic for a follow-up evaluation. The provider notes that the patient appears confused and irritable and is sweating and shaking. What intervention will the provider expect to perform once the point of care blood glucose level is known? Giving a rapid-acting carbohydrate A 38-year-old male patient presents for his annual exam. He reports nervousness and weight loss, but denies any changes in his dietary intake or exercise level. Based on these findings and the following lab values, more what is the most likely diagnosis? TSH 0.01 (normal 0.4-3.8) Free T4 6 (normal 0.8-2.8) Free T3 205 (normal 70-205) Hyperthyroidism TSH is low and T4 is high indicating hyperthyroidism Excessive thirst and volume of dilute urine may be a symptom of: Diabetes insipidus Diabetes insipidus is a condition in which the kidneys are unable to conserve water, often because there is insufficient antidiuretic hormone (ADH) or the kidneys are unable to respond to ADH. Although diabetes mellitus may present with similar symptoms, the disorders are different. Diabetes insipidus does not involve hyperglycemia. Which findings are symptoms of hyperparathyroidism? (Select all that apply.) Cognitive impairment Chvostek’s sign Renal calculi Lef t ventricular hypertrophy Perioral paresthesias Which thyroid stimulating hormone (TSH) level indicates hyperthyroidism? 2.4 uIU/L 0.4 uIU/L 0.2 uIU/L 4.2 uIU/L Some of the hallmark characteristics of hyperosmolar hyperglycemic state are: Blood glucose over 1000 mg/dL Slow onset (over days) Higher prevalence in type 1 diabetics Negative ketones The patient comes into the clinic complaining of weight gain and brings in a nonfasting glucose log indicating glucose ranging from 110-170 mg/dL. This patient: Has impaired fasting glucose Should have a hemoglobin A1c performed Has normal blood glucose values Should continue to monitor glucose at home A postpartum woman develops fatigue, weight gain, and constipation. Laboratory values reveal elevated TSH and decreased T3 and T4 levels. What will the provider tell this patient? A thyroidectomy will be necessary. She will need lifelong medication. This condition may be transient. She should be referred to an endocrinologist. A 76-year-old obese patient has fatigue, thirst, and frequent urination. She was asked to measure a.m. fasting glucose value for one week. The values range from 142-1 75 mg/dL. She is now back to your clinic due to persistent symptoms, this patient: Should have a hemoglobin A1C performed for diagnosis Can be diagnosed with type 2 diabetes Has prediabetes Has impaired fasting glucose Impaired fasting glucose can be diagnosed when to fasting glucose readings are between 100-125 mg/dL. The conjunction of classic symptoms of hyperglycemia with a fasting blood glucose over 126 mg/dL consistently is diagnostic for diabetes. A1c is not required to diagnose diabetes in this case, but can be done to establish a baseline. Untreated hyperglycemia may lead to all of the following complications except: Vitiligo Excessively high blood sugar or prolonged hyperglycemia can cause diabetic ketoacidosis, the condition in which the body breaks down fat for energy and ketones spill into the urine. Diabetic hyperosmolar syndrome occurs when blood sugar is excessively high and available insulin is ineffective. In this case, the body cannot use glucose or fat for energy and glucose is excreted in the urine. Without immediate medical attention, both conditions may result in coma or death. Which choice best describes the most common presentation of a patient with type 2 diabetes? Insidious onset of hyperglycemia with weight gain Most patient's with type 2 diabetes are asymptomatic at presentation. They are identified because of screening and identification of risk factors. Diabetes usually has an insidious onset and is associated with weight gain. An acute onset is typical of patients with type 1 diabetes. Microalbuminuria develops after several years of having diabetes Mr. Jones is a type I diabetic that presents to your clinic with nausea/vomiting and weakness. You do random blood sugar test in clinic and his blood sugar is 550. You are not able to obtain a urine sample at this time. What is your next action? Treat with insulin and order chemistry and urine labs Treat with insulin Send to ED Order a urine test to be done at the lab This patient is a type I diabetic with symptoms and blood glucose that should lead NP to consider DKA. DKA is treated in the inpatient setting as it requires insulin, IV rehydration, and careful monitoring of electrolytes. The patient has a TSH value of 13.1 today. The nurse practitioner had decided to initiate replacement therapy with levothyroxine 88 µg daily. When should the NP recheck the patient's TSH level? 10 weeks 2 weeks 4 weeks 6 weeks Symptoms of hypothyroidism can improve post levothyroxine therapy within 2-3 weeks; however steady TSH concentration are not achieved for at least 6 weeks. TSH can then be monitored annually unless the patient asymptomatic. In order to determine how much T4 replacement of patient needs to reestablish a euthyroid state, the nurse practitioner considers: Patient's body weight A patient who has diabetes has a blood pressure of 140/90 mm Hg and significant >2mg/dL albuminuria. Which initial action by the primary care provider is indicated for management of this patient? 1. Referring to an ophthalmologist 2. Consulting with a nephrologist 3. Prescribing an antihypertensive medication 4. Limiting protein intake Which thyroid stimulating hormone (TSH) level indicates hyperthyroidism? 1. 0.4 2. 4.2 3. 0.2 4. 2.4 A patient who is obese has recurrent urinary tract infections and reports feeling tired most of the time. What initial diagnostic test will the provider order in the clinic at this visit? 1. Thyroid studies 2. Hemoglobin A1c 3. Random serum glucose 4. C-peptide level Mr. Smith, an overweight 48-year-old male with undiagnosed type 2 diabetes mellitus presents to your clinic. Which symptom is least likely associated with type 2 diabetes mellitus? 1. Constipation 2. Impetigo 3. Fatigue 4. Athlete’s foot A 52-year-old presents with thirst and frequent urination today. His glucose is 352. How should this be managed today? 1. Start metformin 2. Have him return tomorrow to recheck his blood glucose 3. Start metformin plus pioglitazone 4. Start insulin A postpartum woman develops fatigue, weight gain, and constipation. Laboratory values reveal elevated TSH and decreased T3 and T4 levels. What will the provider tell this patient? 1. This condition may be transient 2. She should be referred to an endocrinologist 3. A thyroidectomy will be necessary 4. She will need lifelong medication A patient who is obese has recurrent urinary tract infections and reports feeling tired most of the time. What initial diagnostic test will the provider order in the clinic at this visit? 1. Random serum glucose 2. Hemoglobin a1c 3. Thyroid studies 4. c-peptide Excessive thirst and volume of dilute urine may be a symptom of: 1. Diabetes insipidus 2. Viral gastroenteritis 3. Urinary tract infection 4. hypoglycemia A 45-year-old female patient has fatigue for the past 3 months and a 10 pound weight gain. She previously had regular menses occurring about every 30 days, but in the last 3 months her menses have varied. She has high TSH. It was repeated one week later and found to be even higher. Would explain this finding? 1. Hypothyroidism 2. Subclinical hypothyroidism 3. Perimenopause 4. Transient hypothyroidism Mr. Jones, brings his obese 15 year old son in to see you. You examine the 15-year-old and identify acanthosis nigricans. This probably indicates: Insulin resistance Acanthosis nigricans is usually associated with insulin resistance. It is a predictor of development of type 2 diabetes, especially in children, but is not a diagnostic finding on its own. Acanthosis nigricans is associated with all of the following disorders except: Tinea versicolor A 65-year-old patient presents to your clinic with evidence of hyperthyroidism. In assessing her cardiovascular status, which should NP assess immediately? Cardiac enzymes Electrocardiogram A 45-year-old patient who has hypothyroidism and takes levothyroxine. Based on the following lab results, how should the nurse practitioner proceed? TSH 32.7 (normal 0.4-3.8) Free T4 0.09 (normal 0.8-2.8) LDL 190mg/dL Total cholesterol 260mg/gL Increase levothyroxine dose A patient has thyroid nodules and the provider suspects thyroid cancer. To evaluate thyroid nodules for potential malignancy, which test is performed? Thyroid ultrasound Which laboratory values representing parathyroid hormone (PTH) and serum calcium are consistent with a diagnosis of primary hyperparathyroidism? Inappropriate secretion of PTH along with hypercalcemia A 50-year-old female presents for her annual exam. She complains of fatigue and weight gain. She has a following lab results. What should the NP order? TSH 7 (normal 0.4-3.8) Repeat TSH plus free T4 Patient presents with elevated TSH as well as possible symptoms of hypothyroidism. A repeat of TSH and free T4 should be done to determine presence and degree of hypothyroidism. Replacement therapy is generally not initiated until the TSH is greater than 10 and direct measurement of serum T4 is obtained. there is no nodule or other indications in this scenario for an ultrasound. In order to determine how much T4 replacement of patient needs to reestablish a euthyroid state, the nurse practitioner considers: The TSH level A 20-year-old female patient with tachycardia and weight loss but no optic symptoms has the following laboratory values: decreased TSH, increased T3, and increased T4 and free T4. A pregnancy test is negative. What is the initial treatment for this patient? Beta blocker medications A patient who is obese has recurrent urinary tract infections and reports feeling tired most of the time. What initial diagnostic test will the provider order in the clinic at this visit? Hemoglobin A1C A patient has thyroid nodules and the provider suspects thyroid cancer. To evaluate thyroid nodules for potential malignancy, which test is performed? Thyroid ultrasound Which findings are symptoms of hyperparathyroidism? (Select all that apply.) Renal calculi Lef t ventricular hypertrophy Cognitive impairment A postpartum woman develops fatigue, weight gain, and constipation. Laboratory values reveal elevated TSH and decreased T3 and T4 levels. What will the provider tell this patient? This condition may be transient. The patient comes into the clinic complaining of weight gain and brings in a nonfasting glucose log indicating glucose ranging from 110-170 mg/dL. This patient: Should have a hemoglobin A1c performed Weight gain is one of the potential symptoms of diabetes, however it is a nonspecific symptom, the patient did not have any of the other classic diabetes symptoms. His nonfasting glucose log has a wide range. Confirmation of prediabetes versus normal finding versus diabetes should be done with repeat testing, preferably with hemoglobin A1c. A 45-year-old patient who has hypothyroidism and takes levothyroxine. Based on the following lab results, how should the nurse practitioner proceed? TSH 32.7 (normal 0.4-3.8) Free T4 0.09 (normal 0.8-2.8) LDL 190mg/dL Total cholesterol 260mg/dL Increase levothyroxine dose This patient has a low T4 with elevated TSH and hence needs an increase in levothyroxine. Dyslipidemia is common finding when TSH exceeds 10. The NP should first treat hypothyroidism and then proceed with repeat of lipid testing. A 50-year-old female presents for her annual exam. She complains of fatigue and weight gain. She has a following lab results. What should the NP order? TSH 7 (normal 0.4-3.8) Repeat TSH plus free T4 Patient presents with elevated TSH as well as possible symptoms of hypothyroidism. A repeat of TSH and free T4 should be done to determine presence and degree of hypothyroidism. Replacement therapy is generally not initiated until the TSH is greater than 10 and direct measurement of serum T4 is obtained. there is no nodule or other indications in this scenario for an ultrasound. Acanthosis nigricans is associated with all of the following disorders except: Tinea versicolor Acanthosis nigricans is a benign skin condition that is a sign of insulin resistance. It is rarely associated with some types of adenocarcinoma of the GI tract. Tinea versicolor is a superficial infection of the skin that is caused by fungi A 45-year-old female patient has fatigue for the past 3 months and a 10 pound weight gain. She previously had regular menses occurring about every 30 days, but in the last 3 months her menses have varied. She has high TSH. It was repeated one week later and found to be even higher. Would explain this finding? Hypothyroidism The patient's TSH is high on 2 occasions, along with symptoms, this was certainly lead to a diagnosis of hypothyroidism. A postpartum woman develops fatigue, weight gain, and constipation. Laboratory values reveal elevated TSH and decreased T3 and T4 levels. What will the provider tell this patient? She will need lifelong medication. You Answered She should be referred to an endocrinologist. This condition may be transient. A thyroidectomy will be necessary. A patient who is obese has recurrent urinary tract infections and reports feeling tired most of the time. What initial diagnostic test will the provider order in the clinic at this visit? Thyroid studies Hemoglobin A1C C-peptide level Random serum glucose Which findings are part of the 2009 diagnostic criteria for metabolic syndrome? (Select all that apply.) Elevated waist circumference Decreased plasminogen activator inhibitor 1 levels HDL cholesterol </= 40 mg/dL Fasting plasma glucose ≧100 mg/dL Triglycerides ≧150 mg/dL Mr. Smith, an overweight 48-year-old male with undiagnosed type 2 diabetes mellitus presents to your clinic. Which symptom is least likely associated with type 2 diabetes mellitus? Athlete's foot Fatigue Constipation Impetigo Infections could be related to elevated blood glucose levels. Fatigue is a common early symptom of diabetes. Constipation could be due to many factors but not specifically diabetes. The patient comes into the clinic complaining of weight gain and brings in a nonfasting glucose log indicating glucose ranging from 110-170 mg/dL. This patient: Has normal blood glucose values Has impaired fasting glucose Should have a hemoglobin A1c performed Should continue to monitor glucose at home A patient has thyroid nodules and the provider suspects thyroid cancer. To evaluate thyroid nodules for potential malignancy, which test is performed? Radionucleotide imaging Serum calcitonin Thyroid ultrasound Serum TSH level A postpartum woman develops fatigue, weight gain, and constipation. Laboratory values reveal elevated TSH and decreased T3 and T4 levels. What will the provider tell this patient? She should be referred to an endocrinologist. This condition may be transient. A thyroidectomy will be necessary. She will need lifelong medication. Which thyroid stimulating hormone (TSH) level indicates hyperthyroidism? 0.2 uIU/L 2.4 uIU/L 0.4 uIU/L 4.2 uIU/L A 45-year-old patient who has hypothyroidism and takes levothyroxine. Based on the following lab results, how should the nurse practitioner proceed? TSH 32.7 (normal 0.4-3.8) Free T4 0.09 (normal 0.8-2.8) LDL 190mg/dL Total cholesterol 260mg/dL Encourage lifestyle modifications Increase levothyroxine dose Decrease levothyroxine dose Begin statin therapy This patient has a low T4 with elevated TSH and hence needs an increase in levothyroxine. Dyslipidemia is common finding when TSH exceeds 10. The NP should first treat hypothyroidism and then proceed with repeat of lipid testing. The patient has a TSH value of 13.1 today. The nurse practitioner had decided to initiate replacement therapy with levothyroxine 88 µg daily. When should the NP recheck the patient's TSH level? 4 weeks 6 weeks 10 weeks 2 weeks Symptoms of hypothyroidism can improve post levothyroxine therapy within 2-3 weeks; however steady TSH concentration are not achieved for at least 6 weeks. TSH can then be monitored annually unless the patient asymptomatic. Which laboratory values representing parathyroid hormone (PTH) and serum calcium are consistent with a diagnosis of primary hyperparathyroidism? Inappropriate secretion of PTH along with hypercalcemia Week 4 – Cardiovascular (CV) An adult patient reports frequent episodes of syncope and lightheadedness. The provider notes a heart rate of 70 beats per minutes. What will the provider do next? Evaluate the patient’s orthostatic vital signs Which are causes of secondary hypertension? (Select all that apply.) Nonsteroidal anti-inflammatory drugs Sleep apnea Oral contraceptives You're treating your patient for heart failure exacerbation. Which medications could potentially exacerbate heart failure? Naproxen Aspirin Atorvastatin Furosemide While doing the cardiac exam on a 45-year-old, you notice any irregular rhythm with a pulse rate of 110 bpm. The patient is alert and not in distress. What is the likely diagnosis? Atrial fibrillation Atrial fibrillation is an irregular rhythm. In ventricular fibrillation patient would be unstable. Right bundle branch block does not cause rhythm irregularities. Second-degree AV block can cause rhythm irregularity, but is usually accompanied with bradycardia. A child with a history of asthma is brought to the clinic with a rapid heart rate. A cardiac monitor shows a heart rate of 225 beats per minute. The provider notifies transport to take the child to the emergency department. What initial intervention may be attempted in the clinic? Using a vagal maneuver or carotid massage Which are factors can cause a heart murmur? (Select all that apply.) High rates of flow through a normal valve Backward flow through a septal defect Forward flow into a dilated vessel A 28-year-old has a grade 3 murmur. Which characteristic indicates a need for referral? A fixed split A split this created because of closure of valves. For example, and S2 is created by closure of an aortic and pulmonic valve. Normal these split with inspiration and almost never with expiration. Splits should never be fixed. This could indicate pathology such as atrial septal defect, pulmonic stenosis, or possibly mitral regurgitation. Mr. Smith is a 72-year-old patient takes warfarin for chronic atrial fibrillation. His INR today is 4. The nurse practitioner should: Stop the warfarin today and repeat INR tomorrow. INR range for atrial fibrillation is usually 2-3 for chronic atrial fibrillation. Stopping warfarin for 4 days with certainly decrease INR, but his overkill, and would put patient at risk for thromboembolism when INR is subtherapeutic. Vitamin K is not necessary for patients who are not bleeding due to high INR, or whose INR is less than 8. Which are causes of secondary hypertension? (Select all that apply.) Oral contraceptives Isometric exercises Increased salt intake ct! Sleep apnea ! Nonsteroidal anti-inflammatory drugs The AHA recommends early CPR and AED use for adult victims of cardiac arrest outside of a hospital setting because most victims have which arrhythmia? Atrial fibrillation ct! Ventricular fibrillation Atrial flutter Ventricular tachycardia A patient with poorly controlled hypertension and history of myocardial infarction 6 years ago presents today with mild shortness of breath. He takes quinapril, aspirin, metoprolol, and statin daily. What symptom is not indicative of heart failure?Correct! Headache Cough Orthopnea Fatigue Headache is a nonspecific symptom and is not typical for heart failure. All the other symptoms are classic for worsening heart failure. A 28-year-old has a grade 3 murmur. Which characteristic indicates a need for referral? A fixed split An increase in splitting with inspiration Split S2 with inspiration wered Changes in intensity with position changes A split this created because of closure of valves. For example, and S2 is created by closure of an aortic and pulmonic valve. Normal these split with inspiration and almost never with expiration. Splits should never be fixed. This could indicate pathology such as atrial septal defect, pulmonic stenosis, or possibly mitral regurgitation. A health care provider in a clinic finds a patient in a room, unresponsive and pale. Which possible signs should be used to identify the need to initiate cardiopulmonary resuscitation (CPR)? Correct! Assessment of gasping breaths or not breathing Determination of pulselessness or bradycardia An African-American patient who is being treated with a thiazide diuretic for chronic hypertension reports blurred vision and shortness of breath. The provider notes a blood pressure of 185/115. What is the recommended action for this patient? Admit to the hospital for evaluation and treatment A patient reports sustained, irregular heart palpitations. What is the most likely cause of these symptoms? Atrial fibrillation A patient taking atorvastatin for newly diagnosed dyslipidemia complains of fatigue, weakness, muscle aches in his lower back, arms, legs for the past 3 days. It has not improved with rest. How should this be managed initially? Stop atorvastatin immediately This patient has symptoms consistent with rhabdomyolysis. Atorvastatin should be stopped and CPK should be ordered. Liver enzymes would not assess for etiology of myalgia. Nighttime muscle cramps are not usually associated with statin use. A 40-year-old African-American patient has blood pressure readings of 175/110 and repeat reading of 170/102. He has no significant cardiovascular symptoms. What is a reasonable plan of care for this patient today? Initiate amlodipine 5 mg daily Since the patient is African-American, thiazide diuretics or calcium channel blockers should be considered. Hydrochlorothiazide at 12.5 mg daily is unlikely to give him good blood pressure reduction due to low dose, amlodipine is the better choice. A patient reports recurrent chest pain that occurs regardless of activity and is not relieved by rest. The provider administers a nitroglycerin tablet which does not relieve the discomfort. What is the next action? Give the patient a beta blocker medication A patient has shortness of breath. if heart failure is the etiology, which test demonstrates the highest sensitivity in diagnosing this? B-type natriuretic peptide (BNP) When BNP is elevated, the sensitivity and specificity is 98% and 92%, favoring a diagnosis of heart failure. Echocardiograms I used to evaluate internal structures of the heart and established an ejection fraction and is useful and diagnosis of heart failure, but does not give a correlation to patient's current symptoms. chest x-ray can indicate pulmonary congestion such as with congestive heart failure, but BNP is more sensitive measure Which are factors can cause a heart murmur? (Select all that apply.) Forward flow into a dilated vessel rrect! High rates of flow through a normal valve Backward flow into a normal vessel rrect! Backward flow through a septal defect Low rates of flow into a cardiac chamber An elderly female without prior history of cardiovascular disease reports lower leg soreness and fatigue when shopping or walking in the neighborhood. The primary care provider notes decreased pedal pulses bilaterally. Which test will the provider order initially to evaluate for peripheral arterial disease based on these symptoms? Digital subtraction angiography ect! Doppler ankle, arm index Magnetic resonance angiography Segmental limb pressure measurement A child with a history of asthma is brought to the clinic with a rapid heart rate. A cardiac monitor shows a heart rate of 225 beats per minute. The provider notifies transport to take the child to the emergency department. What initial intervention may be attempted in the clinic?ect! Using a vagal maneuver or carotid massage Giving a beta blocker Providing a loading dose of digoxin Administration of intravenous adenosine A 43-year-old Hispanic male presents to clinic for uncomplicated lower leg cellulitis. During her exam, you notice an audible diastolic murmur best heard in the mitral listening point. he does not have any significant cardiovascular symptoms. The murmur is probably: Acute mitral regurgitation Chronic mitral regurgitation Mitral stenosis Mitral valve prolapse A 35-year-old man has a history of an upper respiratory viral infection 4 weeks ago. He reports that he started feeling dyspnea and now complains of sharp pain in the middle of the chest that is worse when he lies down. Physical exam is within normal limits with the exception of pericardial rub on auscultation. The most likely diagnosis would be: Pulmonary embolism Esophageal reflux Pericarditis Pneumonia A patient reports abdominal and back pain with anorexia and nausea. During an exam, the provider notes a pulsatile abdominal mass. What is the initial action? Ultrasound of the mass to determine size Immediate referral to a thoracic surgeon Scheduling an MRI to evaluate for aortic disease Ordering computerized tomography angiography A young female patient has known mitral valve prolapse. During a routine health maintenance exam, the provider notes an apical systolic murmur and a midsystolic click on auscultation. The patient denies chest pain, syncope, or palpitations. What will the provider do? Continue to monitor the patient every 3 years Reassure the patient that these findings are expected Consult with the cardiologist to determine appropriate diagnostic tests Admit the patient to the hospital for evaluation and treatment A 90-year-old is presenting to clinic with isolated systolic hypertension. Which of the following antihypertensive medications are preferred for the treatment of this condition" Calcium channel blockers and thiazide diuretics Beta blockers and potassium sparing diuretics Angiotensin-converting enzyme inhibitors and loop diuretics Alpha blockers and calcium channel blockers And calcium channel blockers and thiazide diuretics are preferred. There is generally no preference 1 over another unless there is other significant comorbidities. Beta blockers are no longer recommended as first line for hypertension treatment. Alpha blockers is a possible treatment but is not first-line. Loop diuretics are not recommended specifically for hypertension treatment. A 55-year-old patient has a blood pressure of 138/85 on three occasions. The patient denies headaches, palpitations, snoring, muscle weakness, and nocturia and does not take any medications. What will the provider do next to evaluate this patient? Order urinalysis, CBC, BUN, and creatinine Assess serum cortisol levels Refer to a specialist for a sleep study Continue to monitor blood pressure at each health maintenance visit A patient is brought to an emergency department with symptoms of acute ST-segment elevation MI (STEMI). The nearest hospital that can perform percutaneous coronary intervention (PCI) is 3 hours away. What is the initial treatment for this patient? Transfer to the PCI-capable institution Administer heparin Give the patient an oral beta blocker Initiate fibrinolytic treatment Current American Heart Association (AHA) recommendations include: (Select all that apply.) A rate of 100 compressions per minute at a minimum A compression depth of 1½ inches or more on an adult Using a ratio of 2 rescue breaths to 30 compressions Untrained rescuers giving compressions without breaths Rescue breaths given during 2 seconds to allow full chest rise A 40-year-old woman comes to the medical office complaining of palpitations and some lightheadedness for the past 6 months. These are and him episodes. You notice a midsystolic click with late systolic murmur that is best heard in the apical area during auscultation of the chest. You would suspect: Atrial fibrillation Sinus arrhythmia Mitral stenosis Mitral valve prolapse MVP occurs when the mitral valve does not close all the way, causing a late systolic murmur heard best in the apical area during auscultation. Following a normal S1, the valve suddenly prolapses, resulting in a midsystolic click. Classic symptoms of deep vein thrombosis include: Swelling, pain, discoloration in lower extremity A 75-year-old patient has aortic stenosis is presenting with symptoms of worsening stenosis. You know that to most common symptoms of worsening stenosis include: Shortness of breath and syncope Patient with diabetes presents with pain in his lower legs when he walks and pain resolution with rest. When specifically asked about pain in his lower leg, he likely will report pain: In the calf muscle In or around the ankle joint Pain in the lower leg that waxes and wanes Radiating down his leg from the thigh The patient's symptoms are typical of claudication related to arteriosclerosis. This usually produces pain in specific muscle group where ischemia occurs. Commonly this is seen in the calf muscle. A patient has a cardiac murmur that peaks in midsystole and is best heard along the left sternal border. The provider determines that the murmur decreases in intensity when the patient changes from standing to squatting and increases in intensity with the Valsalva maneuver. Which cause will the provider suspect for this murmur? Mitral valve prolapse Aortic stenosis Tricuspid regurgitation Hypertrophic cardiomyopathy A patient is diagnosed with PAD and elects not to have angioplasty after an angiogram reveals partial obstruction in lower extremity arteries. What will the provider recommend to help with relief of symptoms in this patient? Walking to the point of pain each day Statin therapy with clopidogrel Daily aspirin therapy to prevent clotting Walking slowly for 15 to 20 minutes twice daily An elderly woman has been taking digoxin for 10 years. Her EKG is showing a new onset of atrial fibrillation. Her pulse is 64 bpm. She denies any significant cardiovascular symptoms. Which of the following interventions is most appropriate? Discontinued digoxin and order another 12-lead EKG Order a serum TSH, digoxin level, electrolyte panel Order an electrolyte panel and digoxin level Order a digoxin level and decrease her digoxin dose by half while waiting for results An adult patient reports frequent episodes of syncope and lightheadedness. The provider notes a heart rate of 70 beats per minutes. What will the provider do next? 1. Monitor patient’s heart rate while the patient is bearing down 2. Evaluate the patient’s ortho vital signs 3. Order an electrocardiogram and exercise stress test A patient reports sustained, irregular heart palpitations. What is the most likely cause of these symptoms? 1. Anemia 2. Extrasystole 3. Atrial Fibrillation 4. Paroxysmal attacks A patient reports abdominal and back pain with anorexia and nausea. During an exam, the provider notes a pulsatile abdominal mass. What is the initial action? 1. Ordering computerized tomography angiography 2. Ultrasound of the mass to determine size 3. Scheduling an MRI to evaluate for aortic dz 4. Immediate referral to a thoracic surgeon A 43-year-old Hispanic male presents to clinic for uncomplicated lower leg cellulitis. During her exam, you notice an audible diastolic murmur best heard in the mitral listening point. he does not have any significant cardiovascular symptoms. The murmur is probably: 1. Acute mitral regurgitation 2. Chronic mitral regurgitation 3. Mitral valve prolapse 4. Mitral stenosis A 90-year-old is presenting to clinic with isolated systolic hypertension. Which of the following antihypertensive medications are preferred for the treatment of this condition" 1. Angiotensin-converting enzyme inhibitors and loop diuretics 2. Calcium channel blockers and thiazide diuretics 3. Beta blockers and potassium sparing diuretics 4. Alpha blockers and calcium channel blockers An African-American patient who is being treated with a thiazide diuretic for chronic hypertension reports blurred vision and shortness of breath. The provider notes a blood pressure of 185/115. What is the recommended action for this patient? 1. Prescribe a calcium channel blocker 2. Admit to the hospital for evaluation and treatment 3. Add a beta blocker to the patient’s regimen 4. Increase the dose of the thiazide medication A patient is diagnosed with PAD and elects not to have angioplasty after an angiogram reveals partial obstruction in lower extremity arteries. What will the provider recommend to help with relief of symptoms in this patient? 1. Daily aspirin therapy to prevent clotting 2. Statin therapy with clopidogrel 3. Walking to the point of pain each day 4. Walking slowly for 15 to 20 minutes twice daily A patient reports recurrent chest pain that occurs regardless of activity and is not relieved by rest. The provider administers a nitroglycerin tablet which does not relieve the discomfort. What is the next action? 1. Administer a second nitroglycerin tablet 2. Prescribe a calcium channel blocker medication 3. Give the patient a beta blocker medication 4. Start statin therapy and refer the patient to a cardiologist An elderly woman has been taking digoxin for 10 years. Her EKG is showing a new onset of atrial fibrillation. Her pulse is 64 bpm. She denies any significant cardiovascular symptoms. Which of the following interventions is most appropriate? 1. Order an electrolyte panel and digoxin level 2. Order a serum TSH, digoxin level, electrolyte panel 3. Order a digoxin level and decrease her digoxin dose by half while waiting for results 4. Discontinued digoxin and order another 12-lead EKG A patient taking atorvastatin for newly diagnosed dyslipidemia complains of fatigue, weakness, muscle aches in his lower back, arms, legs for the past 3 days. It has not improved with rest. How should this be managed initially? 1. Ask about nighttime muscle cramps 2. Check liver enzymes first 3. Stop atorvastatin immediately 4. Order a CPK level A 70 year old male patient complains of a bright red colored spot that has been present in his left eye for 2 days. He denies eye pain, visual changes, or headaches. He ha a new onset cough from a recent URI. The only medicine he is taking is aspirin 1 tablet daily. Which of the following is most likely? 1. Acute uveitis 2. Corneal abrasion 3. Acute bacterial conjunctivitis 4. Subconjunctival hemorrhage The patient presents with complaints of morning eyelash crusting and itchy red eyes. It began on the left and now has become bilateral. based on the most likely diagnosis, what should the NP tell the caregivers about this condition? Pain is normal in the affected eye This usually begins as a viral infection Anterior cervical lymphadenopathy is common It produces blurred vision in the affected eye A patient reports ear pain after being hit in the head with a baseball. The provider notes a large perforated tympanic membrane. What is the recommended treatment? Correct! Order antibiotic ear drops if signs of infection occur Reassure the patient that this will heal without problems Prescribe analgesics and follow up in 1 to 2 days Refer the patient to an otolaryngologist for evaluation A child is hit with a baseball bat during a game and sustains an injury to the nose, along with a transient loss of consciousness. A health care provider at the game notes bleeding from the child’s nose and displacement of the septum. What is the most important intervention at this time? Turn the child’s head to the side to prevent aspiration of blood Correct! Immobilize the child’s head and neck and call 911 Place nasal packing in both nares to stop the bleeding Apply ice to the injured site to prevent airway occlusion A patient has recurrent sneezing, alterations in taste and smell, watery, itchy eyes, and thin, clear nasal secretions. The provider notes puffiness around the eyes. The patient’s vital signs are normal. What is the most likely diagnosis for this patient? Correct! Allergic rhinitis Acute sinusitis Chronic sinusitis Viral rhinitis A patient diagnosed with strep throat received a prescription for azithromycin. She has not improved in 48 hours. What course of action is acceptable? Different macrolide antibiotic should be prescribed Correct! Penicillin or cephalosporin with beta-lactamase coverage should be considered The patient should wait another 24 hours for improvement That antibiotics should be changed to a first generation cephalosporin Improvement with antibiotics and strep throat should generally occur within 48 hours. Initial macrolide therapy would be a poor choice as there is high rates of strep resistance to macrolide antibiotics. penicillin antibiotics are generally first-line A 93 year old demented adult has been recently treated for an upper respiratory infection but drainage from the right nostril persists. What should the NP suspect? Unresolved URI Dental caries Allergic rhinitis Correct! Presence of foreign body There are 2 clinical clues in this question. First the pt has continued drainage despite treatment. Second, the drainage is unilateral. Unilateral drainage from a nostril should prompt the examiner to visualize turbinates. During routine physical exam of an elderly woman, a triangular thickening of the bulbar conjunctiva on the temporal side is noted to be encroaching on the cornea. She denies any eye pain or visual changes. Which of the following is most likely? Corneal arcus Chalazion Correct! Pterygium Pinguecula Which method of treatment is used for traumatic pneumothorax? Tube thoracostomy Observation for spontaneous resolution Correct! Needle aspiration of the pneumothorax Placement of a small-bore catheter A 22 year old tall, thin and athletic man comes into your primary care clinic complaining of pain with breathing and progressively worsening shortness of breath. In order of sequence, what will be your next steps? Obtain more history, order a stat CXR and have patient return to clinic Immediately auscultate the lungs and send for CXR Correct Answer Obtain more history, and immediately send him to the ER Correct! Obtain more history, auscultate the lungs and send pt to the ER for a stat CXR and further evaluation. Dyspnea, tachypnea and pleuritic CP are classic presentation of a pulmonary emboli. If your pt is complaining of calf or thigh leg pain, you should suspect compartment syndrome peripheral neuropathy with fracture Correct! deep vein thrombosis (DVT) pulmonary embolism (PE) Buttaro, Chapter 113, p. 494 A patient develops acute bronchitis and is diagnosed as having influenza. Which medication will help reduce the duration of symptoms in this patient? Correct Answer Azithromycin (I don’t think this is correct. I could be wrong) Trimethoprim-sulfamethoxazole You Answered Oseltamivir Clindamycin The major laboratory abnormality noted in patients who have pneumococcal pneumonia is: Leukopenia Correct! Leukocytosis Eosinophilia Gram stain positive Increased white count is typical in patients with bacterial pneumonia. Eosinophils can be increased in patients but developed pneumonia secondary to irritating substances such as toxic gas. Leukopenia is an ominous finding, especially in older patients, indicating poor prognosis Which patient might be expected to have the worst FEV1? Patient with bronchiolitis A 60-year-old with pneumonia Correct! A 65 her old with emphysema A controlled asthma patient Forced expiratory volume in 1 second (FEV1)is the worst in patients with obstructive disease such as emphysema. An FEV1 should not be performed in patients with pneumonia and bronchiolitis because they would have diminished respiratory capacity related to the infection An African-American patient who is being treated with a thiazide diuretic for chronic hypertension reports blurred vision and shortness of breath. The provider notes a blood pressure of 185/115. What is the recommended action for this patient? Add a beta blocker to the patient’s regimen Prescribe a calcium channel blocker Increase the dose of the thiazide medication Correct! Admit to the hospital for evaluation and treatment A 55-year-old patient has a blood pressure of 138/85 on three occasions. The patient denies headaches, palpitations, snoring, muscle weakness, and nocturia and does not take any medications. What will the provider do next to evaluate this patient? Refer to a specialist for a sleep study Assess serum cortisol levels Correct! Order urinalysis, CBC, BUN, and creatinine mitral stenosis Continue to monitor blood pressure at each health maintenance visit A patient is brought to an emergency department with symptoms of acute ST-segment elevation MI (STEMI). The nearest hospital that can perform percutaneous coronary intervention (PCI) is 3 hours away. What is the initial treatment for this patient? Transfer to the PCI-capable institution Give the patient an oral beta blocker Administer heparin Correct! Initiate fibrinolytic treatment A 43-year-old Hispanic male presents to clinic for uncomplicated lower leg cellulitis. During her exam, you notice an audible diastolic murmur best heard in the mitral listening point. he does not have any significant cardiovascular symptoms. The murmur is probably: Correct! Mitral stenosis Chronic mitral regurgitation Acute mitral regurgitation Mitral valve prolapse Mitral valve prolapse is unlikely as it is a systolic murmur. Acute mitral regurgitation usually develops after rupture of papillary muscles in the heart, patient would be presenting with significant symptoms. Mitral regurgitation is also a systolic murmur. Mr. Smith is a 72-year-old patient takes warfarin for chronic atrial fibrillation. His INR today is 4. The nurse practitioner should: Administer vitamin K and repeat INR in 2 hours Stop the warfarin for the next 4 days and repeat INR on day 5 Correct! Stop the warfarin today and repeat INR tomorrow. Admit to the hospital INR range for atrial fibrillation is usually 2-3 for chronic atrial fibrillation. Stopping warfarin for 4 days with certainly decrease INR, but his overkill, and would put patient at risk for thromboembolism when INR is subtherapeutic. Vitamin K is not necessary for patients who are not bleeding due to high INR, or whose INR is less than 8. A 28-year-old has a grade 3 murmur. Which characteristic indicates a need for referral? An increase in splitting with inspiration Correct! A fixed split Changes in intensity with position changes Split S2 with inspiration A split this created because of closure of valves. For example, and S2 is created by closure of an aortic and pulmonic valve. Normal these split with inspiration and almost never with expiration. Splits should never be fixed. This could indicate pathology such as atrial septal defect, pulmonic stenosis, or possibly mitral regurgitation. A 20-year-old female patient with tachycardia and weight loss but no optic symptoms has the following laboratory values: decreased TSH, increased T3, and increased T4 and free T4. A pregnancy test is negative. What is the initial treatment for this patient? Correct! Beta blocker medications Radioiodine therapy Surgical resection of the thyroid gland Thionamide therapy A postpartum woman develops fatigue, weight gain, and constipation. Laboratory values reveal elevated TSH and decreased T3 and T4 levels. What will the provider tell this patient? She should be referred to an endocrinologist. She will need lifelong medication. Correct! This condition may be transient. A thyroidectomy will be necessary. A patient who is obese has recurrent urinary tract infections and reports feeling tired most of the time. What initial diagnostic test will the provider order in the clinic at this visit? C-peptide level Correct! Hemoglobin A1C Random serum glucose Thyroid studies A 76-year-old obese patient has fatigue, thirst, and frequent urination. She was asked to measure a.m. fasting glucose value for one week. The values range from 142-1 75 mg/dL. She is now back to your clinic due to persistent symptoms, this patient: Should have a hemoglobin A1C performed for diagnosis Has impaired fasting glucose Correct! Can be diagnosed with type 2 diabetes Has prediabetes Impaired fasting glucose can be diagnosed when to fasting glucose readings are between 100-125 mg/dL. The conjunction of classic symptoms of hyperglycemia with a fasting blood glucose over 126 mg/dL consistently is diagnostic for diabetes. A1c is not required to diagnose diabetes in this case, but can be done to establish a baseline. Which choice best describes the most common presentation of a patient with type 2 diabetes? Correct! Insidious onset of hyperglycemia with weight gain Microalbuminuria Hyperlipidemia and the presence of retinopathy Acute onset of hyperglycemia with other symptoms Most patient's with type 2 diabetes are asymptomatic at presentation. They are identified because of screening and identification of risk factors. Diabetes usually has an insidious onset and is associated with weight gain. An acute onset is typical of patients with type 1 diabetes. Microalbuminuria develops after several years of having diabetes A 50-year-old female presents for her annual exam. She complains of fatigue and weight gain. She has a following lab results. What should the NP order? TSH 7 (normal 0.4-3.8) Thyroid ultrasound Levothyroxine 1.6 mcg/kg daily Correct! Repeat TSH plus free T4 Lipid panel Patient presents with elevated TSH as well as possible symptoms of hypothyroidism. A repeat of TSH and free T4 should be done to determine presence and degree of hypothyroidism. Replacement therapy is generally not initiated until the TSH is greater than 10 and direct measurement of serum T4 is obtained. there is no nodule or other indications in this scenario for an ultrasound. Acanthosis nigricans is associated with all of the following disorders except: Colon cancer Correct! Tinea versicolor Diabetes Obesity Acanthosis nigricans is a benign skin condition that is a sign of insulin resistance. It is rarely associated with some types of adenocarcinoma of the GI tract. Tinea versicolor is a superficial infection of the skin that is caused by fungi Week 5 – Gastrointestinal (GI) What is the best treatment for H. pylori-related peptic ulcer disease? PPI and clarithromycin for 14 days A patient has a history of diverticular disease and asks what can be done to minimize acute symptoms. What will the practitioner recommend to this patient? Consuming a diet high in fiber Which symptoms in a patient with abdominal pain are suggestive of appendicitis? (Select all that apply.) Pain accompanied by low-grade fever Abdominal rigidity along with pain Pain occurring prior to nausea and vomiting An 83 year old is diagnosed with diverticulitis. The most common complaint is Bloating and crampiness Left lower quadrant pain Frequent belching and flatulence Rectal bleeding About 70% of pts diagnosed with diverticulitis will have let lower quadrant pain. An older patient presents with left lower quadrant pain. If diverticulitis is suspected, how should the NP proceed? CT scan of abdomen Which is the most common cause of pancreatitis in the United States? Gallstones A patient experiences a sharp pain with swallowing just under the sternum. This is more commonly associated with which condition? Infectious esophagitis A patient's CBC demonstrated anemia. Which diagnosis is likely based on this patient's laboratory values? MCV 74.1 fL (normal 80-95) MCH 24 pg (normal 27-31) MCHC 33% (normal 32-69) RDW 12% (normal 11-14.5) thalassemia Vitamin B12 deficiency anemia Ron deficiency anemia Anemia of chronic disease Thalassemia and iron deficiency anemia are both microcytic anemias. The RDW indicates variation in width of red blood cells, RDW is normal in thalassemia and is typically elevated and iron deficiency anemia. Vitamin B12 deficiency is a macrocytic anemia. Anemia of chronic disease is usually either a normocytic or a microcytic anemia. All of the following relieve symptoms of GERD except: Chewing breath mints A patient has both occasional “coffee ground” emesis and melena stools. What is the most probable source of bleeding in this patient? Upper GI Hepatic Lower GI Rectal A patient's CBC demonstrated anemia. Which diagnosis is likely based on this patient's laboratory values? MCV 114 fL (normal 80-95) MCH 29 pg (normal 27-31) MCHC 33.8% (normal 32-69) RDW 15% (normal 11-14.5) Pernicious anemia This patient has elevated MCV, indicating macrocytic anemia. iron deficiency is a microcytic anemia. Anemia of chronic disease is either a micro or normocytic anemia. Sideroblastic anemia is a microcytic anemia. Which medications may increase the prevalence of GERD? (Select all that apply.) Oral contraceptives Benzodiazepines Hormone replacements Calcium antagonists Aspirin A patient has a recent episode of vomiting and describes the vomitus as containing mostly gastric juice. What does this symptom suggest? Peptic ulcer A patient is in clinic for evaluation of sudden onset of abdominal pain. The provider palpates a pulsatile, painful mass between the xiphoid process and the umbilicus. What is the initial action? Transfer the patient to the emergency department for a surgical consult A 74-year-old man presents with recurrent abdominal cramping and pain associated with diarrhea that occurs from 4-5 times per day. He reports that currently he is having an exacerbation. The stools are bloody with mucus and pus. The patient reports that he has lost weight and is always fatigued. The patient denies recent travel or outdoor camping. Which with the following conditions is most likely? Ulcerative colitis The most important clue for ulcerative colitis is bloody stools that are covered with mucus and pus along with systemic symptoms such as fatigue and low-grade fever What is the best treatment for H. pylori-related peptic ulcer disease? H2RA and clarithromycin for 14 days PPI and clarithromycin for 14 days H2RA, bismuth, metronidazole, and tetracycline for 10 to 14 days PPI, amoxicillin, and clarithromycin for 10 days A patient has fever, nausea, vomiting, anorexia, and right upper quadrant abdominal pain. An ultrasound is negative for gallstones. Which action is necessary to treat this patient’s symptoms? Prescribing ursodeoxycholic acid Supportive care with close follow-up Hospitalization for emergent treatment Empiric treatment with antibiotics Which symptoms in a patient with abdominal pain are suggestive of appendicitis? (Select all that apply.) Pain occurring prior to nausea and vomiting Pain accompanied by low-grade fever Abdominal rigidity along with pain Pain that begins in the left lower quadrant Prolonged duration of right lower quadrant pain A 74-year-old man presents with recurrent abdominal cramping and pain associated with diarrhea that occurs from 4-5 times per day. He reports that currently he is having an exacerbation. The stools are bloody with mucous and pus. The patient reports that he has lost weight and is always fatigued. The patient denies recent travel or outdoor camping. Which with the following conditions is most likely? Correct! Ulcerative colitis Irritable bowel syndrome Giardiasis Diverticulitis An 83 year old is diagnosed with diverticulitis. The most common complaint is Rectal bleeding Frequent belching and flatulence Left lower quadrant pain Bloating and crampiness A 30-year-old woman has right upper quadrant abdominal pain, nausea, and vomiting. Which diagnostic test will the provider order? MRI of the abdomen Abdominal CT with contrast Abdominal ultrasound Plain abdominal radiographs A patient's CBC demonstrated anemia. Which diagnosis is likely based on this patient's laboratory values? MCV 114 fL (normal 80-95) MCH 29 pg (normal 27-31) MCHC 33.8% (normal 32-69) RDW 15% (normal 11-14.5) Pernicious anemia Anemia of chronic disease Iron deficiency anemia Sideroblastic anemia A patient has persistent epigastric pain occurring 2 to 3 hours after a meal. Which test is definitive for diagnosis peptic ulcer in this patient? Endoscopy with biopsy of gastric mucosa A patient develops acute diarrhea and then comes to clinic two weeks later reporting profuse watery, bloody diarrheal stools 6 to 8 times daily. The provider notes a toxic appearance with moderate dehydration. Which test is indicated to diagnose this cause? Stool sample for C. difficile toxin An adult patient has intermittent, crampy abdominal pain with vomiting. The provider notes marked abdominal distention and hyperactive bowel sounds. What will the provider do initially? Obtain upright and supine radiologic views of the abdomen A patient reports a decrease in the frequency of stools and asks about treatment for constipation. Which findings are part of the Rome III criteria for diagnosing constipation? (Select all that apply.) Symptoms present for 3 months Feeling of incomplete evacuation Lumpy stools Presence of irritable bowel syndrome Fewer than 5 stools per week Most patients who has acute hepatitis A infection: Have a self limiting illness A patient present with abdominal pain and has the following laboratory findings. Would does this mean? HBsAg positive anti-HBc positive IgM Anti-HBc positive anti-HBs negative He has acute hepatitis B A patient has intermittent left-sided lower abdominal pain and fever associated with bloating and constipation alternating with diarrhea. The provider suspects acute diverticulitis. Which tests will the provider order? (Select all that apply.) Barium enema examination Plain abdominal radiographs CT scan of abdomen and pelvis Stool for occult blood Rigid sigmoidoscopy Extreme tenderness and involuntary guarding at McBurney's point is a significant finding for possible: Acute diverticulitis Acute appendicitis Acute gastroenteritis Acute cholecystitis A patient who has been taking an NSAID for osteoarthritis pain has newly diagnosed peptic ulcer disease. What is the initial step in treating this patient? Discontinue the NSAID Order prostaglandin therapy Prescribe a proton pump inhibitor Recommend an H2 receptor antagonist What choice below is most commonly associated with pancreatitis? Viral infection and cholecystitis Gallstones and alcohol abuse Hypertriglyceridemia and cholecystitis Appendicitis and renal stones A 24 year old female with pain and tenderness in the right lower abdominal quadrant. Pelvic exam ad UA are normal. WBC is elevated. Urine pregnancy test is negative. What is part of the differential diagnosis? Ectopic pregnancy Pelvic inflammatory disease UTI Appendicitis A patient reports anal pruritus and occasional bleeding with defecation. An examination of the perianal area reveals external hemorrhoids around the anal orifice as the patient is bearing down. The provider orders a colonoscopy to further evaluate this patient. What is the treatment for this patient’s symptoms? Daily laxatives to prevent straining with stools A high-fiber diet and increased fluid intake Infiltration of a local anesthetic into the hemorrhoid Referral for possible surgical intervention All of the following are true statements about diverticula except: Supplementing with fiber, such as psyllium (Metamucil), is recommended Diverticula are located in the colon Most diverticula in the colon are infected with gram negative bacteria A low fiber diet is associated with the condition What medication may be used to treat GERD if a patient has tried over the counter ranitidine without benefit? Pantoprazole Prescription strength ranitidine Calcium carbonate Prescription strength ranitidine An 8-month-old girl is brought by her grandmother to see the nurse practitioner because of intermittent, random episodes of vomiting, abdominal bloating, currant jelly stools, and irritability with poor appetite. The infant is stranding in the 10th percentile on the growth chart and appears lethargic. During the abdominal exam, a sausage like mass is palpated on the right side of the abdomen. The infant's presentation is highly suggestive of which condition? Inflammatory bowel disease Irritable bowel syndrome Intussusception Lactose intolerance The classic triad of intussusception is currant jelly stools, a sausage like mass, and pain. A sausage- shaped abdominal mass may be palpated on the right side of the abdomen. A patient has sudden onset of right upper quadrant and epigastric abdominal pain with fever, nausea, and vomiting. The emergency department provider notes yellowing of the sclerae. What is the probable cause of these findings? Common bile duct obstruction A patient has nausea associated with chemotherapy. Which agent will be prescribed? Ondansetron A patient is in the clinic with a 36 hrs history of diarrhea and moderate dehydration. Interventions should include: Oral rehydration with an electrolyte replenishment solution Which description is more typical of a patient with acute cholecystitis? The patient is ill appearing and febrile A patient with acute cholecystitis usually c/o abd pain in upper right quadrant or epigastric pain. Many also have nausea. The patient lies still on the exam table as this condition is associated with peritoneal inflammation that is worse with movement. Elderly are less likely to exhibit Murphy's sign. Asymptomatic patients have cholelithiasis. A patient who has a history of diverticular disease has left-sided pain and reports seeing blood in the stool. What is an important intervention for these symptoms? Prescribing an antispasmodic medication Ordering a CBC and stool for occult blood Referring the patient for a lower endoscopy Reminding the patient to eat a high-fiber diet A patient has sudden onset of right upper quadrant and epigastric abdominal pain with fever, nausea, and vomiting. The emergency department provider notes yellowing of the sclerae. What is the probable cause of these findings? Chronic cholelithiasis Acute acalculous cholecystitis You Answered Infectious cholecystitis Common bile duct obstruction A 70 year old has bright red blood on the toilet tissue this morning after a bowel movement. He denies pain. What is the LEAST likely cause in this patient? Diverticulitis Anal fissure Colon cancer Hemorrhoids [Anal fissure usually produces pain with bowel movements.] A patient is in clinic for evaluation of sudden onset of abdominal pain. The provider palpates a pulsatile, painful mass between the xiphoid process and the umbilicus. What is the initial action? Transfer the patient to the emergency department for a surgical consult A patient has sudden onset of right upper quadrant and epigastric abdominal pain with fever, nausea, and vomiting. The emergency department provider notes yellowing of the sclerae. What is the probable cause of these findings? Common bile duct obstruction A patient present with abdominal pain and has the following laboratory findings. Would does this mean? HBsAg positive anti-HBc positive IgM Anti-HBc positive anti-HBs negative He has acute hepatitis B -- A positive hepatitis B surface antigen and a positive IgM means that this patient has acute hepatitis B. The first serological marker to be positive is the surface antigen. It can become positive as soon as 3-4 weeks after exposure to hepatitis B. Positive IgM indicates acute infection. PreviousNext A patient reports a sudden onset of constant, sharp abdominal pain radiating to the back. The examiner notes both direct and rebound tenderness with palpation of the abdomen. What is the significance of this finding? Severe acute pancreatitis with peritonitis Presence of a pancreatic pseudocyst Compression of the common bile duct Retroperitoneal hemorrhage Most patient who have an acute hepatitis B infection: Are acutely ill Have varied clinical presentations Develop subsequent cirrhosis Are females A patient has sudden onset of right upper quadrant and epigastric abdominal pain with fever, nausea, and vomiting. The emergency department provider notes yellowing of the sclerae. What is the probable cause of these findings? ● Common bile duct obstruction ● Chronic cholelithiasis ● Infectious cholecystitis ● Acute acalculous cholecystitis A patient has persistent epigastric pain occurring 2 to 3 hours after a meal. Which test is definitive for diagnosis peptic ulcer in this patient? ● Physical exam with percussion of the upper abdomen ● Breath test or stool antigen testing for H. pylori ● Barium swallow with radiography ● Endoscopy with biopsy of gastric mucosa A patient has a recent episode of vomiting and describes the vomitus as containing mostly gastric juice. What does this symptom suggest? ● Peptic ulcer ● Small bowel obstruction ● Gastritis ● Bile duct obstruction A patient's CBC demonstrated anemia. Which diagnosis is likely based on this patient's laboratory values? MCV 114 fL (normal 80-95) MCH 29 pg (normal 27-31) MCHC 33.8% (normal 32-69) RDW 15% (normal 11-14.5) ● Sideroblastic anemia ● Pernicious anemia ● Anemia of chronic disease ● Iron deficiency anemia A patient's CBC demonstrated anemia. Which diagnosis is likely based on this patient's laboratory values? MCV 74.1 fL (normal 80-95) MCH 24 pg (normal 27-31) MCHC 33% (normal 32-69) RDW 12% (normal 11-14.5) ● Thalassemia ● Iron deficiency anemia ● Anemia of chronic disease ● Vitamin B12 deficiency anemia A patient has nausea associated with chemotherapy. Which agent will be prescribed? ● Ondansetron ● Scopolamine ● Diphenhydramine ● meclizine A patient has a history of diverticular disease and asks what can be done to minimize acute symptoms. What will the practitioner recommend to this patient? ● Consuming a diet high in fiber ● Using bran to replace high-fiber foods ● Taking an anticholinergic medication ● Avoiding saturated fats and red meats Which medications may increase the prevalence of GERD? (Select all that apply.) ● Hormone replacements ● Oral contraceptives ● Aspirin ● Benzodiazepines ● Calcium antagonists All of the following are true statements about diverticula except: ● Supplementing with fiber, such as psyllium, is recommended ● Diverticula are located in the colon ● A low fiber diet is associated with the condition ● Most diverticular in the colon are infected with gram negative bacteria A patient present with abdominal pain and has the following laboratory findings. Would does this mean? HBsAg positive anti-HBc positive IgM Anti-HBc positive anti-HBs negative ● He has no immunity to hepatitis B ● He has immunity to hepatitis B ● More data is needed ● He has acute Hepatitis B A 43-year-old female patient reports a possible exposure to hepatitis C about 4 months ago. She now presents to the clinic with concerns due to new onset abdominal pain. She has the following laboratory values. Which statement is true about this patient? HBsAg negative anti-HBc negative Anti-HBs positive anti-HCV nonreactive HCV RNA not detectable The patient does not have hepatitis C, but has immunity to hepatitis B More testing needed to determine this patient's hepatitis B status The patient has hepatitis B and hepatitis C The patient does not have hepatitis B, but could have hepatitis C Which of the following would be usual in a patient with biliary colic? Pain in upper abdomen in response to eating fatty foods Most patient who have an acute hepatitis B infection: Develop subsequent cirrhosis Are females Are acutely ill Have varied clinical presentations Which patient has the least worrisome symptoms associated with his diarrhea? One with: Duration of illness greater than 48 hours Bloody diarrhea Temp >101.3 F Moderate amount of watery diarrhea A patient has both occasional “coffee ground” emesis and melena stools. What is the most probably source of bleeding in this patient? Lower GI Hepatic Rectal Upper GI A 74 year old man presents with recurrent abdominal cramping and pain associated with diarrhea that occurs from 4-5 times per day.. Report that currently he is having an exacerbation. The stools are bloody with mucus and pus. The patient reports that he has lost weight and is always fatigued. He denies recent travel or outdoor camping. Which of the following conditions is most likely? Diverticulitis Irritable bowel syndrome Giardiasis Ulcerative colitis The most important clue is bloody stools covered in mucus and pus and systemic symptoms. The most important clue is bloody stools covered in mucus and pus and systemic symptoms. Which description is more typical of a patient with acute cholecystitis? Most are asymptomatic until a stone blocks the bile duct The elderly patient is more likely to exhibit Murphy's sign The patient rolls from side to side on the exam table The patient is ill appearing and febrile A patient with acute cholecystitis usually c/o abd pain in upper right quadrant or epigastric pain. Many also have nausea. The patient lies still on the exam table as this condition is associated with peritoneal inflammation that is worse with movement. Elderly are less likely to exhibit Murphy's sign. Asymptomatic patients have cholelithiasis. A patient reports anal pruritis and occasional bleeding with defecation. An examination of the perianal area reveals external hemorrhoids around the anal orifice as the patient is bearing down. The provider orders a colonoscopy to further evaluate this patient. What is the treatment for this patient’s symptoms? A high-fiber diet and increased fluid intake A patient has nausea associated with chemotherapy. Which agent will be prescribed? Meclizine Ondansetron Scopolamine Diphenhydramine Which is the most common cause of pancreatitis in the United States? Trauma Ethyl alcohol Gallstones Hyperlipidemia Which medications may increase the prevalence of GERD? (Select all that apply.) Benzodiazepines Calcium antagonists Hormone replacements Oral contraceptives Aspirin Most patients who has acute hepatitis A infection: Have a self limiting illness Develop fulminant disease Become acutely ill Develop subsequent cirrhosis Most patient who have an acute hepatitis B infection: Are acutely ill Have varied clinical presentations Develop subsequent cirrhosis Are females A patient who has been taking an NSAID for osteoarthritis pain has newly diagnosed peptic ulcer disease. What is the initial step in treating this patient? Prescribe a proton pump inhibitor Discontinue the NSAID Recommend an H2 receptor antagonist Order prostaglandin therapy Which medications may increase the prevalence of GERD? (Select all that apply.) Aspirin Benzodiazepines Calcium antagonists Hormone replacements Oral contraceptives Which is the most common cause of pancreatitis in the United States? Gallstones Hyperlipidemia Ethyl alcohol Trauma Most patient who have an acute hepatitis B infection: Are acutely ill Have varied clinical presentations Develop subsequent cirrhosis Are females An 8-month-old girl is brought by her grandmother to see the nurse practitioner because of intermittent, random episodes of vomiting, abdominal bloating, currant jelly stools, and irritability with poor appetite. The infant is stranding in the 10th percentile on the growth chart and appears lethargic. During the abdominal exam, a sausage like mass is palpated on the right side of the abdomen. The infant's presentation is highly suggestive of which condition? Lactose intolerance Inflammatory bowel disease Irritable bowel syndrome Intussusception The classic triad of intussusception is currant jelly stools, a sausage like mass, and pain. A sausage- shaped abdominal mass may be palpated on the right side of the abdomen. Which symptoms in a patient with abdominal pain are suggestive of appendicitis? (Select all that apply.) Pain that begins in the left lower quadrant Pain accompanied by low-grade fever Prolonged duration of right lower quadrant pain Abdominal rigidity along with pain Pain occurring prior to nausea and vomiting A patient has a recent episode of vomiting and describes the vomitus as containing mostly gastric juice. What does this symptom suggest? Small bowel obstruction Bile duct obstruction Gastritis Peptic ulcer A patient has fever, nausea, vomiting, anorexia, and right upper quadrant abdominal pain. An ultrasound is negative for gallstones. Which action is necessary to treat this patient’s symptoms? Prescribing ursodeoxycholic acid Empiric treatment with antibiotics Hospitalization for emergent treatment Supportive care with close follow-up A patient has sudden onset of right upper quadrant and epigastric abdominal pain with fever, nausea, and vomiting. The emergency department provider notes yellowing of the sclerae. What is the probable cause of these findings? Infectious cholecystitis Acute acalculous cholecystitis Common bile duct obstruction Chronic cholelithiasis What is the best treatment for H. pylori-related peptic ulcer disease? H2RA, bismuth, metronidazole, and tetracycline for 10 to 14 days H2RA and clarithromycin for 14 days PPI, amoxicillin, and clarithromycin for 10 days PPI and clarithromycin for 14 days Which of the following would be usual in a patient with biliary colic? Presence of gallstones and unpredictable abdominal pain Presence of gallstones on imaging study Positive Murphy's sign Pain in upper abdomen in response to eating fatty foods Biliary colic refers to discomfort produced by contraction of the gallbladder, which often occurs in response to eating. Which patient has the least worrisome symptoms associated with his diarrhea? One with: Bloody diarrhea Moderate amount of watery diarrhea Duration of illness greater than 48 hours Temp >101.3 F A 24 year old female with pain and tenderness in the right lower abdominal quadrant. Pelvic exam ad UA are normal. WBC is elevated. Urine pregnancy test is negative. What is part of the differential diagnosis? UTI Pelvic inflammatory disease Appendicitis Ectopic pregnancy A 70 year old has bright red blood on the toilet tissue this morning after a bowel movement. He denies pain. What is the LEAST likely cause in this patient? Diverticulitis Anal fissure Hemorrhoids Colon cancer Anal fissure usually produces pain with bowel movements. Most patients who has acute hepatitis A infection: Develop fulminant disease Develop subsequent cirrhosis Become acutely ill Correct! Have a self limiting illness Week 6 – Genitourinary (GU) The provider is counseling a patient who has stress incontinence about ways to minimize accidents. What will the provider suggest initially? Voiding every 2 hours during the day A pregnant woman at 30 weeks gestation has proteinuria. What will the provider do next? Evaluate her blood pressure The provider is evaluating a patient for potential causes of urinary incontinence and performs a postvoid residual (PVR) test which yields 30 mL of urine. What is the interpretation of this result? This is a normal result. A patient's recent blood work indicates acute kidney injury. You know that acute kidney injury can be caused from: Heart failure exacerbation GERD Increase in metoprolol dose atrial fibrillation Heart failure exacerbation and cause decreased perfusion to the kidneys, leading to acute kidney injury. Changes in medications or nephrotoxic can cause acute kidney injury, metoprolol is not one of them. While patients with atrial fibrillation can have decreased cardiac output, it is often compensated to preserve renal perfusion A male patient complaints of dysuria. His urinalysis is positive for nitrates, leukocytes, and bacteria. What medication should be given and for how many days? Trimethoprim-sulfamethoxazole for 7-10 days First line treatment is similar to females which include TMP/SMX, nitrofurantoin, fosfomycin. The correct length of treatment is only in TMP/SMX in the possible answer choices. A patient who has diabetes has symptoms consistent with renal stones. Which type of stone is most likely in this patient? Uric acid A 30-year-old male patient has a positive leukocyte esterase and nitrites on a random urine dipstick during a well patient exam. What type of urinary tract infection does this represent? Uncomplicated Unresoved Complicated Isolation An older male patient reports gross hematuria but denies flank pain and fever. What will the provider do to manage this patient? Perform a 24-hour urine collection Refer for cystoscopy and imaging Monitor blood pressure closely Obtain a urine culture A 16-year-old female patient is being treated for her first UTI. She had an allergic reaction with hives after taking sulfa as a child. Which of the following antibiotics would be contraindicated? Trimethoprim-sulfamethoxazole A female patient with the complaint of dysuria has a urine specimen that is positive for leukocytes and nitrates. There is blood in the specimen. The most appropriate diagnosis is: UTI with hematuria The presence of leukocytes and nitrates in the urine indicates likely infection in the bladder, and more likely from a gram-negative pathogen such as Escherichia coli. The presence of blood is common when patient's has a UTI. A diagnosis of chlamydia cannot be made based on the symptoms and urinalysis results. An adolescent male reports severe pain in one testicle. The examiner notes edema and erythema of the scrotum on that side with a swollen, tender spermatic cord and absence of the cremasteric reflex. What is the most important intervention? Immediate referral to the emergency department Transillumination to assess for a “blue dot” sign Prescribing anti-infective agents to treat the infection Doppler ultrasound to assess testicular blood flow A female patient reports hematuria and a urine dipstick and culture indicate a urinary tract infection. After treatment for the UTI, what testing is indicated for this patient? Bladder scan Voiding cystourethrogram No testing if hematuria is resolved 24-hour urine collection to evaluate for glomerulonephritis A female patient who is 45-year-old states that she is having urinary frequency. She describes episodes of "having to go right now" and not being able to wait. Her urinalysis results are within normal limits. What this part of the differential? Asymptomatic bacteriuria Stress incontinence Lupus Diabetes Patients with diabetes can present with polyuria. In assessment of patient's risk factors should be done with strong consideration even to checking glucose level. Other possible diagnoses include urge incontinence and vaginitis. A urinalysis would show bacteriuria. A healthy 32 year old female has left-sided flank pain, nausea, and fever. Her urinalysis demonstrated hematuria and the presence of leukocytes, nitrates, WBC casts. What is the most likely diagnosis? Cholecystitis Pyelonephritis UTI Renal stone The most common presentation of acute pyelonephritis includes fever, flank pain, and nausea and vomiting. Fever is so strongly correlated with acute pyelonephritis that it is unusual not to have a fever. Renal stone patients may have this presentation, but usually fever is not present. It is unlikely that cholecystitis would present with left sided flank pain. A 3-month-old male infant has edema and painless swelling of the scrotum. On physical examination, the provider is able to transilluminate the scrotum. What will the provider recommend? A short course of empiric antibiotic therapy A Doppler ultrasound to evaluate the scrotal structures Observation and reassurance that spontaneous resolution may occur Immediate referral to a genitourinary surgeon for repair 17-year-old boy reports feeling something on his left scrotum. On palpation, soft and movable blood vessels that feel like "a bag of worms" are noted underneath the scrotal skin. The testicle is not swollen or reddened. The most likely diagnosis is: Varicocele Chronic orchitis Testicular torsion Chronic epididymitis Palpation of varicose veins, described as "a bag of worms", is a classic symptom of varicocele. A patient with urolithiasis is more likely to Demonstrate RBC casts Have frequent UTIs Have chills and fever Be of male gender Males are more likely than females to have urolithiasis. There is no increased incidence of stone formation among patients with frequent UTIs. Patients with your urolithiasis may exhibit fever and chills of infection if associated with a very large stone, but this is not the usual case. RBC casts are formed in the renal tubules, this generally indicates glomerular injury, not urolithiasis. The daughter of an elderly, confused female patient reports that her mother is having urinary incontinence several times each day. What will the provider do initially? Perform a bladder scan to determine distention and retention Tell the daughter that this is expected given her mother’s age and confusion Order serum creatinine and blood urea nitrogen tests Obtain a urine sample for urinalysis and possible culture An asymptomatic pregnant woman has a positive leukocyte esterase and positive nitrites on a urine dipstick screening. What will the provider do next? Admit to the hospital Obtain a urine culture Prescribe trimethoprim-sulfamethoxazole Order a renal ultrasound Mrs. Jackson comes into her clinic complaining of new onset urinary incontinence when she laughs or sneezes. What should be used first line to treat her symptoms? Avoid caffeine and alcohol Kegel exercises Minimize fluids at nighttime Prescribe oxybutynin A patient has acute renal colic, nausea, and vomiting and a urinalysis reveals hematuria, but is otherwise normal. A radiographic exam shows several radiopaque stones in the ureter which are less than 1 mm in diameter. What will the primary provider do initially to manage this patient? Prescribe nifedipine and hospitalize for intravenous antibiotics Prescribe desmopressin and a corticosteroid medication orrec Order a narcotic pain medication and increased oral fluids Obtain a consultation with a urology specialist A 50-year-old male patient reports that he has a sensation of scrotal heaviness. He reports that the sensation is worse at the end of the day. He denies pain. What is likely etiology of these symptoms? Inguinal hernia A pregnant patient is found to have a urinary tract infection. What is the appropriate course of action? Prescribe nitrofurantoin Which factors increase the risk of renal stones? (Select all that apply.) Vitamin D excess Surgical menopause Snow skiing Excess antacid use Strenuous exercise A patient has urinary burning, frequency, and urgency. Her urinalysis is positive for leukocytes and negative for nitrates. A likely explanation is that the patient: Could be pregnant Has diabetes Has a sexually transmitted disease Has a nonobstructive kidney stone The patient is presenting with classic symptoms of a UTI, however this is not one of the possible answers. An alternative diagnosis that the provider should consider is STD. this is not a common presentation for a nonobstructive kidney stone. Pregnancy and diabetes may be associated with frequency and urgency, but not burning. The provider is counseling a patient who has stress incontinence about ways to minimize accidents. What will the provider suggest initially? Increasing fluid intake to dilute the urine Taking pseudoephedrine daily Voiding every 2 hours during the day Referral to a physical therapist Which factors increase the risk of renal stones? (Select all that apply.) Excess antacid use Surgical menopause Vitamin D excess Snow skiing Strenuous exercise A female patient with the complaint of dysuria has a urine specimen that is positive for leukocytes and nitrates. There is blood in the specimen. The most appropriate diagnosis is: Urinary tract infection No answer text provided. UTI with hematuria Asymptomatic bacteriuria UTI or chlamydia A male patient complaints of dysuria. His urinalysis is positive for nitrates, leukocytes, and bacteria. What medication should be given and for how many days? Ciprofloxacin for 3 days Trimethoprim-sulfamethoxazole for 7-10 days Doxycycline for 7 days Nitrofurantoin for 14 days A female patient who is 45-year-old states that she is having urinary frequency. She describes episodes of "having to go right now" and not being able to wait. Her urinalysis results are within normal limits. What this part of the differential? Diabetes Lupus Stress incontinence Asymptomatic bacteriuria Patients with diabetes can present with polyuria. In assessment of patient's risk factors should be done with strong consideration even to checking glucose level. Other possible diagnoses include urge incontinence and vaginitis. A urinalysis would show bacteriuria. Noninfectious epididymitis is common in: Truck drivers Men to wear boxer style underwear Soccer players Marathon runners Noninfectious epididymitis occurs when there is a reflux of urine into the epididymis from the ejaculatory duct and vas deferens. This can occur if males spend a lot of time sitting. Other typical risk factors include vigorous exercise that involve heavy lifting or upper body workout. A 20-year-old male has epididymitis. His most common complaint will be: Scrotal pain The most common complaint for epididymitis is scrotal pain. Usually develops over a period of days. Burning with urination is possible if the underlying cause is a urinary tract infection. However, this is more common in older males. Testicular pain is not a common complaint with epididymitis. Penile discharge may occur with gonorrhea or Chlamydia infections. A young adult male reports a gradual onset 3/10 dull pain in the right scrotum and the provider notes a bluish color showing through the skin on the affected side. Palpation reveals a bag of worms on the proximal spermatic cord. What is an important next step in managing this patient? Reassurance that this is benign and may resolve spontaneously Consideration of underlying causes of this finding Referral to an emergency department for surgical consultation Anti-infective therapy with ceftriaxone or doxycycline A pregnant patient is found to have a urinary tract infection. What is the appropriate course of action? Prescribe nitrofurantoin Prescribe TMP/SMX Prescribe ciprofloxacin Prescribe no antibiotics Nitrofurantoin is considered probably safer to use during pregnancy. It provides coverage for most common UTI pathogens. TMP/SMX is a full of acid antagonist and may be associated with increased risk of congenital malformation. Ciprofloxacin is not to be used first line for any simple UTI, and may not be safe during pregnancy. In some occasions it could still be given if benefits outweigh risks. 17-year-old boy reports feeling something on his left scrotum. On palpation, soft and movable blood vessels that feel like "a bag of worms" are noted underneath the scrotal skin. The testicle is not swollen or reddened. The most likely diagnosis is: Varicocele A physically independent 75 year old was diagnosed with mild cognitive impairment 6 months ago. She resides in an assisted living facility. she is in clinic today for scheduled visit. Her adult daughter reports that about 2 weeks ago her mother had an episode of urinary incontinence, but no episode since then. She is found to have asymptomatic bacteriuria. How should this be managed? Repeat the urinalysis in 4 weeks Repeat the urinalysis in 7 days Treat her today with one dose of an antibiotic Monitor her for symptoms of urinary tract infection Explanation: Approximately 30-50% of older females living in institutions have asymptomatic bacteriuria. No data support treatment of patients to prevent future problems or complications. In fact, asymptomatic bacteriuria is not usually treated unless the patient is pregnant, immunocompromised, or is undergoing a urinary procedure. 1) The provider is evaluating a patient for potential causes of urinary incontinence and performs a postvoid residual (PVR) test which yields 30 mL of urine. What is the interpretation of this result? This is a normal result. 2) A female patient reports hematuria and a urine dipstick and culture indicate a urinary tract infection. After treatment for the UTI, what testing is indicated for this patient? No testing if hematuria is resolved 3) Which factors increase the risk of renal stones? (Select all that apply.) Strenuous exercise Surgical menopause Excess antacid use 4) A healthy 32 year old female has left-sided flank pain, nausea, and fever. Her urinalysis demonstrated hematuria and the presence of leukocytes, nitrates, WBC casts. What is the most likely diagnosis? Pyelonephritis The most common presentation of acute pyelonephritis includes fever, flank pain, and nausea and vomiting. Fever is so strongly correlated with acute pyelonephritis that it is unusual not to have a fever. Renal stone patients may have this presentation, but usually fever is not present. It is unlikely that cholecystitis would present with left sided flank pain. 5) A female patient who is 45-year-old states that she is having urinary frequency. She describes episodes of "having to go right now" and not being able to wait. Her urinalysis results are within normal limits. What this part of the differential? Diabetes Patients with diabetes can present with polyuria. In assessment of patient's risk factors should be done with strong consideration even to checking glucose level. Other possible diagnoses include urge incontinence and vaginitis. A urinalysis would show bacteriuria. 1) The provider is evaluating a patient for potential causes of urinary incontinence and performs a postvoid residual (PVR) test which yields 30 mL of urine. What is the interpretation of this result?rrect! This is a normal result. 2) A patient has acute renal colic, nausea, and vomiting and a urinalysis reveals hematuria, but is otherwise normal. A radiographic exam shows several radiopaque stones in the ureter which are less than 1 mm in diameter. What will the primary provider do initially to manage this patient? Correct! Order a narcotic pain medication and increased oral fluids 3) A 30-year-old male patient has a positive leukocyte esterase and nitrites on a random urine dipstick during a well patient exam. type of urinary tract infection does this represent?ect! Complicated 4) A patient has urinary burning, frequency, and urgency. Her urinalysis is positive for leukocytes and negative for nitrates. A likely explanation is that the patient: Correct! Has a sexually transmitted disease The patient is presenting with classic symptoms of a UTI, however this is not one of the possible answers. An alternative diagnosis that the provider should consider is STD. this is not a common presentation for a nonobstructive kidney stone. Pregnancy and diabetes may be associated with frequency and urgency, but not burning. 5) Mrs. Jackson comes into her clinic complaining of new onset urinary incontinence when she laughs or sneezes. What should be used first line to treat her symptoms? Correct! Kegel exercises The patient has stress incontinence. The first-line approach with this patient is to attempt to strengthen the pelvic floor muscles. Anticholinergic may worsen incontinence because it will cause urinary retention. Avoiding caffeine and alcohol are especially helpful for patients with urge incontinence, would likely have minimal benefit with this patient. Minimizing fluids at nighttime will help if nocturia is a problem. A pregnant patient has asymptomatic bacteriuria. What is the likely pathogen? Staph aureus Klebsiella No pathogen Escherichia coli Of pregnant patient with asymptomatic bacteriuria should be treated with antibiotics because she is at high risk of developing pyelonephritis and/or preterm labor. The most common pathogen is Escherichia coli. An older male patient reports gross hematuria but denies flank pain and fever. What will the provider do to manage this patient? 1. Obtain a urine culture 2. Refer to cystoscopy and imaging 3. Perform a 24-hour urine collection 4. Monitor blood pressure closely A young adult male reports a gradual onset 3/10 dull pain in the right scrotum and the provider notes a bluish color showing through the skin on the affected side. Palpation reveals a bag of worms on the proximal spermatic cord. What is an important next step in managing this patient? 1. Consideration of underlying causes of this finding 2. Referral to an emergency department for surgical consultation 3. Reassurance that this is benign and may resolve spontaneously Which factors increase the risk of renal stones? (Select all that apply.) 1. Surgical menopause 2. Vitamin D excess 3. Strenuous exercise 4. Snow skiing 5. Excess antacid use A healthy 32 year old female has left-sided flank pain, nausea, and fever. Her urinalysis demonstrated hematuria and the presence of leukocytes, nitrates, WBC casts. What is the most likely diagnosis? 1. Cholecystitis 2. Renal stone 3. Pyelonephritis 4. UTI A patient's recent blood work indicates acute kidney injury. You know that acute kidney injury can be caused from: 1. GERD 2. Heart Failure exacerbation 3. Increase in metoprolol dose 4. Atrial fibrillation The provider is counseling a patient who has stress incontinence about ways to minimize accidents. What will the provider suggest initially? 1. Voiding every 2 hours during the day 2. Taking pseudoephedrine daily 3. Increasing fluid intake to dilute the urine 4. Referral to a physical therapist A pregnant woman at 30 weeks gestation has proteinuria. What will the provider do next? 1. Monitor serum blood glucose for gestational diabetes 2. Evaluate her blood pressure 3. Perform a 24-hour urine collection 4. Reassure her that this normal at this stage of pregnancy An adolescent male reports severe pain in one testicle. The examiner notes edema and erythema of the scrotum on that side with a swollen, tender spermatic cord and absence of the cremasteric reflex. What is the most important intervention? 1. Transillumination to assess for a “blue dot” sign 2. Doppler ultrasound to assess testicular blood flow 3. Prescribing anti-infective agents to treat the infection 4. Immediate referral to the emergency department A physically independent 75 year old was diagnosed with mild cognitive impairment 6 months ago. She resides in an assisted living facility. she is in clinic today for scheduled visit. Her adult daughter reports that about 2 weeks ago her mother had an episode of urinary incontinence, but no episode since then. She is found to have asymptomatic bacteriuria. How should this be managed? 1. Monitor her for symptoms of urinary tract infection 2. Treat her today with one dose of antibiotic 3. Repeat urinalysis in 4 weeks 4. Repeat the urinalysis in 7 days A young adult male reports a gradual onset 3/10 dull pain in the right scrotum and the provider notes a bluish color showing through the skin on the affected side. Palpation reveals a bag of worms on the proximal spermatic cord. What is an important next step in managing this patient? Reassurance that this is benign and may resolve spontaneously Referral to an emergency department for surgical consultation Anti-infective therapy with ceftriaxone or doxycycline Correct! Consideration of underlying causes of this finding A female patient reports hematuria and a urine dipstick and culture indicate a urinary tract infection. After treatment for the UTI, what testing is indicated for this patient? Voiding cystourethrogram No testing if hematuria is resolved Bladder scan 24-hour urine collection to evaluate for glomerulonephritis The daughter of an elderly, confused female patient reports that her mother is having urinary incontinence several times each day. What will the provider do initially? Order serum creatinine and blood urea nitrogen tests Perform a bladder scan to determine distention and retention Obtain a urine sample for urinalysis and possible culture Tell the daughter that this is expected given her mother’s age and confusion A healthy 32 year old female has left-sided flank pain, nausea, and fever. Her urinalysis demonstrated hematuria and the presence of leukocytes, nitrates, WBC casts. What is the most likely diagnosis? Cholecystitis Pyelonephritis UTI Renal stone The most common presentation of acute pyelonephritis includes fever, flank pain, and nausea and vomiting. Fever is so strongly correlated with acute pyelonephritis that it is unusual not to have a fever. Renal stone patients may have this presentation, but usually fever is not present. It is unlikely that cholecystitis would present with left sided flank pain. A female patient who is 45-year-old states that she is having urinary frequency. She describes episodes of "having to go right now" and not being able to wait. Her urinalysis results are within normal limits. What this part of the differential? Stress incontinence Diabetes Asymptomatic bacteriuria Lupus Patients with diabetes can present with polyuria. In assessment of patient's risk factors should be done with strong consideration even to checking glucose level. Other possible diagnoses include urge incontinence and vaginitis. A urinalysis would show bacteriuria. A female patient reports hematuria and a urine dipstick and culture indicate a urinary tract infection. After treatment for the UTI, what testing is indicated for this patient? No testing if hematuria is resolved Bladder scan 24-hour urine collection to evaluate for glomerulonephritis Voiding cystourethrogram A patient who has diabetes has symptoms consistent with renal stones. Which type of stone is most likely in this patient? Cysteine Citrate Uric acid Oxalate A 3-month-old male infant has edema and painless swelling of the scrotum. On physical examination, the provider is able to transilluminate the scrotum. What will the provider recommend? Immediate referral to a genitourinary surgeon for repair A Doppler ultrasound to evaluate the scrotal structures A short course of empiric antibiotic therapy Observation and reassurance that spontaneous resolution may occur 17-year-old boy reports feeling something on his left scrotum. On palpation, soft and movable blood vessels that feel like "a bag of worms" are noted underneath the scrotal skin. The testicle is not swollen or reddened. The most likely diagnosis is: Chronic orchitis Chronic epididymitis Testicular torsion Varicocele Palpation of varicose veins, described as "a bag of worms", is a classic symptom of varicocele. A 16-year-old female patient is being treated for her first UTI. She had an allergic reaction with hives after taking sulfa as a child. Which of the following antibiotics would be contraindicated? Cephalexin Trimethoprim-sulfamethoxazole Ampicillin Nitrofurantoin Week 7 – Gynecology (GYN) and Sexually Transmitted Infections (STI) When seen on a wet mount like the following, clue cells would indicate the treatment by which of the following? Rocephin (Ceftriaxone) 250mg IM x 1 and Azithromycin 1 g PO x 1 Flagyl (Metronidazole) 500mg PO BID x 7 days No treatment needed Diflucan (Fluconazole) 150mg PO x 1 The uterus should sound to cm when measuring for Mirena IUD insertion to allow for the arms to open and proper alignment to occur. cm; measured and forgotten about. cm; measured and documented in the chart. 1-3 cm; measured and forgotten about. 3-5 cm; measured and documented in the chart. If cervical stenosis is met when performing IUD insertions, which of the following should be used to overcome resistance? 6 - 9 cm 12 - 15 cm 9 - 12 cm 3 - 6 cm A female patient and her male partner are diagnosed with trichomonas. She has complaints of vulvall itching and discharge. He is asymptomatic. How should they be treated? They both should receive metronidazole A 40 year old female patient returns to your clinic to review her pap smear results from the previous week. You tell her the test is abnormal with “atypical squamous cells of undetermined significance and HPV positive”. What is the appropriate next step of the following? Repeat cytology in 1 year Repeat cytology immediately Perform or refer out for colposcopy Repeat cytology in 2-4 months After an IUD is placed, the threads should be cut so approximately are visible. This should then be . 3 cm; measured and documented in the chart. 3 cm; measured and forgotten about. 6 cm; measured and documented in the chart. 6 cm; measured and forgotten about. You suspect that the patient you are seeing has HIV. which of the following is a sensitive screening test for human immunodeficiency virus? Combination HIV-1 and HIV-2 antibody immunoassay with P 24 antigen HIV antibody test ELISA test Western blot test A 35 year old female presents for her well-woman exam with no complaints and no significant medical history. She admits to occasional unprotected intercourse with her partner, which is an unstable, but safe relationship. Upon exam, you note a green, frothy and malodorous discharge she states she has noticed on occasion, but thought it was normal. Wet mount shows the following image with a positive “Whiff Test”. What is your likely diagnosis?: Bacterial Vaginosis Vaginal candidiasis Trichomonas vaginalis Atrophic vulvovaginitis Clue cells are found in patients who have: Bacterial vaginosis [Clue cells are hallmark sign of bacterial vaginosis and can be seen in a microscopic exam.] A 21-year-old college student has recently been informed that he has HPV infection on the shaft of his penis. With the following methods can be used to visualize subclinical HPV lesions on the penile skin? Apply acetic acid to the penile shaf t to look for acetowhite changes Lesions of HPV infection will turn white with application of acetic acid. Routine use of this procedure to detect mucosal changes due to HPV is not recommended because results do not influence clinical management (per CDC). An initial pharmacological approach to the patient was diagnosed with primary dysmenorrhea could be: Acetaminophen NSAIDs prior to the onset of menses Combination of acetaminophen and NSAIDs NSAIDs at the time symptoms begin or onset of menses Pain associated with dysmenorrhea is likely due to prostaglandins. NSAIDs are prostaglandin synthesis inhibitors. They are usually started at the onset of menses or onset of symptoms and continued for 2-3 days depending on symptom pattern. There is no demonstrated increase in efficacy when acetaminophen is added or given alone. A 45 year old diabetic female presents with c/o vaginal itching and discharge that began after douching post-menstruation approximately one week ago. Upon exam, you find thick, white discharge with a curd- like consistency and erythema generally in the vuvlvogavinal region. Under a wet mount you see the following below. Which of the following is an appropriate treatment for this patient? Rocephin (Ceftriaxone) 250mg IM x 1 and Azithromycin 1 g PO x 1 Flagyl (Metronidazole) 500mg PO BID x 7 days Diflucan (Fluconazole) 150mg PO x 1 No treatment needed A male patient presents with dysuria and penile discharge. He states that his female partner has been diagnosed with an STD, but he is not sure which one. Which of these should be part of the differential? Syphilis and Chlamydia HIV and herpes Chlamydia and gonorrhea Bacterial vaginosis and Trichomonas A 65 year old female presents with c/o vaginal soreness and dysuria that has been intermittent for several years. She notes the pain is worse after intercourse with her husband of 30 years, with whom is in a monogamous relationship. She denies vaginal discharge and has not had a pap smear since her total hysterectomy ten years ago. She currently only takes a multivitamin. Your wet mount reveals few lactobacilli and increased parabasal cells. What is your likely diagnosis? Bacterial Vaginosis Atrophic vulvovaginitis Trichomonas vaginalis Vaginal candidiasis Syphilis may present as: Discharge Dysuria Painful lesions A rash Secondary syphilis can present as a rash, more commonly on the palms of the hands or soles of the feet. Lesions are usually painless. It usually does not produce significant dysuria or discharge. A 54-year-old female presents with small to moderate amount of vaginal bleeding of recent onset. She has been postmenopausal for approximately 2 years. With diagnosis is least likely? Endometrial hyperplasia Ovarian cancer Uterine polyps Endometrial carcinoma Ovarian cancer may present as an adnexal mass, pelvic or abdominal symptoms and a variety of others. Postmenopausal bleeding is an uncommon presentation of ovarian cancer, but can present this way. Other pathologies are usually evaluated before considering ovarian pathology. A 25-year-old female presents with lower abdominal pain. Which finding would indicate the etiology as pelvic inflammatory disease? Temperature greater than 101°F A nurse practitioner identifies filamentous structures and many uniform, oval-shaped structures during a microscopic exam of vaginal discharge. These are probably: Hyphae Typically filamentous structures, hyphae by the mechanism that allows fungal growth. Yeast may look like uniform oval shaped structures. Visualization of this should prompt immediate diagnosis of a fungal infection. Candida albicans is a specific fungus. Since many fungi can produce hyphae and yeast, it is not possible to diagnose Candida albicans specifically. A 31-year-old female patient presents with fatigue, fever, worsening unilateral low back pain for the past 5 days. Her pain is 5 out of 10 on the pain scale which has been unresponsive to ibuprofen. she denies abdominal pain, but is anorexic and nauseous. She denies vaginal discharge. Urinalysis demonstrates hematuria, the presence of WBC casts, leukocytes, nitrates. Which should be included in differential diagnosis? Pyelonephritis The patient's complaint of unilateral low back pain is likely secondary to pyelonephritis. The presence of WBC casts in the urine strongly suggest a renal origin for pyuria. A patient who presents with this scenario has to be considered to have pyelonephritis until proven otherwise. Most women with PID have bilateral abdominal tenderness, usually in the lower quadrants. Purulent vaginal discharge and fever also common. A 27-year-old asymptomatic male presents with generalized lymphadenopathy. He has multiple sexual partners and infrequently uses condoms. Of the following choices, what tests should be performed? Lymph node biopsy RPR Comprehensive metabolic panel HIV test Asymptomatic HIV infections often have persistent generalized lymphadenopathy. The uterus should sound to cm when measuring for Mirena IUD insertion to allow for the arms to open and proper alignment to occur. 3 cm; measured and documented in the chart. 6 cm; measured and forgotten about. 3 cm; measured and forgotten about. 6 cm; measured and documented in the chart. What is your treatment for Atrophic Vulvovaginitis? Diflucan (Fluconazole) 150mg PO x 1 Clindamycin 2% 5g applicator PV x 7 days Premarin cream 0.5g PV 1-3 x wk Flagyl (Metronidazole) 2g PO x 1 Chancroid considered a cofactor for transmission of: HIV Chlamydia Trichomonas Gonorrhea Chancroid is an STD. It is spread by sexual contact or by contacting pus from an infected lesion. The ulcers usually. Painful and then, but usually not painful in women. It is a cofactor in the transmission of HIV. In a private NP clinic, patient presents with Trichomonas. State law requires reporting of STD to the public health department. The patient asks the NP not to report it because her husband works in the public health department. How show this be managed by the NP? Tell the patient that it will not be reported, but report it anyways Respect the patient's right to privacy and not report it Report it to the public health department but don't divulge all the de Report it to the public health department as required by law [If state law requires it, it should be reported. Patient names or other identifying data are not part of the reporting process, so the patient should not worry about being identified and associated with this finding.] A 26-year-old female patient has been diagnosed with gonorrhea. However should she be managed? Ceftriaxone only Ceftriaxone and azithromycin Cefixime and azithromycin Penicillin G Usual treatment for gonorrhea/Chlamydia includes ceftriaxone 250 mg IM in conjunction with 1 g azithromycin by mouth. A sexually active adolescent male has a warty growth on the shaft of his penis. It is painless. This is likely: Herpes Syphilis HPV Trichomonas This is not a clinical presentation of trichomonas because this produces a discharge. Syphilis produces a painless lesion that presents as an ulceration with a hard edge and clean yellow base. Herpes produces lesions but are usually painful. HPV produces warty growths as described above. A 19-year-old student who is on prescription combined oral contraceptive pills is being seen for lower GI pain. The nurse practitioner has obtained a Pap smear and is about to perform the bimanual exam. She gently remove the plastic speculum from the vagina. While the NP is performing the bimanual vaginal exam, the patient complaints of slight discomfort during palpation of the ovaries. Which with the following is a true statement? The uterus and ovaries are both sensitive to any Palpation The ovaries are sensitive to deep palpation but they should not be painful. The fallopian tubes and ovaries are not sensitive to light or deep palpation The uterus and ovaries are not important organs of reproduction Unilateral adnexal pain accompanied by cervical motion tenderness and purulent endocervical discharge suggestive of PID What is included in your treatment plan for #4? (Though not sure which question it’s actually referring to.) Flagyl (Metronidazole) 2g PO x 1 with partner treatment and report to the county Rocephin (Ceftriaxone) 250mg IM of patient only and no report to the county Flagyl (Metronidazole) 2g PO x 1 of patient only and no report to the county Rocephin (Ceftriaxone) 250mg IM with partner treatment and report to the county 50- year-old male comes to in see the nurse practitioner for evaluation. He complains of fever, chills, pelvic pain, dysuria. He should be diagnosed with: Nonbacterial prostatitis Acute bacterial prostatitis Urinary tract infection Chronic bacterial prostatitis Acute bacterial prostatitis should always be considered first in male patient to present with these symptoms. Chronic bacterial prostatitis presents with a more subtle presentation. UTIs far less common in men than women and is usually associated with anal intercourse or being uncircumcised. Nonbacterial prostatitis presents like chronic prostatitis except urine and prostate secretion cultures are negative. If cervical stenosis is met when performing IUD insertions, which of the following should be used to overcome resistance? 1. 3-6 cm 2. 9-12 cm 3. 12-15 cm 4. 6-9 cm A 35 year old female presents for her well-woman exam with no complaints and no significant medical history. She admits to occasional unprotected intercourse with her partner, which is an unstable, but safe relationship. Upon exam, you note a green, frothy and malodorous discharge she states she has noticed on occasion, but thought it was normal. Wet mount shows the following image with a positive “Whiff Test”. What is your likely diagnosis?: 1. Trichomonas vaginalis 2. Vaginal candidiasis 3. Atrophic vulvovaginitis 4. Bacteria Vaginosis What is your treatment for Atrophic Vulvovaginitis? 1. Diflucan 2. Clindamycin 3. Flagyl 4. Premarin cream A sexually active adolescent male has a warty growth on the shaft of his penis. It is painless. This is likely: 1. Trichomonas 2. Syphilis 3. HPV 4. Herpes A male patient presents with dysuria and penile discharge. He states that his female partner has been diagnosed with an STD, but he is not sure which one. Which of these should be part of the differential? 1. Chlamydia and gonorrhea 2. Syphilis and chlamydia 3. Bacterial vaginosis and Trichomonas 4. HIV and herpes A 65 year old female presents with c/o vaginal soreness and dysuria that has been intermittent for several years. She notes the pain is worse after intercourse with her husband of 30 years, with whom is in an monogamous relationship. She denies vaginal discharge and has not had a pap smear since her total hysterectomy ten years ago. She currently only takes a multivitamin. Your wet mount reveals few lactobacilli and increased parabasal cells. What is your likely diagnosis? 1. Trichomonas vaginalis 2. Bacterial vaginosis 3. Vaginal candidiasis 4. Atrophic vulvovaginitis A 40 year old female patient returns to your clinic to review her pap smear results from the previous week. You tell her the test is abnormal with “atypical squamous cells of undetermined significance and HPV positive”. What is the appropriate next step of the following? 1. Repeat cytology in 1 year 2. Repeat cytology immediately 3. Perform or refer out for colposcopy 4. Repeat cytology in 2-4 months A male patient presents with dysuria and penile discharge. He states that his female partner has been diagnosed with an STD, but he is not sure which one. Which of these should be part of the differential? 1. Bacterial vaginosis and trichomonas 2. Syphilis and chlamydia 3. Chlamydia and gonorrhea 4. HIV and herpes Syphilis may present as: 1. Discharge 2. Painful lesions 3. A rash 4. Dysuria In a private NP clinic, patient presents with Trichomonas. State law requires reporting of STD to the public health department. The patient asks the NP not to reported because her husband works in the public health department. How show this be managed by the NP? Report it to the public health department but don't divulge all the details Report it to the public health department as required by law Tell the patient that it will not be reported, but report it anyways Respect the patient's right privacy and not report it If state law requires it, it should be reported. Patient names or other identifying data are not part of the reporting process, so the patient should not worry about being identified and associated with this finding. Week 8 – Obstetrics (OB) A woman comes into clinic due to migraines with aura, but also requests oral contraceptives to prevent pregnancy. Which type of contraceptive will the provider recommend? Contraceptive implant Transdermal combination product Progestin-only contraception Combination oral contraceptive A mother who has been breastfeeding her infant for several weeks develops a fever x 24 hours, malaise, breast erythema, and breast tenderness. What will the provider recommend? Increased frequency of nursing and antibiotics initiated that covers S. aureus Ice packs and increased frequency of nursing Ice packs and cessation of nursing with breast pumping initiated Cessation of nursing and antibiotics initiated that covers S. aureus A patient in her first trimester of pregnancy is found to be infected with Chlamydia and gonorrhea. Which statement below is true? She should be treated now and rescreened later in pregnancy She should be screened for other STDs later in pregnancy She should be treated now and rescreened if symptoms reappear She should be treated in the second trimester This patient should certainly be treated for Chlamydia and gonorrhea with azithromycin 1 g x1 and ceftriaxone 250 mg IM ×1. since the percentage of patients who become reinfected with an STD later in pregnancy is high, this patient should be rescreened later in pregnancy regardless of symptoms. Disastrous effects can occur if she is infected and left untreated while pregnant. A 44-year-old female who is undergoing treatment for infertility complains of not having had a menstrual period for a few months. The night before, she started spotting and is now having cramp-like pains in her pelvic area. Her blood pressure is 160/80, pulse 110, she is afebrile. Her labs revealed mild anemia, mild leukocytosis. On exam, the uterine fundus is noted to be above the symphysis pubis. The cervical os is dilated to 3 cm. Which of the following is most likely? Inevitable abortion The classic symptoms of ectopic pregnancy are: Amenorrhea, vaginal bleeding, abdominal pain A woman at 32 weeks of gestation has a positive throat culture for Streptococcus pyogenes (strep throat). she denies allergies but becomes very nauseated with erythromycin. Which of the following is the best choice for this pregnant patient? Penicillin A 3-day-old, full-term female infant who is breastfeeding develops some jaundice. The transcutaneous bilirubin(TcB) reading in the office is in the intermediate risk zone. What will the provider tell the mother? To switch to formula and return to clinic within 5 days for reassessment To use a breast pump to increase her milk supply and increase baby to breast and return to clinic within 5 days for reassessment To decrease the frequency of breastfeeding and return to clinic within 5 days for reassessment To supplement feedings with extra water and return to clinic within 5 days for reassessment A woman who is taking oral contraceptive pills (OCPs) to prevent pregnancy calls the provider to report forgetting to take the pills for 4 days. She has not had sexual intercourse during that time. What will the provider recommend? Take 2 pills daily for 4 days and use an alternative method for 4 days Stop the OCP, use an alternative method, and resume OCPs at the next cycle Use a morning after pill today and resume the OCPs now Resume the pills and use a backup method the remainder of the cycle A pregnant woman who is overweight has no previous history of hypertension or diabetes. Her initial screening exam reveals a blood pressure of 140/90, she is asymptomatic. And a fasting blood glucose of 128 mg/dL. What will the practitioner do? Initiate insulin therapy Monitor blood pressure and fasting blood glucose closely. Do 2 hr GTT (glucose tolerance test) early, before 24 weeks. Prescribe an antihypertensive medication Refer the patient to a high-risk pregnancy specialist A pregnant patient complains of urinary frequency and dysuria. Her urinalysis is positive for nitrates, leukocyte esterase, and bacteria. Will course of action is most appropriate? Prescribe an antibiotic only Order urine culture and begin antibiotics Repeated the urinalysis Order a urine culture only quadrant quadrant A female calls the provider to report having unprotected sexual intercourse approximately 4 days prior. Which regimen will the provider recommend? Ulipristal Acetate taken one time Plan-B One-Step twice daily for 5 days Plan-B One-Step daily for 5 days Ulipristal Acetate twice daily for 3 days A 30-year-old female presents with lower abdominal pain. The nurse practitioner immediately considers ectopic pregnancy as the cause. Which factor listed below does not increase her risk of an ectopic pregnancy? IUD use Her age History of PID Prior history of ectopic pregnancy Young age as a low risk factor for ectopic pregnancy. Which with the following conditions as a possible complication of severe eclampsia? Placenta previa Placenta abruptio Uterine rupture Erythroblastosis fetalis With elevated blood pressure, the placenta can pull away from the uterine lining, which causes painful, bright red bleeding. A pregnant patient is found to have positive leukocytes and positive nitrates in her urine. She is asymptomatic. What medication should be given? Pyridium Trimethoprim-sulfamethoxazole Nitrofurantoin Doxycycline Patient's urinalysis indicates UTI. Nitrofurantoin would be a good choice to treat patient, it is generally considered safe during pregnancy. Doxycycline is not considered safe during pregnancy. The classic presentation of placenta previa is: Painless vaginal bleeding after the 20th week Painless vaginal bleeding is associated with placenta previa if occurs after the 20th week. Bleeding is likely at this time because the lower uterine segment develops and uterine contractions occur. At this time, the cervix dilates and effaces. Placenta can become detached and bleeding can occur. A young primigravida reports to you that she is starting to feel the baby's movement in her uterus. This is considered to be rechecked the following? Possible sign Positive sign Probable sign Presumptive signs Presumptive signs or sensations that are felt by the mother, but they could also be caused by other conditions. They are some of the earliest symptoms of pregnancy, such as nausea, fatigue, breast tenderness, amenorrhea, and quickening. A woman who is currently pregnant reports that she has had three previous pregnancies: twins delivered at 35 weeks gestation (both living), one at 38 weeks gestation (living), and one miscarriage at 16 weeks gestation. How will this be recorded as her G/TPAL in her electronic medical record? G5P:1213 G4P:2113 G4P:1213 G4P:1113 G/TPAL T = refers to term births, after 37 wks gestation P = refers to premature births, < 37 wks A = refers to abortions (refers to the total # of spontaneous or induced abortions or miscarriages, except ectopic pregnancies, prior to 20 wks. If a fetus is aborted after 20 wks, spontaneously or electively, then it is counted as a premature birth and P will increase but L will not. L = refers to living children Hyperemesis gravidarum is: Morning sickness Persistent, intractable vomiting during pregnancy Always associated with hydatiform mole Indicative of multiple gestations [Hyperemesis gravidarum is a severe form of nausea and vomiting that occurs during pregnancy. A common definition used to define hyperemesis gravidarum is persistent vomiting that produces a weight loss exceeding 5% of prepregnancy body weight]. Hegar's sign is considered a:You Answered Positive sign of pregnancy Presumptive sinus pregnancy Probable sign of pregnancy Problem in pregnancy [Hegar's sign is a softening of the lower portion of the uterus and is considered a probable sign of pregnancy.] A 24-year-old patient presents to clinic with intermittent nausea and vomiting for the past 5 days. She feels fine otherwise. She is afebrile. Her vital signs are within normal limits. What should the nurse practitioner ordered initially? metabolic panel and potassium level Electrolytes and qualitative beta hCG CBC and urine for ketones You Answered Electrolytes and quantitative beta hCG [Pregnancy must be ruled out for this patient. The other concern is her electrolyte status, especially her potassium level as it tends to drop with vomiting] 14-year-old girl with amenorrhea as tested for pregnancy and has a positive result. The patient tells a nurse practitioner that she is seriously considering terminating the pregnancy. She tells the NP that she wants to be referred to a Planned Parenthood clinic. The NP's personal believes in religious beliefs are pro-life. Which with the following is the best action for the NP? NP should excuse herself from the case NP should refer patient to an obstetrician NP should advise patient that a peer who is working with NP can help answer the patient's questions more thoroughly NP should tell the patient about her personal believes and advise her against getting an abortion In general, discussing personal beliefs are considered unprofessional behavior. respecting the patient's right to choose is an example of supporting patient of autonomy A female patient is coming to your clinic complaining of nausea. You go to urine pregnancy test and it is negative, however the patient still thinks she is pregnant. You know that since she has been contemplating pregnancy, she should initiate folic acid... In the third trimester At the diagnosis of pregnancy In the second trimester Now Folic acid has been found to significantly decrease the incidence of neural tube defects. It should be supplemented at a dosage of 0.4 mg daily for all women of childbearing age before becoming pregnant. 38-year-old multigravida who is at 32 weeks gestation calls to family nurse practitioner complaining of vaginal bleeding, abdominal pain, and uterine contractions. There is no watery discharge. She states that her uterus feels hard and is very painful. Which with the following conditions is most likely? Ectopic pregnancy Placenta previa Molar pregnancy Placental abruption Symptoms of placental abruption or bright red vaginal bleeding, boardlike uterus on palpation, and pain. Placenta previa is painless bleeding. Ectopic and molar pregnancy would not progress to 32 weeks. The mother of a 3-day-old newborn reports that her infant nurses every 3-4 hours during the day and sleeps 6 hours at night. What will the provider recommend? Pumping her breasts to maintain her milk supply so that baby will have extra milk later after the initial newborn period. Ensuring that her infant nurses for 15 to 20 minutes each time on one breast only, switching to the other breast at the next feeding Awakening the baby every 3-4 hours to nurse if the baby is not gaining weight Continuing this schedule until the baby is 6 months old, then the interval between feedings can be increased Which with the following is contraindicated in the care of a pregnant woman with placenta previa? Pelvic ultrasound Abdominal ultrasound Intravaginal ultrasound Echocardiogram No type of vaginal exam should be performed and the patient diagnosed with placenta previa. Intravaginal ultrasound and pelvic exams are contraindicated A 30-year-old woman complains of having had no period for the last 12 weeks. She is sexually active and her partner has been using condoms inconsistently. The patient has had history of irregular menstrual cycles and severe dysmenorrhea. The urine pregnancy test result is positive. Which of the following is true statement regarding this pregnancy? The fundus of the uterus should be at the level of the symphysis pubis Quickening starts during this period The cervix should be dilated about half an inch at this time of gestation Hegar's sign is present during this period of pregnancy At 12 weeks of gestation, the fundus of the uterus should be located approximately at the symphysis pubis A woman who is taking oral contraceptive pills (OCPs) to prevent pregnancy calls the provider to report forgetting to take the pills for 4 days. She has not had sexual intercourse during that time. What will the provider recommend? Stop the OCP, use an alternative method, and resume OCPs at the next cycle Use a morning after pill today and resume the OCPs now Take 2 pills daily for 4 days and use an alternative method for 4 days Resume the pills and use a backup method the remainder of the cycle The mother of a 3-day-old newborn reports that her infant nurses every 3-4 hours during the day and sleeps 6 hours at night. What will the provider recommend? Awakening the baby every 3-4 hours to nurse if the baby is not gaining weight Ensuring that her infant nurses for 15 to 20 minutes each time on one breast only, switching to the other breast at the next feeding Pumping her breasts to maintain her milk supply so that baby will have extra milk later after the initial newborn period. Continuing this schedule until the baby is 6 months old, then the interval between feedings can be increased A 3-day-old, full-term female infant who is breastfeeding develops some jaundice. The transcutaneous bilirubin(TcB) reading in the office is in the intermediate risk zone. What will the provider tell the mother? To decrease the frequency of breastfeeding and return to clinic within 5 days for reassessment To supplement feedings with extra water and return to clinic within 5 days for reassessment To use a breast pump to increase her milk supply and increase baby to breast and return to clinic within 5 days for reassessment To switch to formula and return to clinic within 5 days for reassessment A mother who has been breastfeeding her infant for several weeks develops a fever x 24 hours, malaise, breast erythema, and breast tenderness. What will the provider recommend? Cessation of nursing and antibiotics initiated that covers S. aureus Increased frequency of nursing and antibiotics initiated that covers S. aureus Ice packs and increased frequency of nursing Ice packs and cessation of nursing with breast pumping initiated A woman who is currently pregnant reports that she has had three previous pregnancies: twins delivered at 35 weeks gestation (both living), one at 38 weeks gestation (living), and one miscarriage at 16 weeks gestation. How will this be recorded as her G/TPAL in her electronic medical record? G4P:2113 G4P:1113 G5P:1213 G4P:1213 G/TPAL T = refers to term births, after 37 wks gestation P = refers to premature births A = refers to abortions (refers to the total # of spontaneous or induced abortions or miscarriages, except ectopic pregnancies, prior to 20 wks. If a fetus is aborted after 20 wks, spontaneously or electively, then it is counted as a premature birth and P will increase but L will not. L = refers to living children A 44-year-old female who is undergoing treatment for infertility complains of not having had a menstrual period for a few months. The night before, she started spotting and is now having cramp-like pains in her pelvic area. Her blood pressure is 160/80, pulse 110, she is afebrile. Her labs revealed mild anemia, mild leukocytosis. On exam, the uterine fundus is noted to be above the symphysis pubis. The cervical os is dilated to 3 cm. Which of the following is most likely? Acute pelvic inflammatory disease Inevitable abortion Incomplete abortion Threatened abortion Inevitable abortion is defined and vaginal bleeding with pain, cervical dilation and/or cervical effacement. threatened abortion is defined as vaginal bleeding with absent or minimal pain and closed cervix. Incomplete abortion involves moderate or diffuse bleeding with the passage of tissue and painful uterine cramping or contractions. A female patient is coming to your clinic complaining of nausea. You go to urine pregnancy test and it is negative, however the patient still thinks she is pregnant. You know that since she has been contemplating pregnancy, she should initiate folic acid... Now In the third trimester In the second trimester At the diagnosis of pregnancy Folic acid has been found to significantly decrease the incidence of neural tube defects. It should be supplemented at a dosage of 0.4 mg daily for all women of childbearing age before becoming pregnant. 14-year-old girl with amenorrhea as tested for pregnancy and has a positive result. The patient tells a nurse practitioner that she is seriously considering terminating the pregnancy. She tells the NP that she wants to be referred to a Planned Parenthood clinic. The NP's personal believes in religious beliefs are pro-life. Which with the following is the best action for the NP? NP should excuse herself from the case NP should refer patient to an obstetrician NP should advise patient that a peer who is working with NP can help answer the patient's questions more thoroughly NP should tell the patient about her personal believes and advise her against getting an abortion In general, discussing personal beliefs are considered unprofessional behavior. respecting the patient's right to choose is an example of supporting patient of autonomy A pregnant patient complains of urinary frequency and dysuria. Her urinalysis is positive for nitrates, leukocyte esterase, and bacteria. Will course of action is most appropriate? Order a urine culture only Repeated the urinalysis Order urine culture and begin antibiotics Prescribe an antibiotic only The findings of the urinalysis along with symptoms of dysuria are consistent with UTI. Empiric antibiotic should be initiated and culture ordered. Pregnant female start high risk of developing pyelonephritis if UTI is left untreated. Which with the following conditions as a possible complication of severe eclampsia? Uterine rupture Placenta abruptio Erythroblastosis fetalis Placenta previa With elevated blood pressure, the placenta can pull away from the uterine lining, which causes painful, bright red bleeding. Week 9 – Geriatric (GERI) When prescribing medications to an 80-year-old patient, the provider will Review all patient medications at the annual health maintenance visit. Begin with higher doses and decrease according to the patient’s response. Consult the Beers list to help identify potentially problematic drugs. Ensure that the patient does not take more than five concurrent medications. An 80-year-old woman who lives alone is noted to have a recent weight loss of 5 pounds. She appears somewhat confused, according to her daughter, who is concerned that she is developing dementia. The provider learns that the woman still drives, volunteers at the local hospital, and attends a book club with several friends once a month. What is the initial step in evaluating this patient? Referring the patient to a dietician for nutritional evaluation Referring the patient to a neurologist for evaluation for AD Obtain a CBC, serum electrolytes, BUN, and glucose Ordering a CBC, serum ferritin, and TIBC An 80-year-old woman who lives alone is noted to have a recent weight loss of 5 pounds. She appears somewhat confused, according to her daughter, who is concerned that she is developing dementia. The provider learns that the woman still drives, volunteers at the local hospital, and attends a book club with several friends once a month. What is the initial step in evaluating this patient? Ordering a CBC, serum ferritin, and TIBC Referring the patient to a dietician for nutritional evaluation Obtain a CBC, serum electrolytes, BUN, and glucose Referring the patient to a neurologist for evaluation for AD When prescribing medications to an 80-year-old patient, the provider will Consult the Beers list to help identify potentially problematic drugs. Review all patient medications at the annual health maintenance visit. Begin with higher doses and decrease according to the patient’s response. Ensure that the patient does not take more than five concurrent medications. What medication may be used to treat GERD if a patient has tried over the counter ranitidine without benefit? Prescription strength ranitidine Calcium carbonate Pantoprazole Prescription strength ranitidine Noninfectious epididymitis is common in: Soccer players Truck drivers Marathon runners Men to wear boxer style underwear Noninfectious epididymitis occurs when there is a reflux of urine into the epididymis from the ejaculatory duct and vas deferens. This can occur if males spend a lot of time sitting. Other typical risk factors include vigorous exercise that involve heavy lifting or upper body workout. A 26-year-old female patient has been diagnosed with gonorrhea. However should she be managed? Penicillin G Cefixime and azithromycin Ceftriaxone only Ceftriaxone and azithromycin Usual treatment for gonorrhea/Chlamydia includes ceftriaxone 250 mg IM in conjunction with 1 g azithromycin by mouth. A patient in her first trimester of pregnancy is found to be infected with Chlamydia and gonorrhea. Which statement below is true? She should be screened for other STDs later in pregnancy She should be treated in the second trimester She should be treated now and rescreened later in pregnancy She should be treated now and rescreened if symptoms reappear This patient should certainly be treated for Chlamydia and gonorrhea with azithromycin 1 g x1 and ceftriaxone 250 mg IM ×1. since the percentage of patients who become reinfected with an STD later in pregnancy is high, this patient should be rescreened later in pregnancy regardless of symptoms. Disastrous effects can occur if she is infected and left untreated while pregnant. An 80-year-old woman who lives alone is noted to have a recent weight loss of 5 pounds. She appears somewhat confused, according to her daughter, who is concerned that she is developing dementia. The provider learns that the woman still drives, volunteers at the local hospital, and attends a book club with several friends once a month. What is the initial step in evaluating this patient? Ordering a CBC, serum ferritin, and TIBC Referring the patient to a neurologist for evaluation for AD Referring the patient to a dietician for nutritional evaluation Obtain a CBC, serum electrolytes, BUN, and glucose When prescribing medications to an 80-year-old patient, the provider will Review all patient medications at the annual health maintenance visit. Begin with higher doses and decrease according to the patient’s response. Consult the Beers list to help identify potentially problematic drugs. Ensure that the patient does not take more than five concurrent medications. An older patient presents with left lower quadrant pain. If diverticulitis is suspected, how should the NP proceed? Order a chest and abdominal xray CT scan of abdomen Ultrasound of the abdomen Barium enema A female patient who is 45-year-old states that she is having urinary frequency. She describes episodes of "having to go right now" and not being able to wait. Her urinalysis results are within normal limits. What this part of the differential? Asymptomatic bacteriuria Diabetes Stress incontinence Lupus Patients with diabetes can present with polyuria. In assessment of patient's risk factors should be done with strong consideration even to checking glucose level. Other possible diagnoses include urge incontinence and vaginitis. A urinalysis would show bacteriuria. Syphilis may present as: Discharge Painful lesions Dysuria A rash Secondary syphilis can present as a rash, more commonly on the palms of the hands or soles of the feet. Lesions are usually painless. It usually does not produce significant dysuria or discharge. A female patient is coming to your clinic complaining of nausea. You go to urine pregnancy test and it is negative, however the patient still thinks she is pregnant. You know that since she has been contemplating pregnancy, she should initiate folic acid... In the third trimester At the diagnosis of pregnancy In the second trimester Now Folic acid has been found to significantly decrease the incidence of neural tube defects. It should be supplemented at a dosage of 0.4 mg daily for all women of childbearing age before becoming pregnant. Week 10 – Mental Health Conditions and Psychiatric Disorders CNS stimulants cause increase alertness, excitation, and sometime euphoria. Stimulant drugs include the following: Cocaine, Amphetamine and Ecstasy Ectasy, Cocaine and Alcohol Naloxone, Benzodiazepines and Alcohol Benzodiazepines, Cocaine and Amphetamines A patient is diagnosed with panic disorder and begins taking a selective serotonin reuptake inhibitor medication. Six weeks later, the patient reports little relief from symptoms. What will the provider do next to manage this patient? Discontinue the medication Refer to a mental health provider Change the medication to buspirone Increase the medication dose A patient is coming to your clinic and you suspect opiate withdrawal. You know the most common signs of opiate withdrawal include: Vomiting, vision loss, ulcerations Dysphoric mood, piloerection, insomnia, weakness Hypertension, muscle pain, urinary issues Anxiety, hair loss, muscle spasms A patient with an eating disorder might exhibit evidence of: Sexual abuse Anxiety disorders Thyroid disease Sleep disorders In patients with eating disorders, it is common to identify affective disorders, anxiety disorders, or substance abuse issues. Obsessive-compulsive disorder is also commonly observed. There is no evidence of patients with eating disorders exhibit a higher incidence of sleep disorders, or had been sexually abused. Thyroid disease should always be assessed in patients with eating disorders, but this does not represent the reason for weight loss when eating disorder is present. A patient is coming in to see you at your urgent care with vague symptoms. You note that she has been at your urgent care multiple times over the last several weeks complaining of vague symptoms. You note that she seems to respond poorly to medical treatment that has been given to her. What should be considered when obtaining a history from her? Physical abuse or depression Anemia or depression Hepatitis or HIV Depression or HIV A college student is brought to the emergency department by a roommate who is concerned about symptoms of extreme restlessness, nausea, and vomiting. The provider notes elevations of the pulse and blood pressure and pupillary dilation, along with hyperactive bowel sounds. The provider suspects withdrawal from which substance? LSD Opioids Cocaine Alcohol Which medications are useful in treating both obsessive-compulsive disorder and PTSD? (Select all that apply.) Serotonin-norepinephrine reuptake inhibitors Buspirone Selective serotonin reuptake inhibitors Tricyclic antidepressants Benzodiazepines A patient with an eating disorder might exhibit evidence of: Sexual abuse Anxiety disorders Thyroid disease Sleep disorders A patient is coming in to see you at your urgent care with vague symptoms. You note that she has been at your urgent care multiple times over the last several weeks complaining of vague symptoms. You note that she seems to respond poorly to medical treatment that has been given to her. What should be considered when obtaining a history from her? Depression or HIV Hepatitis or HIV Anemia or depression Physical abuse or depression Which class of antidepressants is first line to treat obsessive compulsive disorder? TCA SNRI MAOI SSRI A 86-year-old male reports feeling anhedonia for the last month. What should be part of the nurse practitioner's initial assessment? Suicidal ideation Libido Mania Depression Anhedonia is the loss of pleasure interest in things that have always brought pleasure or interest. It is a red flag for depression. Screening for depression is necessary, however, suicidal ideation should be the priority assessment. A 29-year-old postpartum female reports that she is having difficulty with concentration, sleep, and has feelings of guilt. She states that she feels sad most of the time. The symptoms have been present since the birth of her baby about one month ago. She can be diagnosed with: Minor depressive disorder Dysthymia Hypothyroidism Postpartum depression Post partum depression is diagnosed with depression begins within the first month after delivery. A patient is coming to the clinic complaining of feeling depressed. You know that a typical symptom of depression is: Keeping late-night hours reading Early morning wakening Snoring Difficulty falling asleep Sleeping difficulty is a common complaint among patients with depression. Patients with difficulty falling asleep or often anxious. Frequent wakening and early morning wakening are complaints by many patients with depression. You are seeing a patient and considering serotonin syndrome as a potential differential diagnosis. The patient is taking an SSRI. Which of the other medications the patient is taking may cause serotonin syndrome? Dextromethorphan Necessary rind dextromethorphan can cause serotonin syndrome. This is a potentially life-threatening condition. 4 days ago, 79-year-old female lost her husband of 55 years. She presents today with her daughter because she believes that she is "going crazy." She reports that she often hears her husbands voice though she realizes that he has died. She has not slept well since his death and hasn't been eating much. She has taken her usual medications for hypertension, osteoporosis, osteoarthritis, and hypothyroidism. She has no history of psychiatric illness. Should the NP manage this? Tell her that this is a normal response and will resolve This imagined hearing or seeing of the deceased person is referred to as "searching behavior" and is not indicative of a psychiatric illness. It is a common response after the death of a lot of the 1, especially after 55 years of marriage. This patient and her daughter should be educated regarding the stages of grief and a variable length of each of those stages. A patientwith acute anxiety will experience the fastest relief of symptoms when he is treated with? A benzodiazepine The most rapid relief of anxiety symptoms will occur with the benzodiazepine. You are seeing a 22 year old patient in your primary care clinic. The patient complains of abdominal. You do a urine pregnancy test and it is positive. You referred the patient to an obstetrician who will see the patient in 4 weeks. You note that the patient also takes sertraline for depression. How should the sertraline be managed? Let the obstetricians and patient make a decision about continuing sertraline Sertraline is one of the best studied selective serotonin reuptake inhibitors in pregnancy and lactation. While the health care provider would rather this patient not taking medication while she is pregnant, consideration must be given the severity of her depression and her response to treatment. The risk of exposure to medication must always be weighed against the risk of not treating this patient. All psychotropic medications cross the placenta and so developing fetuses are exposed to these medications. Which findings suggested a patient may be abusing alcohol? Rhinophyma, hypotension, peripheral neuropathy Telangiectasias, flat affect, thyroid dysfunction Correct! Macrocytosis, tremulousness, hypertension Hepatosplenomegaly, murmur, osteoarthritis [Findings that should trigger the examiner to suspect alcohol abuse patients are tremors, hypertension, right of femur, peripheral neuropathy, telangiectasias, and hepatosplenomegaly. Macrocytosis is common in alcoholics because there is a high rate of B12 deficiency and folate deficiency]. A 86-year-old male reports feeling anhedonia for the last month. What should be part of the nurse practitioner's initial assessment? Mania Libido You Answered Depression Suicidal ideation [Anhedonia is the loss of pleasure interest in things that have always brought pleasure or interest. It is a red flag for depression. Screening for depression is necessary, however, suicidal ideation should be the priority assessment.] A patient reports symptoms of restlessness, fatigue, and difficulty concentrating. The provider determines that these symptoms occur in relation to many events and concerns. What other things will the provider question this patient about? Headaches and bowel habits Ability to manage social situations You Answered Occupational performance Body image and eating habits You are seeing a new 10 year old patient in your primary care clinic. You believe the patient has symptoms congruent with bipolar disorder. However, you also know that the usual age of onset of symptoms with bipolar disorder is: Third decade Childhood Correct! Between 15 and 30 years old Adolescence [The usual age of onset of bipolar disease symptoms as between 15-30 years old. Onset of symptoms almost never occurs in patients older than 65 or younger than 15 years old.] A patient is coming in to your urgent care and notes that he has been on edge lately and needs help. You read in his chart that he is bipolar. The patient states that he recently purchased at $10,000 grand piano. He does not play the piano. This behavior is typical during: Depression Psychosis Hypomania Mania Which class of antidepressants is first line to treat obsessive compulsive disorder? SNRI MAOI SSRI TCA A 29-year-old postpartum female reports that she is having difficulty with concentration, sleep, and has feelings of guilt. She states that she feels sad most of the time. The symptoms have been present since the birth of her baby about one month ago. She can be diagnosed with: Postpartum depression A patient is diagnosed with panic disorder and begins taking a selective serotonin reuptake inhibitor medication. Six weeks later, the patient reports little relief from symptoms. What will the provider do next to manage this patient? Refer to mental health provider A patient is coming in to see you at your urgent care with vague symptoms. You note that she has been at your urgent care multiple times over the last several weeks complaining of vague symptoms. You note that she seems to respond poorly to medical treatment that has been given to her. What should be considered when obtaining a history from her? Physical abuse or depression A 38-year-old patient diagnosed with bipolar disease has taken lithium for many months. His mood has stabilized. He was told to report frequent urination while taking lithium. What might be the underlying cause of his frequent urination? Diabetes insipidus The most common side effect of lithium therapy is nephrogenic diabetes insipidus. Polyuria and polydipsia secondary to nephrogenic diabetes insipidus occur in about 20% of patients who take lithium. A patient is taking a generic version of an SSRI. She reports intermittent nausea and mild headaches daily since she started taking this medication 5 days ago. How should NP respond? These are typical complaints of patient's with take SSRI A college student is brought to the emergency department by a roommate who is concerned about symptoms of extreme restlessness, nausea, and vomiting. The provider notes elevations of the pulse and blood pressure and pupillary dilation, along with hyperactive bowel sounds. The provider suspects withdrawal from which substance? Cocaine LSD Alcohol Opioids Which medications are useful in treating both obsessive-compulsive disorder and PTSD? (Select all that apply.) Tricyclic antidepressants Benzodiazepines Buspirone Selective serotonin reuptake inhibitors Serotonin-norepinephrine reuptake inhibitors A patient with acute anxiety will experience the fastest relief of symptoms when he is treated with? A beta blocker An SSRI A benzodiazepine A TCA The most rapid relief of anxiety symptoms will occur with the benzodiazepine. A 29-year-old postpartum female reports that she is having difficulty with concentration, sleep, and has feelings of guilt. She states that she feels sad most of the time. The symptoms have been present since the birth of her baby about one month ago. She can be diagnosed with: Hypothyroidism Minor depressive disorder Postpartum depression Dysthymia Post partum depression is diagnosed with depression begins within the first month after delivery. Which class of antidepressants is first line to treat obsessive compulsive disorder? SSRI SNRI MAOI TCA CNS stimulants cause increase alertness, excitation, and sometime euphoria. Stimulant drugs include the following: Naloxone, Benzodiazepines and Alcohol Cocaine, Amphetamine and Ecstasy Benzodiazepines, Cocaine and Amphetamines Ectasy, Cocaine and Alcohol A patient is diagnosed with panic disorder and begins taking a selective serotonin reuptake inhibitor medication. Six weeks later, the patient reports little relief from symptoms. What will the provider do next to manage this patient? Change the medication to buspirone Increase the medication dose Discontinue the medication Refer to a mental health provider Which class of antidepressants is first line to treat obsessive compulsive disorder? MAOI TCA SNRI SSRI A patient is taking a generic version of an SSRI. She reports intermittent nausea and mild headaches daily since she started taking this medication 5 days ago. How should NP respond? Correct Answer These are typical complaints of patient's with take SSRI The symptoms are common in patients with depression This sounds like a viral syndrome. Continue the SSRI Brand-name medication may help the side effects resolve Typical symptoms of SSRIs include mild headache, nausea, insomnia, restlessness, agitation. The emergence of the symptoms is typically dose related and will resolve within 2 weeks. 4 days ago, 79-year-old female lost her husband of 55 years. She presents today with her daughter because she believes that she is "going crazy." She reports that she often hears her husbands voice though she realizes that he has died. She has not slept well since his death and hasn't been eating much. She has taken her usual medications for hypertension, osteoporosis, osteoarthritis, and hypothyroidism. She has no history of psychiatric illness. Should the NP manage this? Prescribe a benzodiazepine for relief of anxiety Encourage her daughter to consider assisted living placement Prescribe an antidepressant and follow-up in one to 2 weeks Tell her that this is a normal response and will resolve This imagined hearing or seeing of the deceased person is referred to as "searching behavior" and is not indicative of a psychiatric illness. It is a common response after the death of a lot of the 1, especially after 55 years of marriage. This patient and her daughter should be educated regarding the stages of grief and a variable length of each of those stages. Which medications are useful in treating both obsessive-compulsive disorder and PTSD? (Select all that apply.) Buspirone Selective serotonin reuptake inhibitors Tricyclic antidepressants Serotonin-norepinephrine reuptake inhibitors Benzodiazepines A patient reports symptoms of restlessness, fatigue, and difficulty concentrating. The provider determines that these symptoms occur in relation to many events and concerns. What other things will the provider question this patient about? Occupational performance Body image and eating habits Ability to manage social situations Headaches and bowel habits A patient is diagnosed with panic disorder and begins taking a selective serotonin reuptake inhibitor medication. Six weeks later, the patient reports little relief from symptoms. What will the provider do next to manage this patient? Increase the medication dose Change the medication to buspirone Refer to a mental health provider Discontinue the medication CNS stimulants cause increase alertness, excitation, and sometime euphoria. Stimulant drugs include the following: Naloxone, Benzodiazepines and Alcohol Ectasy, Cocaine and Alcohol Benzodiazepines, Cocaine and Amphetamines Cocaine, Amphetamine and Ecstasy Question 5 10 / 10 pts A college student is brought to the emergency department by a roommate who is concerned about symptoms of extreme restlessness, nausea, and vomiting. The provider notes elevations of the pulse and blood pressure and pupillary dilation, along with hyperactive bowel sounds. The provider suspects withdrawal from which substance? Cocaine Alcohol Correct! Opioids LSD A patient is coming in to see you at your urgent care with vague symptoms. You note that she has been at your urgent care multiple times over the last several weeks complaining of vague symptoms. You note that she seems to respond poorly to medical treatment that has been given to her. What should be considered when obtaining a history from her? Anemia or depression Depression or HIV Physical abuse or depression Hepatitis or HIV Patient's who have been victims of violence or more likely to utilize health care and to have a poor response to treatment. If the patient is suspected to have been a victim of violence, they should also be screened for anxiety, depression, and PTSD. A patient is coming in to your urgent care and notes that he has been on edge lately and needs help. You read in his chart that he is bipolar. The patient states that he recently purchased at $10,000 grand piano. He does not play the piano. This behavior is typical during: Depression Mania Psychosis Hypomania Which findings suggested a patient may be abusing alcohol? Hepatosplenomegaly, murmur, osteoarthritis Rhinophyma, hypotension, peripheral neuropathy Macrocytosis, tremulousness, hypertension Telangiectasias, flat affect, thyroid dysfunction Findings that should trigger the examiner to suspect alcohol abuse patients are tremors, hypertension, right of femur, peripheral neuropathy, telangiectasias, and hepatosplenomegaly. Macrocytosis is common in alcoholics because there is a high rate of B12 deficiency and folate deficiency. A patientwith acute anxiety will experience the fastest relief of symptoms when he is treated with? A beta blocker A benzodiazepine A TCA An SSRI The most rapid relief of anxiety symptoms will occur with the benzodiazepine. You are seeing a 22 year old patient in your primary care clinic. The patient complains of abdominal. You do a urine pregnancy test and it is positive. You referred the patient to an obstetrician who will see the patient in 4 weeks. You note that the patient also takes sertraline for depression. How should the sertraline be managed? Wean it off as she waits to see the obstetrician Let the patient make a decision about discontinuing the medication Let the obstetricians and patient make a decision about continuing sertraline Discontinue immediately Sertraline is one of the best studied selective serotonin reuptake inhibitors in pregnancy and lactation. While the health care provider would rather this patient not taking medication while she is pregnant, consideration must be given the severity of her depression and her response to treatment. The risk of exposure to medication must always be weighed against the risk of not treating this patient. All psychotropic medications cross the placenta and so developing fetuses are exposed to these medications. The mother of a 7 year old boy tells the NP that his teacher has complained to her of her son's frequent episodes of daydreaming. The mother reports that sometimes when her son is at home, he seems not to hear her, seeming to "blank out" for a short period of time. Which of the following is most likely? An absence siezure (petit mal seizure) A 70 year old male is diagnosed with vertigo. Which choice below indicates that the vertigo is more likely to be of central etiology? Persistent symptoms You are examining a patient who has just been diagnosed with Bell's palsy. Bells's palsy is characterized by all of the following except: Inability to swallow Which of the following is NOT a characteristic of delirium? Patient is coherent What characteristic is true of tension headaches, but not of cluster headaches? Tension headaches are always bilateral Which tests are indicated as part of the initial evaluation for women of childbearing age who report syncope? (Select all that apply.) Complete blood count, Fasting blood sugar, 12-lead electrocardiogram A patient who sustained a head injury has a Glasgow Coma score of 14. The patient’s spouse reported that the patient lost consciousness for approximately 7 minutes after falling down the stairs. A head CT does not reveal brain lesions. Which treatment is indicated? Admission to the hospital with a neurosurgical evaluation An elderly patient reports sensations of being off balance when walking, but does not experience dizziness. The provider will refer this patient to which specialist for further evaluation? Neurologist Which medications may be useful in treating tension-type headache? (Select all that apply.) NSAIDS, Muscle relaxants A patient who has a seizure disorder and who takes levetiracetam is brought to an emergency department with a seizure which has persisted for15 minutes and which immediately followed another 15 minute seizure. What is the priority action for this patient? Administer lorazepam and monitor cardiorespiratory status Week 11 – Neurology Which symptoms may occur with Bell’s palsy? (Select all that apply.) Alteration in taste Drooling Inability to open the eye Correct! Decreased hearing Tinnitus A patient is brought to the emergency department after being hit in the head with a baseball. The patient is awake and talking, but is confused and disoriented and does not obey simple commands. The patient is able to point to the area of pain and opens eyes only when commanded to do so. Bystanders report a period of unconsciousness lasting almost 5 minutes. Which severity of traumatic brain injury is likely? Severe Moderate Normal You Answered Mild You are examining a patient who has just been diagnosed with Bell's palsy. Bells's palsy is characterized by all of the following except: Inability to close the eye on the affected side Correct! Inability to swallow Drooling Drooping of the corner of the mouth on the affected side A 70 year old male is diagnosed with vertigo. Which choice below indicates that the vertigo is more likely to be of central etiology? Brief duration Correct! Persistent symptoms Nystagmus present Nausea and vomiting What characteristic is true of tension headaches, but not of cluster headaches? Tension headaches cause photosensitivity Cluster headaches are always bilateral Cluster headaches always cause nausea Correct! Tension headaches are always bilateral [Cluster headaches are always unilateral] A patient exhibits visual field defect, ataxia, and dysarthria and complains of a mild headache. A family member reports that the symptoms began several hours prior. An examination reveals normal range of motion of the neck. What type of cerebrovascular event is most likely? Ischemic stroke Hypertensive intracerebral hemorrhage Transient ischemic attack Hemorrhagic stroke A 72 year old patient with history of polymyalgia rheumatica complains of new onset unilateral headache and visual changes. Her sedimentation rate is elevated. Her neurological exam is otherwise normal. What is the most likely reason for her symptoms? Meningitis TIA CVA Temporal arteritis A 50 year old man complains of marked scalp tenderness accompanied by a bad headache at his left temple. He reports a sudden loss of vision in the left eye for the past several hours. The neurological exam is normal except for the loss of vision in the left eye. Which of the following conditions is most likely? Cluster headache Migraine headache without aura Migraine headache with aura Giant cell arteritis A patient reports recurrent headaches occurring 1 or 2 times per month that generally occur with weather changes or when sleep patterns are disrupted and describes them as severe, with throbbing on one side of the head and sometimes accompanied by nausea. What is the recommended treatment for this type of headache? Gabapentin Rizatriptan Topiramate Propranolol A patient who is 82 years old is brought into the clinic. His wife states that he was working in his garden today and became disoriented and had slurred speech. She helped him back into the house, gave him cool fluids, and within 15 minutes his symptoms resolved. He appears in his usual state of health when he is examined. He states that although he was scared by the event, he feels fine now. How should the NP proceed? Prescribe an aspirin daily Send him to the emergency department Order an EKG Re-examine him tomorrow [This patient likely suffered a TIA. Patients have an increased risk of CVA within 48 hours of TIA. He needs urgent diagnostic testing (imaging and labs) and evaluation] Which finding in a patient with migraine headache symptoms would compel the examiner to order an imaging study? Rapidly increasing intensity of headache First occurrence with typical migraine symptoms Fully reversible speech disturbance Nausea and photophobia A healthy 20-year-old patient reports having had 1 or 2 episodes of syncope without loss of consciousness. Which is the most likely type of syncope in this patient? Cardiac Orthostatic hypotensive You Answered Neurogenic Reflex syncope The mother of a 7 year old boy tells the NP that his teacher has complained to her of her son's frequent episodes of daydreaming. The mother reports that sometimes when her son is at home, he seems not to hear her, seeming to "blank out" for a short period of time. Which of the following is most likely? A partial siezure Correct! An absence siezure (petit mal seizure) A grand mal seizure An atonic seizure (drop attack) Which of the following is a diagnostic criteria for migraine headache without aura?orrect Pain lasts 4-72 hrs Pain is episodic during the headache Photophobia must be present There are underlying neurological abnormalities Which of the following is NOT a characteristic of delirium? Sudden onset Brief duration Correct! Patient is coherent Worse in the evening An elderly patient reports experiencing syncope each morning when getting out of bed. Which assessment will the health care provider perform first to evaluate the patient’s sumptoms? Cardiac enzyme levels Fasting blood glucose Orthostatic blood pressures Electroencephalogram What are initial approaches when managing delirium in a hospitalized patient who is agitated and confused? (Select all that apply.) Attend to hydration and toileting needs Apply physical restraints Discontinue any non-essential medications Decrease stimulation Administer medications for sleep What is recommended to prevent ophthalmic complications in patients with Bell’s palsy? Lubricating eye drops Acupuncture Patching of the eye Sunglasses A 50 year old man complains of marked scalp tenderness accompanied by a bad headache at his left temple. He reports a sudden loss of vision in the left eye for the past several hours. The neurological exam is normal except for the loss of vision in the left eye. Which of the following diagnostic tests would be most helpful in the diagnosis of this illness? Sedimentation rate Cranial nerve exam CBC with differential CT scan of the brain An elderly patient reports sensations of being off balance when walking, but does not experience dizziness. The provider will refer this patient to which specialist for further evaluation? Cardiologist Neurologist Audiologist Otolaryngologist When assessing a patient suspected of having vertigo, which description provided by the patient is most consistent with the diagnosis? A sensation of lightheadedness when changing positions from reclining to standing Sensation of spinning or rotating A sensation of "passing out" A sensation of imbalance while walking Which medications may be useful in treating tension-type headache? (Select all that apply.) Anticonvulsants Lithium Correct! NSAIDs Oxygen Correct! Muscle relaxants An 80-year-old patient becomes apathetic, with decreased alertness and a slowing of speech several days after hip replacement surgery alternating with long periods of lucidity. What is the most likely cause of these symptoms? Stroke Correct! Delirium Anesthesia effects Pain medications A patient with a seizure disorder has seizures which begin with eye twitching and occasionally visual hallucinations. Which site in the brain is the seizure focus?Correct! Occipital Frontal Parietal Temporal A 35-year-old patient reports suddenly experiencing an asymmetric smile along with drooping and tearing in one eye. The patient has a history of a recent viral illness, but is otherwise healthy. During the exam, the provider notes that there is unilateral full face paralysis on the right side. What is the initial intervention for this patient? Prescribe oral prednisolone Refer the patient to a neurologist Recommend wearing an eye patch Perform confirmatory diagnostic tests An 80-year-old patient becomes apathetic, with decreased alertness and a slowing of speech several days after hip replacement surgery alternating with long periods of lucidity. What is the most likely cause of these symptoms? Delirium Stroke Pain medications Anesthesia effects A 50 year old man complains of marked scalp tenderness accompanied by a bad headache at his left temple. He reports a sudden loss of vision in the left eye for the past several hours. The neurological exam is normal except for the loss of vision in the left eye. Which of the following diagnostic tests would be most helpful in the diagnosis of this illness? CBC with differntial Sedimentation rate Cranial nerve exam CT scan of the brain Which symptom below is true of cluster headache but not migraine headaches? The most common characteristic is family history The length of the headache duration is about 30-90 minutes Sunlight is a common trigger It is more common in females A 50 year old man complains of marked scalp tenderness accompanied by a bad headache at his left temple. He reports a sudden loss of vision in the left eye for the past several hours. The neurological exam is normal except for the loss of vision in the left eye. Which of the following conditions is most likely? Giant cell arteritis Migraine headache without aura Migraine headache with aura Cluster headache A healthy 20-year-old patient reports having had 1 or 2 episodes of syncope without loss of consciousness. Which is the most likely type of syncope in this patient? Neurogenic Cardiac Reflex syncope Orthostatic hypotensive A patient is in the emergency department after sustaining a blow to the head in a motor vehicle accident. The patient’s Glasgow Coma score is 14 and the patient is drowsy. The patient has a small amount of blood in one external auditory canal. Which is a priority in diagnosing the extent of injury in this patient? Non-enhanced computed tomography of the head Close monitoring of pulse, respiration, and oxygenation Magnetic resonance imaging of the head Continued assessment of neurological status A 50 year old man complains of marked scalp tenderness accompanied by a bad headache at his left temple. He reports a sudden loss of vision in the left eye for the past several hours. The neurological exam is normal except for the loss of vision in the left eye. Which of the following diagnostic tests would be most helpful in the diagnosis of this illness? CT scan of the brain Sedimentation rate CBC with differntial Cranial nerve exam The mother of a 7 year old boy tells the NP that his teacher has complained to her of her son's frequent episodes of daydreaming. The mother reports that sometimes when her son is at home, he seems not to hear her, seeming to "blank out" for a short period of time. Which of the following is most likely? A grand mal seizure An atonic seizure (drop attack) A partial siezure An absence siezure (petit mal seizure) A 72 year old patient with history of polymyalgia rheumatica complains of new onset unilateral headache and visual changes. Her sedimentation rate is elevated. Her neurological exam is otherwise normal. What is the most likely reason for her symptoms? CVA Meningitis TIA Temporal arteritis A patient who sustained a head injury has a Glasgow Coma score of 14. The patient’s spouse reported that the patient lost consciousness for approximately 7 minutes after falling down the stairs. A head CT does not reveal brain lesions. Which treatment is indicated? Discharge to home with close observation by the patient’s spouse for 24 hours Correct Answer Admission to the hospital with a neurosurgical evaluation Discharge to home with close observation by the patient’s spouse for 24 hours Dismissal to home with a referral for follow up with a neurologist A patient has recurrent cluster headaches and asks about abortive therapy. Which therapy is effective for a majority of patients with cluster headaches?You Answered Verapamil Lithium Oxygen NSAIDs A patient reports a recurrent sensation of spinning associated with nausea and vomiting. Which test will the provider order to confirm a diagnosis for this patient? Neuroimaging with computerized tomography Electroencephalogram Holter monitoring and electrocardiogram Correct! The Hallpike-Dix positioning maneuver A previously healthy 30-year-old patient is brought to the emergency department with signs of stroke. Diagnostic testing determines an ongoing ischemic cause. The patient’s spouse reports that symptoms began approximately 2 hours prior to transport. What is the recommended treatment? Observation for complications prior to initiating tPA Administration of low-molecular-weight heparin Correct! Tissue plasminogen activator (tPA) administration Neurosurgical consultation for possible surgery Which of the following is NOT true regarding cervical whiplash injury? Occipital pain and headache can occur It is identifiable on MRI or CT, but not on Xray It may be accompanied by severe pain, spasm This occurs after a traumatic event [Cervical whiplash is not seen on CT, MRI, or xray. ] A 40 year old man complains to the NP of severe stabbing pains behind his left eye for the past 2 days. They are accompanied by some nasal congestion and rhinorrhea, which is clear in color. The patient denies pharyngitis and fever. Which of the following conditions is most likely? Migraine headache with aura Cranial neuralgia! Cluster headache Tic douloureux A 52 year old patient presents with an acute dropping upper right eyelid. She stated that the right side of her face feels numb. Stroke has been ruled out. Based on the most likely etiology, how should she be manged? Steroids plus an antiviral agent Antiviral agent only Steroids only Immidiate referral to the emergency department When assessing a patient suspected of having vertigo, which description provided by the patient is most consistent with the diagnosis? A sensation of "passing out" A sensation of lightheadedness when changing positions from reclining to standing A sensation of imbalance while walking Sensation of spinning or rotating An elderly patient is brought to the emergency department after being found on the floor after a fall. The patient has unilateral sagging of the face, marked slurring of the speech, and paralysis on one side of the body. The patient’s blood pressure is 220/190 mm Hg. What is the likely treatment for this patient? Neurosurgical consultation Which tests are indicated as part of the initial evaluation for women of childbearing age who report syncope? (Select all that apply.) Cardiac enzyme levels 12-lead electrocardiogram Fasting blood sugars Complete blood count Electroencephalography A previously lucid patient with early-stage Alzheimer’s disease is hospitalized after a surgical procedure and exhibits distractibility and perceptual disturbances that occur only in the late afternoon. The patient has difficulty sleeping at night and instead sleeps much of the morning. What is the likely cause of these symptoms? Sundowner syndrome Hyperactive delirium Hypoactive delirium Worsening dementia A patient is in the emergency department after sustaining a blow to the head in a motor vehicle accident. The patient’s Glasgow Coma score is 14 and the patient is drowsy. The patient has a small amount of blood in one external auditory canal. Which is a priority in diagnosing the extent of injury in this patient? Non-enhanced computed tomography of the head Close monitoring of pulse, respiration, and oxygenation Magnetic resonance imaging of the head Continued assessment of neurological status Which medications may be useful in treating tension-type headache? (Select all that apply.) Lithium NSAIDs Anticonvulsants Oxygen Muscle relaxants What is recommended to prevent ophthalmic complications in patients with Bell’s palsy? Patching of the eye Sunglasses Acupuncture Lubricating eye drops A patient has recurrent cluster headaches and asks about abortive therapy. Which therapy is effective for a majority of patients with cluster headaches? Lithium Verapamil Oxygen NSAIDs A previously healthy 30-year-old patient is brought to the emergency department with signs of stroke. Diagnostic testing determines an ongoing ischemic cause. The patient’s spouse reports that symptoms began approximately 2 hours prior to transport. What is the recommended treatment? Administration of low-molecular-weight heparin Observation for complications prior to initiating tPA Neurosurgical consultation for possible surgery Tissue plasminogen activator (tPA) administration Which of the following is NOT a characteristic of delirium? Worse in the evening Patient is coherent Brief duration Sudden onset A patient who is 82 years old is brought into the clinic. His wife states that he was working in his garden today and became disoriented and had slurred speech. She helped him back into the house, gave him cool fluids, and within 15 minutes his symptoms resolved. He appears in his usual state of health when he is examined. He states that although he was scared by the event, he feels fine now. How should the NP proceed? Order an EKG Re-examine him tomorrow Send him to the emergency department Prescribe an aspirin daily This patient likely suffered a TIA. Patients have an increased risk of CVA within 48 hours of TIA. He needs urgent diagnostic testing (imaging and labs) and evaluation A 70 year old male is diagnosed with vertigo. Which choice below indicates that the vertigo is more likely to be of central etiology? Nystagmus present Brief duration Persistent symptoms Nausea and vomiting Which symptom below is true of cluster headache but not migraine headaches? Sunlight is a common trigger The most common characteristic is family history It is more common in females Correct! The length of the headache duration is about 30-90 minutes What characteristic is true of tension headaches, but not of cluster headaches? Correct! Tension headaches are always bilateral Tension headaches cause photosensitivity Cluster headaches always cause nausea A patient who has a seizure disorder and who takes levetiracetam is brought to an emergency department with a seizure which has persisted for15 minutes and which immediately followed another 15 minute seizure. What is the priority action for this patient? Administer lorazepam and monitor cardiorespiratory status Administer a dose of levetiracetam now and repeat in 10 minutes Administer phenytoin and phenobarbital along with oxygen Admit the patient to the hospital for a diagnostic work up Which drug is used to treat patients with focal epilepsy and complex partial seizures? Lamotrigine Topiramate Ethosuximide Carbamazepine An elderly patient reports experiencing syncope each morning when getting out of bed. Which assessment will the health care provider perform first to evaluate this patient’s symptoms? Electroencephalogram Orthostatic blood pressures Fasting blood glucose Cardiac enzyme levels Week 12 – Dermatology When collecting a specimen to determine a diagnosis of tinea corporis, the provider will scrape which portion of the lesion? The active, leading border The papular lesions The area of central clearing The erythematous plaque A patient has a unilateral vesicular eruption which is described as burning and stabbing in intensity. To differentiate between herpes simplex and herpes zoster, which test will the provider order? Polymerase chain reaction analysis Serum immunoglobulins Tzanck test Viral culture A patient has been diagnosed with scabies. What is the medication of choice to treat this? Coal tar Mupirocin Ketoconazole Permethrin A microscopic examination of the sample taken from a skin lesion indicates hyphae. What type of infection might this indicate? Fungal Bacterial Parasitic Viral Under microscopic exam, hyyphae are long, thin and branching and indicate dermatophyte infections. A 15-month-old who is eating and behaving normally is found to have a high fever. After a few days, the fever resolves and the child breaks out in a maculopapular rash. This is a description of which with the following conditions: Erythema infectiosum Fifth disease Scarlet fever Roseola infantum Roseola is caused by herpes virus 6. Signs and symptoms including high fever for a few days with a maculopapular rash occurring after the fever breaks. A provider is considering an oral contraceptive medication to treat acne in an adolescent female. Which is an important consideration when prescribing this drug? A progesterone-only contraceptive is most beneficial for treating acne. Yaz, Ortho Tri-Cyclen, and Estrostep are approved for acne treatment. Oral contraceptives are effective because of their androgen enhancing effects. Combined oral contraceptives are effective for non-inflammatory acne only. A previously healthy patient has an area of inflammation on one leg which has well-demarcated borders and the presence of lymphangitic streaking. Based on these symptoms, what is the initial treatment for this infection? Sulfamethoxazole-trimethoprim Doxycycline Clindamycin Amoxicillin-clavulanate A patient has seborrheic dermatitis. Which vehicle would be most appropriate to use in the hairline area to treat this? Powder Foam Cream Ointment Skin hairy areas over the body, foams are specifically used because they spread easily and are transparent. A patient has sustained a human bite on his hand during a fist fight. Which is especially concerning with this type of bite injury? Sepsis from Capnocytophaga canimorsus infection Possible exposure to rabies virus Transmission of human immunodeficiency virus Potential septic arthritis or osteomyelitis The patient presents with small vesicles on the lateral edges of his fingers and intense itching. On close inspection, there are small vesicles on the palmar surface of the hand. What is just called? Varicella zoster Seborrheic dermatitis Herpes zoster Dyshidrotic dermatitis This dermatitis is intensely paretic and involves the palms and soles and lateral aspect of the fingers. Over a couple of weeks, the vesicles desquamate. The vesicles might raise suspicion of a viral infection, but this is not present in this case The mother of 80-year-old reports the presence of a round red rash on the child's left lower leg. It appeared one week after the child returned from visiting his grandparents, who live in Massachusetts. During the skin exam, the maculopapular rash is noted to have areas of central clearing making it resemble around target. Which with the following is best described? Erythema migrans Rocky Mountain spotted fever Meningococcemia Larva migrans Which is the primary symptom causing discomfort in patients with atopic dermatitis? Lichenification Pruritis Dryness Erythema A new patient who is a 40-year-old female postal worker being evaluated for complaints of new onset erythematous rash on both cheeks and the bridge of the nose, accompanied by fatigue. She reports a history of Hashimoto's thyroiditis and is currently being treated with Synthroid 1.25 mg daily. Which are the following conditions is most likely? Atopic dermatitis Rosacea Thyroid disease Lupus erythematosus Classic symptoms of lupus erythematosus or butterfly rash across both cheeks and the bridge of the nose, and fatigue. Risk factors also include being female and 40 years old. Which medications may be used as part of the treatment for a patient with hidradenitis suppurativa? (Select all that apply.) Correct Answer Isotretinoin Infliximab Erythromycin Chemotherapy Prednisone A patient has seborrheic dermatitis. Which vehicle would be most appropriate to use in the hairline area to treat this? Cream Powder Ointment Foam Skin hairy areas over the body, foams are specifically used because they spread easily and are transparent. Which is the primary symptom causing discomfort in patients with atopic dermatitis? Dryness Erythema Lichenification Pruritis The patient exhibits petechiae on both lower legs but has no other complaints. How should the NP proceed? Referr to hematology Order blood cultures Stop aspirin and reassess in 1 week Order a CBC The presence of petechia on the lower legs should prompt NP to consider a problem that is related to low platelet count. A CBC should be checked. Blood cultures are of no value to this patient. If platelets are found to be low, then a referral to hematology can be made. A patient has been diagnosed with scabies. What is the medication of choice to treat this? Coal tar Mupirocin Ketoconazole Permethrin A child has irritant contact dermatitis with lesions on the extremities and face. Which treatment is recommended for this patient? Antihistamines Medium- to high-potency topical corticosteroids Oral corticosteroids Topical calcineurin inhibitors A child has vesiculopustular lesions around the nose and mouth with areas of honey-colored crusts. The provider notes a few similar lesions on the child’s hands and legs. Which treatment is appropriate for this child? Amoxicillin-clavulanate Culture and sensitivity of the lesions Topical antiseptic ointment Sulfamethoxazole-trimethoprim A provider is prescribing a topical dermatologic medication for a patient who has open lesions on a hairy area of the body. Which vehicle type will the provider choose when prescribing this medication? Cream Gel Ointmen Powder A patient has been diagnosed with MRSA. She is allergic to sulfa. Which medication could be used to treat her? TMP/SMX Doxycycline Ceftriaxone Augmentin [Typically, MRSA are treated with TMP/SMX, but patients that are allergic to sulfa should not use this medication. An alternative can be doxycycline.] A 15-year-old male has worked this summer as a lifeguard at a local swimming pool. He complains of itching in the groin area. He is diagnosed with tinea cruris. The nurse practitioner is likely to identify: Well marginated half-moon macules on the inner thigh Swelling of the scrotum Macular lesions on the penis Maceration of the scrotal folds with erythema of the penis [Tinea cruris, also known as jock itch, his common during warm months and in humid areas. It is a fungal infection that affects the scrotum and inner thigh, but never affects the penis and is never evidenced by scrotal swelling.] When recommending an over-the-counter topical medication to treat a dermatologic condition, which instruction to the patient is important to enhance absorption of the drug? Place an occlusive dressing over the medication Use a lotion or cream instead of an ointment preparation Put cool compresses over the affected area after application Apply a thick layer of medication over the affected area What is the initial approach when obtaining a biopsy of a potential malignant melanoma lesion? Excisional biopsy Shave biopsy Wide excision Punch biopsy The patient presents with plaques on the extensor surface of the elbows, knees, and back. The plaques are erythematous and take, silvery scales are present. This is likely: Guttate psoriasis Atopic dermatitis Staph cellulitis Plaque psoriasis This is a classic characterization of plaque psoriasis. A low potency topical hydrocortisone cream would be most appropriate in a patient who has been diagnosed with: Impetigo Cellulitis Atopical dermatitis Low dose steroid creams are almost never potent enough to treat psoriasis. Impetigo is a superficial bacterial infections and a steroid cream would be contraindicated. Cellulitis would require an oral antibiotic. Psoriasis A patient who has been exposed to poison ivy presents with inflammation and a vesicular rash on one arm. The provider recommends a topical steroid, but the next day the patient calls to report similar lesions appearing on the face. What will the provider tell this patient? The patient must have been re-exposed to the irritant. The vesicles may continue to develop for up to 2 weeks. The rash is spreading through self-inoculation. The rash may spread over the next 8 weeks. The patient has been in the sun for the past few weeks and has developed darkened skin and numerous 3-6 mm light colored, flat lesions on his s trunk. What is the likely etiology? Tinea versicolor Tinea versicolor typically visualized during the spring and summer months when the patient has become darkened after sun exposure. The areas that are infected did not tan and so become very noticeable. Tinea corporis Tinea unguium Human papilloma virus When counseling a patient with rosacea about management of this condition, the provider may recommend (Select all that apply.) using topical antibiotics. eliminating spicy foods. avoiding oil-based products. Which types of medications are associated with urticarial type rashes? (Select all that apply.) Erythromycin NSAIDs Penicillins A preschool girl who is home schooled is brought by her mother to a walk-in clinic because of acute onset of fever, runny nose, cough, sore throat, and red eyes with a morbilliform rash. The mother reports that her daughter has never been immunized. The family recently returned from vacation. Which is the following conditions is most likely? Fifth disease Rubella Rubeola Varicella Rubeola, also known as measles, is a very contagious virus and is transmitted via droplets like the common cold. Koplik's spots are present during the prodromal period. The blotchy pink rash is also known as a morbilliform rash. A 74 year old women is diagnosed with shingles. The NP is deciding how to best manage her care. What should be prescribed: A topical steroid only An oral antiviral An oral antiviral plus topical steroid An oral antiviral plus oral steroid And oral antiviral agent such as acyclovir should be prescribed, especially if it can be initiated within 72 hours after the onset of symptoms. The addition of oral corticosteroids to oral antiviral therapy demonstrates only modest benefit. Adverse events to therapy are more commonly reported in patients receiving oral Corticosteroids. there is no evidence that corticosteroid therapy decreases the incidence of duration of neurology or improved quality of life. They should be limited to use in patients with acute neuritis or who have not has benefit from analgesics. A child is brought to a clinic with a sudden onset of rash after taking an antibiotic for 2 days. The provider notes all over wheals with pruritis, which the parent reports seem to come and go. Which action is correct? Admit the child to the hospital for treatment and observation Which type of bite is generally closed by delayed primary closure? (Select all that apply.) Bites to the hand Wounds 8 hours old Deep puncture wounds A patient comes to the clinic after being splashed by boiling water while cooking. The patient has partial thickness burns on both forearms, the neck, and the chin. What will the provider do? Refer the patient to the emergency department When counseling a patient who has dry skin about ways to minimize exacerbations, what will the provider include? (Select all that apply.) Cleanse the skin frequently Use topical corticosteroids regularly Use fragrance-free detergents Eat soups and stews frequently Take tepid-water baths A patient is taking a sulfonamide antibiotic and develops a rash that begins peeling. Which type of rash is suspected? Wheal and flare Stevens-Johnson Erythema multiforme Urticaria An infant has atopic dermatitis and seborrheic dermatitis with lesions on the forehead and along the scalp line. Which is correct when prescribing a corticosteroid medication to treat this condition? Initiate treatment with 0.1% triamcinolone acetonide (Kenalog lotion) Monitor the infant closely for systemic adverse effects during use Prescribe 0.05% fluocinonide (Lidex-E Cream) to apply liberally Place an occlusive dressing over the medication after application A primary care provider is performing a Tzanck test to evaluate possible herpes simplex lesions. To attain accurate results, the provider will blanch the lesions while examining them with a magnifying glass. perform a gram stain of exudate from the lesions. gently scrape the lesions with a scalpel onto a slide. remove the top of the vesicles and obtain fluid from the lesions. A patient suffers chemical burns on both arms after a spill at work. What is the initial action by the providers in the emergency department? Request the Material Safety Data information Contact the poison control center Remove the offending chemical and garments Begin aggressive irrigation of the site Which medication will the provider prescribe as first-line therapy to treat tinea capitis? Oral ketoconazol Topical tolnaftate Oral griseofulvin Topical clotrimazole An older patient experiences a herpes zoster outbreak and asks the provider if she is contagious because she is going to be around her grandchild who is too young to be immunized for varicella. What will the provider tell her? Contagion is possible until all of her lesions are crusted. An antiviral medication will prevent transmission to others. As long as her lesions are covered, there is no risk of transmission. Varicella zoster and herpes zoster are different infections. A child who has atopic dermatitis has recurrent secondary bacterial skin infections. What will the provider recommend to help prevent these infections? Topical antibiotic ointments Bleach baths twice weekly Frequent bathing with soap and water Low-dose oral antibiotics A patient who has had a previous herpes zoster outbreak experiences a second outbreak and asks the provider about treatment to reduce the duration and severity of symptoms. What will the provider recommend? Lidocaine patch Oral corticosteroids Acyclovir Topical corticosteroids A female patient is diagnosed with hidradenitis suppurativa and has multiple areas of swelling, pain, and erythema, along with several abscesses in the right femoral area. When counseling the patient about this disorder, the practitioner will include which information? Antibiotic therapy is effective in clearing up the lesions. It is often progressive with relapses and permanent scarring. The condition is precipitated by depilatories and deodorants. The lesions are infective and the disease may be transmitted to others. Although lesions may be treated with antibiotics, other medications, and drainage, the disease is often progressive, with relapses and permanent scarring. Deodorants and depilatories are not implicated as a cause. The disease is not transmitted to others, although the organisms may cause other infections in other people. Curing a total body skin examination for skin cancer, the provider notes a raised, shiny, slightly pigmented lesion on the patient’s nose. What will the provider do? Refer the patient for possible electrodessication and curettage Reassure the patient that this is a benign lesion Tell the patient this is likely a squamous cell carcinoma Consult with a dermatologist about possible melanoma A patient has acne and the provider notes lesions on half of the face, some nodules, and two scarred areas. Which treatment will be prescribed? Topical benzoyl peroxide and clindamycin Oral clindamycin for 6 to 8 weeks Topical erythromycin Oral isotretinoin The parent of a 10-month-old child with atopic dermatitis asks what can be done to minimize the recurrence of symptoms in the child. What will the provider recommend? Lubricants and emollients When examining a patient’s skin, a practitioner uses dermoscopy in order to (Select all that apply.) determine whether lesion borders are regular or irregular. http://www.samuelmerritt.edu/library differentiate fluid masses from cystic masses in the epidermis. visualize skin fissures, hair follicles, and pores in lesions. accentuate changes in color of pathologic lesions by fluorescence. assess changes in pigmentation throughout various lesions. A patient who has chronically dry skin who has been using emollients and moisturizers reports an uneven diamond pattern and redness on the lower legs and arms. What will the provider recommend? A topical antibiotic ointment Increasing sodium consumption Referral to a dermatologist Using antihistamines at night A patient with a purulent skin and soft tissue infection (SSTI). A history reveals a previous MRSA infection in a family member. The clinician performs an incision and drainage of the lesion and sends a sample to the lab for culture. What is the next step in treating this patient? Prescribe oral clindamycin Wait for culture results before ordering an antibiotic Begin treatment with amoxicillin-clavulanate Apply moist heat until symptoms resolve When evaluating scalp lesions in a patient suspected of having tinea capitis, the provider uses a Wood’s lamp and is unable to elicit fluorescence. What is the significance of this finding? The patient may have tinea capitis. When counseling a patient who has dry skin about ways to minimize exacerbations, what will the provider include? (Select all that apply.) Use fragrance-free detergents Use topical corticosteroids regularly Eat soups and stews frequently Cleanse the skin frequently Take tepid-water baths A patient has sustained a human bite on his hand during a fist fight. Which is especially concerning with this type of bite injury? Sepsis from Capnocytophaga canimorsus infection Potential septic arthritis or osteomyelitis Possible exposure to rabies virus Transmission of human immunodeficiency virus A patient is taking a sulfonamide antibiotic and develops a rash that begins peeling. Which type of rash is suspected? Stevens-Johnson Wheal and flare Urticaria Erythema multiforme A provider is prescribing a topical dermatologic medication for a patient who has open lesions on a hairy area of the body. Which vehicle type will the provider choose when prescribing this medication? Gel Cream Ointment Powder An older patient experiences a herpes zoster outbreak and asks the provider if she is contagious because she is going to be around her grandchild who is too young to be immunized for varicella. What will the provider tell her? An antiviral medication will prevent transmission to others. Contagion is possible until all of her lesions are crusted. As long as her lesions are covered, there is no risk of transmission. Varicella zoster and herpes zoster are different infections. A low potency topical hydrocortisone cream would be most appropriate in a patient who has been diagnosed with: Impetigo Atopical dermatitis Cellulitis Psoriasis Low dose steroid creams are almost never potent enough to treat psoriasis. Impetigo is a superficial bacterial infections and a steroid cream would be contraindicated. Cellulitis would require an oral antibiotic. The patient presents with plaques on the extensor surface of the elbows, knees, and back. The plaques are erythematous and take, silvery scales are present. This is likely: Staph cellulitis Guttate psoriasis Atopic dermatitis Plaque psoriasis This is a classic characterization of plaque psoriasis. A microscopic examination of the sample taken from a skin lesion indicates hyphae. What type of infection might this indicate? Parasitic Fungal Viral Bacterial Under microscopic exam, hyyphae are long, thin and branching and indicate dermatophyte infections. A 15-month-old who is eating and behaving normally is found to have a high fever. After a few days, the fever resolves and the child breaks out in a maculopapular rash. This is a description of which with the following conditions: Fifth disease Scarlet fever Erythema infectiosum Roseola infantum Roseola is caused by herpes virus 6. Signs and symptoms including high fever for a few days with a maculopapular rash occurring after the fever breaks. A 15-year-old male has worked this summer as a lifeguard at a local swimming pool. He complains of itching in the groin area. He is diagnosed with tinea cruris. The nurse practitioner is likely to identify: Macular lesions on the penis Swelling of the scrotum Well marginated half-moon macules on the inner thigh Maceration of the scrotal folds with erythema of the penis Tinea cruris, also known as jock itch, his common during warm months and in humid areas. It is a fungal infection that effects the scrotum and inner thigh, but never affects the penis and is never evidenced by scrotal swelling. Week 13 – Pediatrics An 8-year-old boy with type 1 diabetes is being seen for 3 day history of urinary frequency and nocturia. He denies flank pain and is afebrile. The urinalysis results is negative for blood and nitrates but is positive for large amount of leukocytes and ketones. He has a trace amount of protein. Which is the following is best test to order initially? A urine culture and sensitivity A 4-week-old boy seen in the clinic for a complaint of forceful vomiting that occurs immediately after feeding. The vomitus is composed of infant formula and is not bilious. The infant is bottle-fed with infant formula that was recommended by the pediatrician. The mother reports that the infant seems hungry and sucks on the bottle without any problems. His birth weight is 3.4 kg and his current weight is 3.2 kg. Which with the following clinical findings is an important clue regarding the possible cause of the infant's vomiting? Positive rooting reflex Irritable and crying infant Round olive like mass located in the right upper quadrant of the abdomen Sunken anterior fontanelle and dry lips The stem is asking about the possible cause of the infant's vomiting, it is not asking about the symptoms. Projectile of forceful vomiting after feeding is a classic symptom of infantile hyperrtrophic pyloric stenosis. The other signs and symptoms are dehydration, irritabillity, crying. Positive rooting reflex is normal finding in a 4-week-old. A 7-year-old presents with encopresis. The NP might suspect: Urinary tract infection Laxative abuse Delayed maturation Constipation [The underlying cause of encopresis , repetitive soiling of stool by child was 4 years of age or older who should be potty train, is usually chronic constipation.] A mother brings her 4-year-old daughter, who just started attending preschool, to the health clinic. She tells the nurse practitioner that her child is complaining of burning and itching that started in the left eye. Within 2 days involved both eyes, and the child developed a runny nose and a sore throat. During the physical exam, the child's eyes appear injected bilaterally with no purulent discharge. The throat is red, the inferior nasal turbinates are swollen, and shotty nodes are palpated in front of each year. Which of the following is most likely: Viral conjunctivitis Viral conjunctivitis is often present with or without cold symptoms. Patient's complaint of itchy red eyes and may have clear discharge accompanied by peri-ocular lymphadenopathy. A mother of a 7-year-old boy tells the family nurse practitioner that his teacher has a complaint to her of her sons frequent episodes of daydreaming. The mother reports that sometimes when her son is at home, he seems not to hear her, seeming to "blank out" for a short period of time. Which of the following is most likely? A grand mal seizure A partial seizure An absence seizure An atonic seizure And absence seizure is a brief seizure that usually last less than 15 seconds. During the seizure, the child may appear not to be listening, to have blanked out, or to be day dreaming. A 16-year-old female patient is being treated for her first UTI. She had an allergic reaction with hives after taking sulfa as a child. Which are the following antibiotics would be contraindicated? Ampicillin TMP-SMX Cephalexin Macrobid TMP-SMX contains sulfa. A mother brings her 4-year-old daughter, we just started attending preschool, to the health clinic. She tells the nurse practitioner that her child is complaining of burning and itching that started in the left eye. Within 2 days involved both eyes, and the child developed a runny nose and a sore throat. During the physical exam, the child's eyes appear injected bilaterally with no purulent discharge. The throat is red, the inferior nasal turbinates are swollen, and shotty nodes are palpated in front of each year. Which of the following is most likely: Corneal ulcer Herpes keratitis Viral conjunctivitis Bacterial conjunctivitis Viral conjunctivitis is often present with or without cold symptoms. Patient's complaint of itchy red eyes and may have clear discharge accompanied by periocular lymphadenopathy. A 8-year-old child is seen as a walk-in appointment by nurse practitioner. The mother reports that the child is febrile for 2 days and is not eating well due to painful sores inside the mouth. During the physical exam, the nurse practitioner notices several small blisters and shallow ulcers on the child's pharynx and oral mucosa. The child has small round red rashes on both palms and soles. Which are the following conditions is most likely? Hand-foot-and-mouth disease Herpes simplex infection Varicella infection Secondary syphilis infection Hand foot and mouth disease is caused by coxsackie virus. Symptoms in the question align with this disease. A cauliflower-like growth with foul-smelling discharges seen during an otoscopic exam of the left ear of an 8-year-old boy with a history of chronic otitis media. The tympanic membrane and ossicles are not visible, and the patient seems to be having difficulty hearing the nurse practitioner's instructions. Which with the following conditions is best described? Chronic mastoiditis Chronic perforation of tympanic membrane with secondary bacterial infection Cholesteatoma Cancer of the middle ear An abnormal skin growth in the middle year behind the eardrum is called cholesteatoma. repeated infections and/or a tear or pulling inward of the eardrum can allow skin into the middle ear. A 15-year-old passable player who is 6 foot tall is seen for complaints of painful lumps on his knees. Upon inspection, the nurse practitioner notes a bone-like growth on the upper tibia midline below the kneecaps on both knees. The patient has full range of motion with no joint tenderness, redness, or swelling. Which are the following conditions is best described? Osgood Schlater disease Paget's disease of the bone Juvenile rheumatoid arthritis Osteosarcoma of the tibia Osgood Slater disease as characterized by pain over the tibial tuberosity with palpation of the bony mass over the anterior tubercle of one or both knees. Exercise worsens the pain. A 12-year-old with hip pain presents to the NP clinic. Hip pain has occurred with activity for the past 4-6 weeks, but his pain is worse and now involves the knee. There is no history of trauma. How should the workup be initiated? Order a hip x-ray and a sedimentation rate Perform Trendelenburg's test in the office Have the child squat in the office to assess the hips Order a hip and knee x-ray There are several diagnoses in the differential. The assessment of this child's pain should begin in the office. Asking the child to stand on the affected leg to perform the Trendelenburg test. If there are weak adductor muscles in the affected hip, a pelvic tilt will be visible in the affected hip. After assessment of the hip, knee, and gait in the office, a hip x-ray to include AP and lateral should be ordered. A 8-year-old boy with type 1 diabetes is being seen for 3 day history of urinary frequency and nocturia. He denies flank pain and is afebrile. The urinalysis results is negative for blood and nitrates but is positive for large amount of leukocytes and ketones. He has a trace amount of protein. Which is the following is best test to order initially? 24-hour urine for protein and creatinine clearance 24 urine for microalbumin Intravenous pyelogram A urine culture and sensitivity In 8-year-old male patient with diagnosis of diabetes has a high risk of UTIs. Urinalysis is showing possible UTI. The urine culture would be ordered because he has high risk of infection. A 14-year-old boy is brought in by his mother who reports that her son has been complaining for several months of recurrent bloating, stomach upset, and occasional loose stools. She reports that he has difficulty gaining weight and is short for his age. She has noticed that his symptoms are worse after eating large amounts of crackers, cookies, and breads. She denies seeing blood in the boy stool. Which are the following conditions is most likely? Malabsorption Celiac disease Amebiasis Crohn's colitis A 3-year-old has been recently treated for an upper respiratory infection but drainage from the right nostril persists. What should the NP suspect? Allergic rhinitis Dental caries Unresolved URI Presence of foreign body To clinical clue should make the examiner suspect a foreign body. First, the patient has continued drainage despite treatment. Second, the drainage is uniilateral. A mother brings her 4-year-old daughter, we just started attending preschool, to the health clinic. She tells the nurse practitioner that her child is complaining of burning and itching that started in the left eye. Within 2 days involved both eyes, and the child developed a runny nose and a sore throat. During the physical exam, the child's eyes appear injected bilaterally with no purulent discharge. The throat is red, the inferior nasal turbinates are swollen, and shotty nodes are palpated in front of each year. Which of the following is most likely: Corneal ulcer Viral conjunctivitis Bacterial conjunctivitis Herpes keratitis A 7-year-old has a complaint of ear pain. If he has otitis externa, which complaint is most likely? Tragal pain A 6-year-old complains that his leg hurt. His mother states that he has complained for the past 2 weeks, and she thought it was from playing outside too much. When asked to identify the painful areas, the child points to the mid shaft of the femur. He grimaces slightly when asked to walk. What is the most important initial intervention? Complete blood count Bone pain is common in children, especially adolescence. However, 6-year-old with complaints of mid bone pain should be evaluated for acute lymphoblastic leukemia. A CBC should be ordered initially to evaluate this child's white blood cell count, red blood cell couunt, and platelet count. A 7-year-old has a complaint of ear pain. If he has otitis externa, which complaint is most likely? Tragal pain Classic symptom of otitis externa history goal pain. Systemic symptoms don't usually accompanying otitis externa. Upper respiratory infection occurring concurrently is more likely to be otitis media in children especially. A 16-year-old female patient is being treated for her first UTI. She had an allergic reaction with hives after taking sulfa as a child. Which are the following antibiotics would be contraindicated? TMP-SMX TMP-SMX contains sulfa. T A 4-week-old boy seen in the clinic for a complaint of forceful vomiting that occurs immediately after feeding. The vomitus is composed of infant formula and is not bilious. The infant is bottle-fed with infant formula that was recommended by the pediatrician. The mother reports that the infant seems hungry and sucks on the bottle without any problems. His birth weight is 3.4 kg and his current weight is 3.2 kg. Which with the following clinical findings is an important clue regarding the possible cause of the infant's vomiting? Round olive like mass located in the right upper quadrant of the abdomen The stem is asking about the possible cause of the infant's vomiting, it is not asking about the symptoms. Projectile of forceful vomiting after feeding is a classic symptom of infantile hyperrtrophic pyloric stenosis. The other signs and symptoms are dehydration, irritabillity, crying. Positive rooting reflex is normal finding in a 4-week-old. A 14-year-old boy is brought in by his mother who reports that her son has been complaining for several months of recurrent bloating, stomach upset, and occasional loose stools. She reports that he has difficulty gaining weight and is short for his age. She has noticed that his symptoms are worse after eating large amounts of crackers, cookies, and breads. She denies seeing blood in the boy stool. Which are the following conditions is most Celiac disease Patients with celiac diseaseshould avoid foods with containing gluten, which causes malabsorption. Foods to avoid are weak, right, and barley. Oates to not damage the mucosa in celiac disease. A 6-year-old complains that his leg hurt. His mother states that he has complained for the past 2 weeks, and she thought it was from playing outside too much. When asked to identify the painful areas, the child points to the mid shaft of the femur. He grimaces slightly when asked to walk. What is the most important initial intervention? Complete blood count Bone pain is common in children, especially adolescence. However, 6-year-old with complaints of mid bone pain should be evaluated for acute lymphoblastic leukemia. A CBC should be ordered initially to evaluate this child's white blood cell count, red blood cell couunt, and platelet count. Week 14 – Musculoskeletal (MSK) A school-age child falls off a swing and fractures the humerus close to the elbow joint. What is the most important assessment for this patient to evaluate possible complications of this injury? Salter-Harris classification Palpation for joint laxity Evaluation of pain with extension The presence of a spiral fracture A patient has symptoms of carpal tunnel syndrome. Which diagnostic tests will help confirm this disorder? (Select all that apply.) Erythrocyte sedimentation rate Plain radiographs Anti-nuclear antibody Electromyography Nerve conduction studies A 62-year-old female presents with atraumatic right knee pain. On exam, she has mildly swollen right knee that is not warm or tender to touch. She has negative McMurray test. Her x-ray demonstrates tibial femoral joint space narrowing and visualization of osteophytes. How should the nurse practitioner interpret the results of this x-ray? Age-related changes in the knee A Baker's cyst Gouty arthritis Osteoarthritis In the knee, joint space narrowing indicates a loss of articular cartilage and usually worsening osteoarthritis. Osteophytes formed in response to degeneration of cartilage and joints. The McMurray test is commonly used to assess joint motion and meniscal injury. A patient presents with right shoulder pain after an acute shoulder injury yesterday. He fell against a brick wall while working at his home. He reports pain that radiates into his upper arm. How should this be managed? Prescribe physical therapy for the patient Ordered x-ray of the right shoulder Rest, ice, and naproxen for one week Immobilize the right shoulder for 3 days And x-rays generally the initial test used to evaluate trauma that results in acute pain. An x-ray could demonstrate fracture. It would not be helpful for evaluation of most soft tissue injuries. No other intervention is appropriate until fracture and dislocation have been ruled out. An adolescent athlete has injured his ankle playing basketball. He has right ankle pain, ecchymosis, and significant edema, and he is unable to bear weight at the time of the clinical exam. Which diagnosis is least likely? Grade 3 sprain Grade 2 sprain Grade 1 sprain An avulsion fracture A grade 1 sprain results from minimal stretching or small tears in the ligament. There is mild tenderness and edema, and the patient is able to bear weight. A grade 2 sprain is more significant, the clinical signs are more severe stretching and tearing of ligament with moderate pain, edema, tenderness and ecchymosis. Weightbearing is painful but the patient is able to walk. A grade 3 sprain is the most severe, and involves complete tear of the ligament, there is joint instability, severe pain, edema, tenderness, ecchymosis, patients are unable to bear weight due to pain. An adult patient who has been taking high-dose corticosteroids reports a dull, aching pain in the groin and presents with a limp. What condition does the provider suspect, based on this history? Avascular necrosis of the femoral head What are included in the initial management of bursitis of the heel? (Select all that apply.) Nonsteroidal anti-inflammatory medications Rest, ice, compression, and elevation Activity modification and bracing A patient comes to a provider with reports of unilateral arm pain and weakness with mild neck pain. The provider notes that the patient prefers holding the affected arm crossed in front of the throat. A history reveals a recent onset of sexual dysfunction. What does the provider suspect based on this history? Cervical myelopathy A patient takes HCTZ daily for hypertension. He developed severe pain in his great toe yesterday. He was diagnosed with gout today and started on the medication. Which medication listed below would be contraindicated at this time? Prednisone Colchicine Indomethacin Allopurinol Allopurinol has no role in treatment of acute gout because it has no role in inflammation. A 75-year-old female who Knits daily has a positive Finkelstein test. What is her likely diagnosis? Gamekeeper's thumb DeQuervain's tenosynovitis Trigger thumb Osteoarthritis of the thumb DeQuervain's tenosynovitis were present inflammation of the exterior and flexor tendon in the thumb. The classic finding includes a positive Finkelstein test. Pain can radiate up to the forearm. The probable precipitant is this patient's daily knitting. A long distance runner is diagnosed with a tibial stress fracture. Which stayed in the street about the injury? Rest and an alternative activity are recommended Plain x-ray will confirm the diagnosis The patient should be casted for 6 weeks The pain worsens with rest Stress factors are an example of an overuse injury. X-ray usually do not demonstrate stress fractures, so history and exam are important. Engaging in an alternative exercise that does not stress the fractured area is recommended. Casting and crutches are reserved for use when conservative treatment are not affected. A 45-year-old patient who is in good health presents with complaints of pain in his left heel. He states that the first few steps out of the bed in the morning are extremely painful. He has no history of trauma. What is the likely etiology of his pain? Calcaneal spur Arthritis of the foot Plantar fasciitis Achilles tendinitis Symptoms are classic for plantar fasciitis A 12-year-old with hip pain presents to the NP clinic. Hip pain has occurred with activity for the past 4-6 weeks, but his pain is worse and now involves the knee. There is no history of trauma. How should the workup be initiated? Perform Trendelenburg's test in the office There are several diagnoses in the differential. The assessment of this child's pain should begin in the office. Asking the child to stand on the affected leg to perform the Trendelenburg test. If there are weak adductor muscles in the affected hip, a pelvic tilt will be visible in the affected hip. After assessment of the hip, knee, and gait in the office, a hip x-ray to include AP and lateral should be ordered. A 8-year-old boy with type 1 diabetes is being seen for 3 day history of urinary frequency and nocturia. He denies flank pain and is afebrile. The urinalysis results is negative for blood and nitrates but is positive for large amount of leukocytes and ketones. He has a trace amount of protein. Which is the following is best test to order initially? A urine culture and sensitivity In 8-year-old male patient with diagnosis of diabetes has a high risk of UTIs. Urinalysis is showing possible UTI. The urine culture would be ordered because he has high risk of infection. A mother brings her 4-year-old daughter, we just started attending preschool, to the health clinic. She tells the nurse practitioner that her child is complaining of burning and itching that started in the left eye. Within 2 days involved both eyes, and the child developed a runny nose and a sore throat. During the physical exam, the child's eyes appear injected bilaterally with no purulent discharge. The throat is red, the inferior nasal turbinates are swollen, and shotty nodes are palpated in front of each year. Which of the following is most likely: Viral conjunctivitis Vital conjunctivitis is often present with or without cold symptoms. Patient's complaint of itchy red eyes and may have clear discharge accompanied by periocular lymphadenopathy. A 4-week-old boy seen in the clinic for a complaint of forceful vomiting that occurs immediately after feeding. The vomitus is composed of infant formula and is not bilious. The infant is bottle-fed with infant formula that was recommended by the pediatrician. The mother reports that the infant seems hungry and sucks on the bottle without any problems. His birth weight is 3.4 kg and his current weight is 3.2 kg. Which with the following clinical findings is an important clue regarding the possible cause of the infant's vomiting? Round olive like mass located in the right upper quadrant of the abdomen The stem is asking about the possible cause of the infant's vomiting, it is not asking about the symptoms. Projectile of forceful vomiting after feeding is a classic symptom of infantile hyperrtrophic pyloric stenosis. The other signs and symptoms are dehydration, irritabillity, crying. Positive rooting reflex is normal finding in a 4-week-old. A 8-year-old child is seen as a walk-in appointment by nurse practitioner. The mother reports that the child is febrile for 2 days and is not eating well due to painful sores inside the mouth. During the physical exam, the nurse practitioner notices several small blisters and shallow ulcers on the child's pharynx and oral mucosa. The child has small round red rashes on both palms and soles. Which are the following conditions is most likely? Hand-foot-and-mouth disease Hand foot and mouth disease is caused by coxsackie virus. Symptoms in the question align with this disease. A 50-year-old patient reports acute pain in his lower back that started 2 weeks ago after working in his yard. The pain radiates into his right leg intermittently. He has been managing his pain with ibuprofen. There are no red flags in history or on exam. When should consideration be given to imaging studies? There is no need Now At 4 weeks At 8 weeks Imaging studies are usually not indicated for uncomplicated acute low back pain. The probable precipitant, physical labor, like yard work, is a common cause of low back pain. As long this pain is of less than 4 weeks duration and there are no red flags in history or exam, radiological tests can be delayed. Imaging should be considered initially in patients who have a history of cancer, age greater than 50, significant trauma or neurological deficits, or pain in consistent with history. A 40-year-old woman reports pain at the thumb base in one hand radiating to the distal radius. The provider learns that the woman knits for a hobby and is able to elicit the pain by asking the patient to pour water from a pitcher. Which condition is suspected in this patient? Trigger finger Palmar fibrosis Tenosynovitis Carpal tunnel syndrome A patient has pain at the base of one thumb and reports frequently dropping things because of pain and weakness in that joint. During physical examination, what will the examiner do to help diagnose this condition? Adduct the first metacarpal and hyperextend the metacarpal phalanx Flex the thumb while placing a finger on the metacarpophalangeal joint Place the thumb on the palm while deviating the hand toward the ulna Passively extend the thumb and observe for puckering of the skin A patient with shoulder pain is seen by an orthopedic specialist who notes erythema, warmth, and fluctuance of the shoulder joint. What is the next step in treatment for this patient? Admit to the hospital for intravenous antibiotics Perform a shoulder ultrasound to further evaluate the cause Order a plain radiograph of the shoulder to identify possible fracture Inject lidocaine into the joint and reassess in 5 to 10 minutes An examiner is evaluating a patient who reports unilateral shoulder pain and notes limited active and passive range of motion in the affected shoulder along with erythema and bulging on the anterior shoulder. What diagnosis is likely with this presentation? Adhesive capsulitis Acromioclavicular joint disease Inflammatory bursitis Rotator cuff tear A 60-year-old patient is otherwise healthy presents with acute onset of right knee pain. She denies injury but reports that she walked up a lot of steps yesterday. She is diagnosed with prepatellar bursitis. Was a common finding? Posterior knee pain, anterior knee edema, and redness Tenderness to touch of the tibial tubercle Swelling and pain to touch in the anterior knee Limping and erythema above the anterior knee Prepatellar bursitis is often precipitated by an increase in activity involving the knees. If it's more infectious in origin, erythema would be present, but based and patient history and absence of risk factors that is less likely. Prepatellar bursitis is characterized by swelling and inflammatory changes anterior to the patella, with no symptoms posteriorly. What does a positive anterior drawer test demonstrated in a patient who has an injured knee? Instability of the anterior cruciate ligament Injury to the lateral meniscus Stability of the anterior cruciate ligament Stability of the lateral knee A patient has chronic elbow pain associated with arthritis. What is included in management of this condition? (Select all that apply.) Splinting of the elbow Avoidance of certain activities Long-term NSAIDs Occupational therapy Balanced rest and exercise A patient with elbow pain without localized erythema or warmth is diagnosed with bursitis of the elbow and serum laboratory results are pending. What is the initial treatment while waiting for these results? Physical and occupational therapy Corticosteroid injection into the bursal sac Aspiration of the bursal sac for culture Elbow pads, NSAIDs, rest, and ice A soccer player is brought to the emergency department after twisting an ankle during a game. An examination of the affected joint reveals ecchymosis and edema of the ankle and limited joint laxity along with pain on weight-bearing, although movement with pain is intact. Which grade sprain is likely? Grade II Grade IV Grade III Grade I A male patient who injured his back lifting a heavy object report that he has low back pain. He is diagnosed with a lumbar strain. He is afraid to continue activities of daily living, especially walking, because he has pain with these activities. What statement below is true? Stop performing all activities of daily living for 3 days Gradually increase activities of daily living and walking as tolerated Bedrest will help your back pain Continue your activities of daily living, but stopped walking, except to go to the bathroom Pain associated with a lumbar strain does produce pain with activity and walking. patient should be educated that some pain as expected and that it will not produce permanent injury. He should be encouraged to gradually increase activities of daily living and normal walking as tolerated. A high school soccer player sustains a knee injury when kicked on the lateral side of the knee by another player. The provider notes significant swelling of the knee, with pain at the joint line on the medial aspect of the knee. What will the provider do to treat this injury? Refer for a same-day orthopedic consultation Instruct about RICE management and follow up in 1 week Splint the knee and refer for orthopedic consultation in 1 to 2 weeks Schedule a magnetic resonance imaging (MRI) exam A 40-year-old nurse complains of new onset back pain secondary to her job. She reports lifting some obese patients while working the previous night shift. She reports to the Worker's Compensation clinic where she was referred. She describes the pain as starting in her right buttocks area and radiating down the back per thigh. It becomes worse when she sits down for long periods. You would suspect: Acute muscle spasm Sciatica Acute muscle strain Cauda equina syndrome Sciatica is defined as pain that begins in the buttocks area and radiates down one leg. Other symptoms include weakness and tingling. A long distance runner is diagnosed with a tibial stress fracture. What is the best conservative treatment recommendation? The patient should be casted for 6 weeks The pain worsens with rest Plain x-ray will confirm the diagnosis Rest and an alternative activity are recommended Stress factors are an example of an overuse injury. X-ray usually do not demonstrate stress fractures, so history and exam are important. Engaging in an alternative exercise that does not stress the fractured area is recommended. Casting and crutches are reserved for use when conservative treatment are not affected. A patient who is a distance runner reports pain in one heel that is worse in the morning and seems to improve with exercise. The provider notes localized swelling and a bony prominence at the heel. What is the initial treatment for this condition? Crutches and partial weight bearing Cessation of all sports activities and exercise Physical therapy for ultrasound therapy Referral to an orthopedist for MRI and evaluation A 16-year-old complains that his knee hurts. his mother states that he has complained of knee pain for the past 2 weeks. He has a prominent tibial tubercle. Which should be part of the differential diagnosis? Psychogenic pain Acute lymphocytic leukemia Growing pains Osgood-Schlatter disease Osgood Slater disease is in osteochondritis of the tibial tubercle that can produce pain in the knees of adolescence. pain gradually increases over time and can become severe. The diagnosis can be made on clinical presentation without the need for imaging studies. A patient has pain on the plantar aspect of the heel with weight bearing after rest. The pain is worsened with dorsiflexion of the foot. What is the initial treatment for this patient? Avoiding all high-impact activities A series of steroid injections Night splints Wearing flat shoes only A patient who has osteoarthritis in the carpometacarpal joints of both thumbs asks about corticosteroid injections to treat symptoms. What will the provider tell this patient about this therapy? Corticosteroid therapy reduces inflammation and improves joint mobility Intra-articular injections provide significant pain relief for 3 to 4 months This treatment may cause a temporary increase in pain, warmth, and redness Injections may be administered as needed up to 6 times per year A patient has an acute onset of lower back pain associated with lifting heavy objects at work. A physical examination reveals no loss of lower extremity function or neurological symptoms. What is the initial intervention for this patient? Treatment with a nonsteroidal anti-inflammatory drug An older adult with complaint of shoulder pain has positive "drop arm" test. What is his likely diagnosis? Torn rotator cuff A positive drop arm test should give the examiner good idea of the diagnosis of torn rotator cuff. An MRI confirms the diagnosis. A patient reports severe back pain located in the lumbar spine. To evaluate whether the patient has axial pain or radicular pain, which assessment is necessary? Assessing reflexes and asking about tingling or numbness Noting whether pain is mitigated with frequent position shifts Asking the patient to perform the Valsalva maneuver Determining whether the pain is present with prolonged sitting Patient reports that his knee "locks" sometimes and feels like it will "give out." She denies injury. She has no complaints about her other knee. What is her likely problem? Meniscal tear A 50-year-old woman reports pain in one knee upon awakening each morning that goes away later in the morning. A knee radiograph is negative for pathology and serum inflammatory markers are normal. What will the provider tell this patient? To take acetaminophen 1 gram three times daily for pain Which maneuver during a physical examination is used to assess the anterior cruciate ligament? Anterior drawer test An emergency department provider is giving instructions for rest, ice, compression, and elevation (RICE) treatment in a patient with a sprain. What is included in teaching about this home care? (Select all that apply.) Place a cloth between the ice pack and the skin Apply ice packs for 20 minutes three times daily ! Moist heat therapy may be applied if muscle spasm occurs Proximal joints should be elevated higher than distal joints An elastic bandage is sufficient for compression A patient injures an ankle while playing soccer and reports rolling the foot inward while falling with immediate pain and swelling of the lateral part of the joint. The patient is able to bear weight and denies hearing an audible sound at the time of injury. What does this history indicate? Mild soft tissue injury only Likely ankle sprain with a possible fracture Serious ankle injury with certain fracture Mild ankle injury without fracture A patient reports a deep ache in one shoulder and the provider suspects tendonitis secondary to repetitive activity. To determine whether the pain is caused by impingement on the acromion, the provider will ask the patient to abduct the arm. adduct the arm. internally rotate the shoulder. shrug the shoulders. A 45-year-old patient reports a recent onset of unilateral shoulder pain which is described as diffuse and is associated with weakness of the shoulder but no loss of passive range of motion. What does the provider suspect as the cause of these symptoms? Cervical radicular pain Acromioclavicular joint disease Glenohumeral arthritis Rotator cuff injury A previously healthy patient reports a sensation of one knee locking or feeling like it will give way when descending stairs. The patient has no recollection of injury to the knee and denies pain. What is the most likely treatment for this disorder? Conservative management with RICE and activity modification A patient reports elbow pain and the examiner elicits pain with resisted wrist flexion, forearm pronation, and passive wrist extension on the affected side. What is a likely cause of this pain? Medial epicondylitis A 60-year-old patient is otherwise healthy presents with acute onset of right knee pain. She denies injury but reports that she walked up a lot of steps yesterday. She is diagnosed with prepatellar bursitis. What is a common finding on physical exam? Tenderness to touch of the tibial tubercle Posterior knee pain, anterior knee edema, and redness Swelling and pain to touch in the anterior knee Limping and erythema above the anterior knee [Show More]

Last updated: 2 years ago

Preview 1 out of 333 pages

Buy Now

Instant download

We Accept:

We Accept
document-preview

Buy this document to get the full access instantly

Instant Download Access after purchase

Buy Now

Instant download

We Accept:

We Accept

Reviews( 0 )

$17.00

Buy Now

We Accept:

We Accept

Instant download

Can't find what you want? Try our AI powered Search

162
0

Document information


Connected school, study & course


About the document


Uploaded On

Jul 30, 2021

Number of pages

333

Written in

Seller


seller-icon
securegrades

Member since 5 years

118 Documents Sold

Reviews Received
24
3
3
0
5
Additional information

This document has been written for:

Uploaded

Jul 30, 2021

Downloads

 0

Views

 162

Document Keyword Tags

Recommended For You

Get more on EXAM »

$17.00
What is Scholarfriends

In Scholarfriends, a student can earn by offering help to other student. Students can help other students with materials by upploading their notes and earn money.

We are here to help

We're available through e-mail, Twitter, Facebook, and live chat.
 FAQ
 Questions? Leave a message!

Follow us on
 Twitter

Copyright © Scholarfriends · High quality services·